Pharmacology 1 1

You might also like

Download as pdf or txt
Download as pdf or txt
You are on page 1of 124

PHARMACOLOGY 1

PCOL 211

study of disease became clear and they discovered that


Part 1: INTRODUCTION TO diseases are caused by microorganisms. They began to
use traditional drugs.
PHARMACOLOGY ○ Penicillin - first discovered medicine (from Penicillium
notatum). It was accidentally discovered by Alexander
PHARMACOLOGY Fleming (used during WW1, and WW2, but later on,
● The science of drugs and their effects on biological resistance occurs)
systems ● Materia Medica - the science in preparation and medical
● What the drug does to the body (mechanism of action) use of drugs. The precursor of pharmacology.
● Studies the effect of drugs. Types of drugs: DRUG
A. Agonist
B. Competitive inhibitor ● is any substance that brings about a change in biological
C. Allosteric activator function through its chemical actions.
D. Allosteric inhibitor ● It is used in:
● Some of them can bind to a specific regulatory ○ Prevention (Vaccines and oral contraceptives)
molecule called the receptor. Once this receptor binds ■ Once the vaccine is given, it will attach to
to a drug, either agonist or antagonist, it will produce a immunoglobulins and the immune system will
pharmacologic response or effect and will respond recognize it, and they will make a copy of that
either agonistic or antagonistic. virus or bacteria, and once you are exposed
● Agonist drugs will cause a pharmacologic action either again in that same strain of bacteria or virus, they
positive or negative. will fight it.
● Inhibitors or antagonist drugs, once they bind to a ■ Oral contraceptives act on hormones.
receptor, their effect is usually inhibitory. ○ Diagnosis (BaSO4)
■ Barium sulfate (BaSO4) is a contrast media used
DIVISIONS OF PHARMACOLOGY in the x-ray of GIT.
● Clinical pharmacology - the application of pharmacology ○ Mitigation (Analgesics)
to the patient (patient-centered application). Explains to the ■ to slow down the response of the body to the
patient when the drug takes effect, what is adverse drug disease or to reduce the symptoms.
reaction, toxic dose, etc. ○ Treatment / Cure (Antibiotics)
● Neuro pharmacology - studies about the medication on ■ Antibiotics are drugs that fight microorganisms.
the central and peripheral nervous system. They will eliminate or reduce the growth of
○ Ex: Antipsychotics, Parkinson’s, Alzheimer's. microorganisms.
● Pharmacogenetics / Pharmacogenomics - the study of Nature of Drugs
genetic variations that cause differences in drug response.
The relation of the individual’s genetic makeup to his or her 1. Size and Molecular Weight
response to specific drugs. ● Sizes from MW 7 (Lithium) to over MW 50,000
○ For example, Asians are fast acetylators (can rapidly (thrombolytic enzymes, other proteins)
metabolize isoniazid). Caucasians are slow ○ Low MW (7) can pass through our small molecule
acetylators. or in the blood-brain barrier, which is very
● Pharmacoepidemiology - the study of the effects of drugs lipid-soluble.
on a large number of people. ■ Lithium - former drug of choice for bipolar
○ For example, the spread of resistance to one specific disorder
drug to its other same-category drugs. ○ Thrombolytic enzymes and other proteins (MW
● Toxicology - the study of adverse effects, and toxic doses 50,000) can only bind in large molecules like
of drugs. blood components, and clots.
● Posology - the study of dose. It depends on various ■ Thrombo means ‘clot’ and lytic means ‘to
factors such as age, climate, weight, and sex. break down’ (for vein blockage).
● Pharmacognosy - the study of the special composition of ● Majority of drugs have MW between 100 and 1000.
the plant (constituents such as flavonoids, alkaloids, etc) or ○ Drugs smaller than 100 are rarely sufficiently
biological origin where we can get or make drugs. selective because they can pass through small
● Environmental pharmacology - a new discipline and its molecules including lipid-soluble.
focus is being given to understand gene-environment, ○ Drugs larger than 1000 are often poorly absorbed
drug-environment, and toxin-environment interaction. because they are too large to be absorbed fast.
○ It involves how we properly dispose of drugs. 2. Drug-Receptor Bonds
● When a drug binds to a receptor, it produces a
HISTORY pharmacologic action. Depending on bonds on how
● 1,500 years - people used to believe that a disease was long the medicine lasts in the body.
caused by excess bile or excess blood. ● Covalent bonds form strong bonding with the receptor
○ Acupressure and usually causes an irreversible or permanent
○ Acupuncture action. It causes a toxic response; it lasts longer or
○ They are used to apply ointments, salve, and leaves. stays in the body like poisonous drugs.
● 17th Century - Around the end of the 17th century, the
PCOL 211 - Pharmacology 1

● Ionic bonds are somewhat weak. It forms a reversible ■ Insulin is given to a diabetic person to reduce
action (not permanent). blood sugar
● Hydrogen bonds are a much weaker interaction than ■ Estrogen (females). Excess of estrogen can
covalent and ionic. It forms a weak bond and is easier cause contraceptive effects.
to eliminate in the body. ■ Testosterone (male). Excess of testosterone
● Van der Waals is also a much weaker bond and can can cause contraceptive effects.
be easily eliminated. B. IV Fluids/Electrolyte/Oral Rehydration Solution (ORS)
■ Electrolytes contain Na, K, and Ca
● Kapag kinulang sa electrolytes, hindi
gagana ang nerves. Kapag hindi gumana
ang nerves/neurons, hindi magfufunction
ang brain. Kapag hindi nag function ang
brain, ang buong katawan ay pwedeng
mag shut down.
● Dehydration can cause death.
■ Dextrose contains NaCl
C. Multivitamins
■ For the person who has micronutrition problems
or is malnourished.
■ Contains Vitamin C, B complex, Iron, and Folic
3. How are drugs named?
acid (Vitamin B9)
● 2-(p-isobutylphenyl) propionic acid = Chemical name
● In the cause of pernicious anemia (autoimmune disease;
● Ibuprofen = Generic name (nonproprietary name),
lacks in RBC) that destroys parietal cells which secrete
International Nonproprietary Name (INN) or United
HCl and intrinsic factors, take Vitamin B12.
States Adoptive Name (USAN)
○ Kapag anemic, kulang ang distribution ng energy at
○ Analgesic (painkiller)
nutrition, nagiging unhealthy.
○ Can cause ulcer; after taking Ibuprofen, you
● Pernicious anemia may also be caused by PPIs, H2
should eat after 30 minutes
Blockers, and Fish Tapeworms.
○ Can cause an anti-inflammatory effect which
○ PPIs - proton pump inhibitors (for ulcers)
reduces mucus in the stomach
Ex: Omeprazole
● Mortin® = Trade name (proprietary name/brand name)
■ Reduces acid = reduced breakdown of food.
● Chemical name - based on the chemical composition ■ Ang Vitamin B Complex ay nakukuha sa mga
of the drug. kinakain natin. Kapag bawas ang acid sa
● Generic name - based on the chemical name stomach, bawas din yung pag breakdown ng
● Trade name - based on the manufacturer’s wants
food kaya kumokonti yung Vitamin B12 sa body
natin, which can result to pernicious anemia.
CLASSIFICATION OF DRUGS ACCORDING TO USE ○ H2 Blockers - Histamine 2 Blocker - also reduces
acid production
1. Functional Modifiers
○ Fish Tapeworms - Diphyllobothrium latum
● Alter or modulate the normal physiologic functions ■ Absorbs nutrition in the intestine, including
○ Drugs could change biological function (normal Vitamin B12
response of the body when sick) ● Low acid = low breakdown of food = low B12 → pernicious
● Examples: anemia
A. Analgesic drugs - reduces pain ● Vitamin B Complex, should be given when having ulcer,
■ Mefenamic gastritis, and reflux.
■ Ibuprofen
Vitamins that are important to take:
■ Paracetamol ● FeCCaDSeE
B. Anti-pyretic drugs - pyrexia means ‘fever’ ○ Ferrous sulfate + Vitamin C, Calcium and
■ Paracetamol (Acetaminophen - Tylenol® in US) Vitamin D, Selenium and Vitamin E
C. Anti-inflammatory drugs
■ NSAID - Non-Steroidal Anti-inflammatory Drugs 3. Diagnostic Agents
(Ex: aspirin)
● Agents used to determine the presence or absence of a
■ Steroids (Ex: corticosteroids)
condition or a disease.
D. Anti-hypertensive drugs - for high blood pressure
● Examples:
■ Amlodipine
A. Edrophonium (Tensilon’s Test)
■ Losartan
B. Histamine
2. Replenishers ■ Uses allergens to test if there is an allergic
reaction to the medication
● Supplement endogenous substances that are lacking or
■ “Skin Test” - intradermally
deficient in the body.
C. Some Radiopharmaceuticals
● Examples:
■ BaSO4
A. Hormones
■ Tc 99m (Technetium 99m) - for diagnosis of
Ischemia (vein blockage)

2
PCOL 211 - Pharmacology 1

● Ischemia - reduction of O2 supply because ○ Absorption (How will it get in?)


of clot formation inside a blood vessel. ○ Distribution (Where will it go?)
Kapag may clot ang blood vessel, hindi ○ Metabolism (How is it broken down?)
nagpa-pass through ang dugo, kapag ○ Excretion
walang presence ng RBC, reduce ang
■ Metabolism and excretion = elimination (the phase of
oxygen supply.
inactivation of a drug or removal from the body when
■ Thallium 209 - for the diagnosis of Infarction
metabolism and excretion are together)
(heart attack)
■ Biodisposition = sometimes used in the process of
D. Barium sulfate
metabolism and excretion
■ Pinapainom para kapag nag xray, nag-iiluminate
o umiilaw yung GIT.
E. Dobutamine and Dipyridamole for “Pharmacologic LIBERATION
Stress Testing” for CAD (Coronary Artery Disease) ● The release of active ingredient from its dosage form
and MI (Myocardial Infarction) ● It is affected by:
■ Stress Test is an electrocardiographic test (using 1. Disintegration property of the drug
a treadmill) if the heart has a normal capacity to ● The breakdown of a drug, reduction of particle
supply blood in the different parts of the body size, and enhancement of surface area so later
before, during, and after a controlled period of on it can be absorbed in the body.
increasingly strenuous exercise. 2. Dissolution - rate-limiting step in the absorption of
■ Dobutamine - beta-1 agonist drug or activates solid dosage forms and consequently onset, intensity,
beta-1 receptor found in the heart and increases and duration of action of the drug.
the function of the heart. ● In solutions or liquids, there is no mo dissolution.
● Ginagamit para patibukin ang puso ng tao.
Factors that Influence the Dissolution Rate
■ Dipyridamole - anti-platelet drug (inhibitor of clot
formation) 1. SURFACE AREA - the smaller the particle size = the
larger the surface area = the easier/faster it will be
4. Chemotherapeutic Agents
absorbed by the body.
● Agents used to kill or inhibit the growth of cells or nucleic ● Dahil maliliit yung particle size, mabilis siyang
acid considered as foreign to the body. Selective toxicity mapunta sa iba’t ibang parte ng katawan kaya
is considered. lumalawak yung surface area kasi nadistribute na
○ Lahat ng hindi parte ng katawan natin, basta pumasok siya. And because it is widely distributed, mabilis ang
at nag cause ng sakit, papatayin o irereduce ng absorption.
chemotherapeutic agents. ● Kapag large molecule, naka stay lang siya sa isang
○ Selective toxicity - kung ano lang ang abnormal sa parte ng katawan at doon lang siya mag take effect or
katawan, yun lang ang i-eliminate nila. doon lang possible maabosrb.
● Examples: 2. SALT FORMS - dissolve much more readily when
○ Anti-infectives (Antibiotics or Antimicrobial) compared to the drugs in their free form
■ Antibacterial ● Salt can be easily solubilized of liquids or fluids.
■ Antiviral Humans have 75-90% of fluids in the body.
■ Antifungal ● Ex: Losartan Potassium, HCl salts (can be absorbed
■ Antiparasitic easily, especially in GIT)
■ Anthelmintic 3. STATE OF HYDRATION
■ Antiamoebic ○ Anhydrous form is more readily soluble than the
○ Antineoplastics (Anti-cancer) hydrated form.
■ Pati yung katabi na normal cell napapatay. Wala ■ Anhydrous example: Powder forms (ready soluble
pang nadidiscover na very selective anti-cancer. because of smaller particle size)
○ Not all hydrated form contains water, it can be oil.
4. CRYSTAL FORM OR AMORPHOUS DRUG FORMS
Part 2: REVIEW OF PHARMACOKINETICS ○ Ex: Chloramphenicol palmitate is inactive in crystalline
form because it is not readily soluble but when
administered in amorphous form, like powder form,
Two Main Areas of Pharmacology absorption in the GIT is rapid and with good
therapeutic response
1. Pharmacokinetics - what the body does to the drug
○ Ex: Insulin
(LADMER system)
a. Short-acting - Semilente (100% amorphous)
2. Pharmacodynamics - what the drug does to the body
b. Intermediate - Lente (30% amorphous and 70%
(Mechanism of action)
crystalline)
PHARMACOKINETICS c. Long-acting - Ultralente (100% crystalline; it takes
● The study of the fate of drugs in the body 24 hours before well-absorbed)
● Drug disposition - the way in which the body handles ABSORPTION
drugs
● Rate and extent of drug entry into systemic circulation
● What is the fate of the drug in vivo?
○ Liberation

3
PCOL 211 - Pharmacology 1

○ Bioavailability - the proportion of a drug that is


5. Use of motility-enhancing drugs
delivered to its site of action in the body which will - Senna (SenokotⓇ) = for defecation
produce the pharmacologic action. 6. Diabetes mellitus
- Diabetic person takes oral hypoglycemic
Factors A ecting Absorption
agents (Metformin)
1. DOSE SIZE ADMINISTERED - the larger the dose, the - Side effect of Metformin: diarrhea
higher the drug absorption.
2. DEGREE OF PERFUSION OF THE ABSORBING 6. DOSAGE FORM - solid drugs are not well-absorbed, do
ENVIRONMENT - the more the blood supply = the greater not have 100% bioavailability because it will undergo
extent/faster absorption LADMER which decreases bioavailability, unlike parenteral
● Perfusion - high blood supply (blood distributes drugs, they are directly administered in blood vessels.
nutrition, drugs, and energy) Because it is well-perfused, it can be easily delivered in
● The IV, parenteral drugs have 100% bioavailability. different parts of the body.
3. AREAS OF ABSORBING SURFACE - many absorbing ● Solid drug → Dissolution
surfaces = increase absorption ● Drug in solution → Absorption
● The microvilli in the small intestine absorbs nutrition, 7. DRUG SOLUBILITY - for a drug to be absorbed, its
drug, and fluid. solubility must correspond to the characteristics of the
4. pH OF THE ABSORBING ENVIRONMENT absorption site.
● ‘Like dissolves like’ ● If a drug is acidic, it is well-absorbed in an acidic
● Acid + acid or Basic + basic = absorbed environment. A basic drug is well-absorbed in basic
● Acid + basic or Basic + acid = excrete/eliminated environment.
● Example, aspirin is easily absorbed when passing 8. FIRST-PASS EFFECT - some drugs are partially
through the stomach because they are acids. There metabolized in the liver or portal vein before passing into
are drugs that are inserted in the intestine because circulatory system.
they are similar in the environment. ● It removes 50% of the bioavailability of orally taken
drugs.
a. If we put an acidic drug in an environment with a lot of ● It is very helpful to remove toxic materials (coverage
H+ (low pH) what will this equilibrium do? of tablets like plastics, binders, glues, starch,
● It will be absorbed (low pH is acid)
colorants, and flavorants)
○ pH more than 7 is basic, pH less than 7 is
acidic. ● Sublingual, buccal, transdermal patches, vaginal-type
● Lot of hydrogen ions form an acid. dosage form, rectal, intramuscular, intravenous,
● Non-ionized form predominates. subcutaneous, intranasal, inhalation do not undergo
b. Aspirin is an acidic drug. In the stomach will exist mostly first-pass effect
in ionized or non-ionized form? 9. ENTEROHEPATIC RECYCLING - some drugs travel intact
● Non-ionized because aspirin and stomach have through the biliary tract after initial absorption and are then
the same pH (acidic)
○ Ionized - evaporates/removed reabsorbed into the bloodstream through the intestine and
○ Non-ionized - stays/absorbed it will decrease its effect or bioavailability.
● Example are drugs that are inserted in the intestine
5. GASTRIC EMPTYING - the time it takes for the stomach (suppositories)
to empty its contents ○ Kapag nag-insert ng suppository, dapat nasa
● Can increase or decrease (Pwedeng magtagal o gitna ng rectum. Kapag lumagpas at nakarating
pwedeng bumilis) sa kaloob-looban ng intestine, there are so many
● Orally taken drug takes time to stay in the stomach, blood vessels sa intestine that can distribute or
and will go to the small intestine, then to the large transport the drug papunta sa liver, and will
intestine. undergo enterohepatic recycling.
10. ROUTES - a drug’s route of administration affects its rate
Factors that increase gastric emptying time: (Pinapahaba o
and extent of absorption.
pinapatagal)
1. Stress ● Enteral - oral, sublingual, buccal, rectal
2. Heavy exercise ○ A drug is absorbed into the systemic circulation
3. Gastric ulcers through the oral or gastric mucosa, the small
4. Hot food (protein-rich, lipid-rich foods) intestine, or the rectum.
5. Lying on the left side ○ Oral - 50% bioavailability (undergoes first-pass
6. Drugs that inhibit gastric motility effect)
- Antidiarrheal drugs (pinipigilan yung movement
ng peristalsis ng GIT) ○ Sublingual, buccal, and rectal - 90-100%
Factors that decrease gastric emptying time: (Pinapaikli o bioavailability (does not undergo first-pass effect)
pinapabilis) ● Parenteral - IV (fastest), IM, SC, ID, others
1. Mild exercise ○ Because they will be inserted in a highly-perfused
2. Gastrectomy area like the bloodstream, they will be distributed
- A surgical procedure where they connect the easily and will be absorbed faster.
stomach directly to almost the end of the small
● Topical - skin (including transdermal patch), eyes,
intestine or straight to the large intestine. It
could be done to obese people. ears, nose, lungs (inhalation), vagina
3. Cold foods/drinks ○ 100% bioavailability
4. Lying on the right side

4
PCOL 211 - Pharmacology 1

11. TRANSPORT MECHANISMS - means of movement of ○ Pinocytosis - cell drinking. Requires energy
drug molecules across the cell membrane (ATP). For hydrophilic molecules or water-loving.
■ Griseofulvin
Specific Modes of Drug Transport (Permeation)
■ Fat-soluble vitamins
1. Passive Diffusion - from higher concentration to lower ● Exocytosis - an intracellular vesicle or
concentration. The movement of small molecules along the membrane-bounded sphere moves to the plasma
concentration gradient (fast). It does not require energy. membrane, and subsequent fusion happens.
● Going with the gradient
2. Carrier-Mediated Transport - for passage through the
membrane of large lipid-insoluble molecules and ions.
They need carriers.
● Types:
○ Facilitated diffusion - it is similar to passive
diffusion, it moves along the concentration
gradient from higher to lower.
○ Active transport - from a lower concentration
5. Ion-Pair Transport - quaternary ammonium compound (+)
gradient, it needs the energy (ATP) to be
and mucin (-) = unionized (no charge) so it can easily pass
transported to the higher concentration gradient.
through the cell membrane
It is going against the gradient. Characterized as
● Unionized - absorbed; they are usually lipid-soluble,
pushing a rock uphill.
and that is the reason why they can pass through the
cell membrane.
● Example is Propranolol (beta-blocker used as an
anti-hypertensive and stage fright). Should not be
given to an asthmatic patient because it can cause
anaphylactic shock.

DISTRIBUTION
● The transport of a drug in the body by the bloodstream to
its site of action.
● The extent to which the drug passes into different tissues
and fluid compartments in the body.

Physiologic Factors A ecting Distribution


● Protein binding
● Water-soluble vs fat-soluble
● Blood-brain barrier
● Size of the organ which determines the concentration
3. Convective (Pore) Transport - a small drug molecule gradient between blood and the organ
moves along with fluid through the pores in cell wall. This ○ For example, the brain is small and a small amount of
is mediated through water-filled pores channels or drug will go into it. Because the brain (skull) is small,
aquaporins. only small molecules will pass through it, and only
● Examples: those that are lipid-soluble can pass through it,
○ Inorganic and organic electrolytes up to 150 to especially there is a blood-brain barrier that protects
400 MW our brain.
○ Ions of opposite charge of pore-lining ○ Sa malalaking organs katulad ng skin, kung marami
○ Ionized sulfonamides kang ilalagay na patch, mas marami rin yung
magiging distribution.
● Cardiac Output - similar to perfusion
○ If a person has a high cardiac output, has a highly
perfused organ, it will be distributed faster and will be
absorbed well.
○ Normal = 2.2 - 3.5 liters/min/square meter
○ Low cardiac output in the case of congestive heart
failure → low distribution (kasi konti yung dugo na
lalabas sa puso) = slower onset of action of drugs
(kasi hindi sila madidistribute, hindi nagiging highly
4. Vesicular Transport perfused yung mga organs dahil kulang yung cardiac
● Examples are fats, glycerin, starch, parasite eggs, output.
plastic particles, hairs and yeast cells, ferritin, and ○ Liver, kidneys, and brain - 25% cardiac output.
insulin Lungs - 100% cardiac output
● Endocytosis - cells absorb molecules such as proteins Adipose tissues and bones - <1% cardiac output
by engulfing them (cell eating). ○ Pneumonia: 7-14 days treatment because 100%
cardiac output

5
PCOL 211 - Pharmacology 1

○ Osteomyelitis (namamaga yung mga adipose tissue, ● α1 globulin bind to a number of steroidal drug cortisone,
buto): 6 weeks to 6 months treatment because of very prednisolone, thyroxine, cyanocobalamin
low cardiac output. ● α2 globulin (ceruloplasmin) bind to Vitamin A, D, E, K
● Regional Blood Flow - fraction of cardiac output that is ● β1-globulin (transferrin) bind to ferrous ion
delivered to specific tissues/organs ● β2-globulin bind to carotenoid
○ Areas of high blood flow (highly perfused): heart, liver, ● y-globulin - bind to antigen
kidneys, brain - drugs are easily distributed, and can
Factors a ecting protein binding
be easily treated in a short period of time.
○ Areas of low blood flow (low perfusion): muscle, skin, 1. The drug itself
fat, bone - drug distribution takes a week and long 2. The protein itself
treatment. 3. Affinity between the drug and the protein
● Bawal pinag ssabay ang gamot
PARAMETERS
● For example, Warfarin and Aspirin. Si Aspirin, pwede
● Volume of Distribution niyang i-displace or paalisin sa protein yung Warfarin.
● Protein Binding Kapag nadisplace yung Warfarin ng mas high
● Blood-Brain Barrier and Placental Barrier protein-bound, may tendency na mag produce ng
toxic effect, which is bleeding.
1. Volume of Distribution (VD)
4. Drug interactions
● The hypothetical volume of body fluid necessary to 5. Physiologic condition of the patient
dissolve a given amount or dose of a drug to achieve a
3. Special Barriers
concentration equal to that of the drug plasma
concentration. ● Placental: It protects the fetus from external factors. Most
● Depende sa body size. In real life, mas matagal malasing small molecular weight drugs cross the placental barrier
yung medyo malalaki ang size o mataba, dahil mas large (can cause abnormality), although fetal blood levels are
yung volume of distribution nila unlike sa mapayat na mas usually lower than maternal.
mabilis malasing kasi konti lang yung areas na ○ That is why pregnant women are not required to take
pagdidistribute-an nung alak. different kinds of drugs other than vitamins and
● High VD: supplements.
○ Basic drugs ○ Drugs have 100-1000 MW and can cross placenta.
○ Examples: atropine, chloroquine, raloxifene ● Blood-Brain: It is permeable only to lipid-soluble drugs or
● Low VD: those of very low molecular weight.
○ Acidic drugs ○ Lithium (7 MW), lipid-soluble like antipsychotics
○ Examples: chlorpropamide, tolbutamide
● Drugs with a very small VD that is <10L are mainly Prediction of Extent of Distribution
confined to the intravascular fluid.
● Drugs with a high VD are orally taken and intramuscular. BODY FLUID % BODY 70 KG BODY
WEIGHT WEIGHT
2. Protein Binding
Total Body Fluid 60% 42 Liters
● It is the phenomenon that occurs when a drug combines
with plasma (particularly albumin) or tissue protein to form 1. Intracellular 40% 28 L
a complex.
○ Drug + protein = drug-protein complex 2. Extracellular 20% 14 L
○ When a drug binds to a protein like albumin, it will be
inactivated and will stay in the body. 2.a. Intravascular (High MW 5% 3-4 L
■ Kaya minsan once or twice a day and pag take ng and high protein bound drugs
gamot kasi nakakapit pa sila sa mga proteins like Heparin)
natin, kaya tumatagal na 6 or 8 hours nasa loob
2.b. Interstitial (Low MW and 15% 10-11 L
pa ng katawan, tapos kailangan na ulit uminom hydrophilic drugs like
ng panibagong gamot. Aminoglycosides)
Acidic drugs Albumin
METABOLISM
Basic drugs Alpha acid glycoprotein
● Biotransformation, inactivation, and detoxification.
Hormones Globulin ● It removes almost 50% of the drug content or the toxic
substances of the food and drug that we take.
Lipids or proteins Lipoproteins ● Why is drug biotransformation necessary?
○ To terminate or alter biologic activity, to remove toxic
Exogenous/endogenous Erythrocytes (RBC) materials
compounds
Drug Biotransformation Reactions
● Active Drug can be converted after metabolism into:
Free drug - opposite of protein-bound drugs.
- Dumidiretso sa bloodstream ○ Polar Metabolite
- Distributed faster ○ Inactive Metabolite
○ Active Metabolite - especially is prodrug

6
PCOL 211 - Pharmacology 1

○ Reactive Metabolite inadd sa kanila. Therefore, kumokonti yung effect o


● Prodrug (inactive) can be converted after metabolism into: concentration ng gamot na inadd sa kanila.
○ Active Metabolite ○ Examples:
● Xenobiotics - other term for drugs. ■ Carbamazepine
■ Phenobarbital
Active Drug to Reactive Metabolite
■ Phenytoin
■ Rifampicin
■ Smoking
■ Chronic
■ Alcoholism
● Enzyme Inhibitors - decrease the metabolic activity of an
enzyme, increase the pharmacologic action of
co-administered drugs. (They aren’t going anywhere)
○ Pampabagal ng metabolism. Kapag inadd sa ibang
gamot, nagii-stay yung gamot sa loob ng katawan na
inadd sa kanila. Therefore, nagiging cause na maging
toxic yung added drug dahil sa tulong ng mga enzyme
inhibitors.
○ Examples:
■ Cimetidine
■ Ketoconazole
Where do drug Biotransformations occur? ■ Chloramphenicol
■ Disulfiram
● Liver - major site ■ Grapefruit juice
● Extrahepatic microsomal enzymes ■ Acute alcoholism
○ Brain, lungs, intestines, bladder
○ Oxidation, conjugation Phases of Drug Metabolism
● Hepatic microsomal enzymes ● Phase I - makes the drug more polar or more
○ Liver water-soluble, adds a chemically reactive group (a
○ Oxidation, conjugation ‘handle’) permitting conjugation (‘functionalization’).
● Hepatic non-microsomal enzymes ○ Oxidation
○ Liver ○ Reduction
○ Acetylation, sulfation, GSH or glutathione, ○ Hydrolysis
alcohol/aldehyde dehydrogenase, hydrolysis, ● Phase II - adds an endogenous compound increasing
oxidation/reduction polarity to be easily removed from the body.
First Pass Metabolism ○ Conjugation
■ Glucuronyl
● The phenomenon whereby drugs may be metabolized ■ Sulphate
following absorption before reaching systemic circulation ■ Methyl
● a.k.a. “First Pass Effect” ■ Acetyl
● Examples: ■ Glycyl
○ Morphine ■ Glutathione groups
○ Meperidine ● Some drugs are excreted unchanged.
○ Pentazocine
○ Catecholamines Possible routes for the metabolism of drugs
■ Norepinephrine
■ Epinephrine
■ Dopamine
■ Serotonin
○ Propranolol
○ Beta-blockers (olol)

Enzyme Inducers and Enzyme Inhibitors


● Enzyme inducers and inhibitors should be always
considered if a person or patient is to take two drugs to tell
them what is the time interval of their medications.
● Enzyme Inducers - increase the metabolic activity of an
enzyme, and decrease the pharmacologic action of
co-administered drugs. (They move fast)
○ They can fasten up the metabolism which results if a Phase I Reactions
drug is added to them, kapag sinabay sila sa ibang
● Reactions that convert the parent drug to a more polar
gamot, pinapabilis nila yung metabolism ng gamot na
(water-soluble) or more reactive product by unmasking or

7
PCOL 211 - Pharmacology 1

inserting a polar functional group such as -OH, -SH, or


3A4 Acetaminophen, Barbiturates, Azamulin,
-NH2. alfentanil, amiodarone, carbamazepin- diltiazam,
● a.k.a. “Functionalization Reaction” astemizole, cisapride, e, erythromycin,
A. Oxidation cocaine, cortisol, glucocorticoids fluconazole,
cyclosporine, dapsone, , macrolide grapefruit juice
B. Reduction diazepam, antibiotics, (furanocoumari
C. Hydrolysis dihydroergotamine, pioglitazone, -ns)
dihydropyridines, phenytoin, itraconazole,
a. Oxidation diltiazem, rifampin ketoconazole,
erythromycin, ethinyl ritonavir,
1. CYP 450 Mediated - one of the main and popular enzyme estradiol, gestodene, troleandomyci-
that metabolizes drugs or foods indinavir, lidocaine, n
lovastatin, macrolides,
methadone,
CYP Substrates Inducers Inhibitors micronazole,
midazolam,
1A2 Acetaminophen, Smoking, Galangin, mifepristone (RU 486),
antipyrine, caffeine, charcoal- furafylline, nifedipine, paclitaxel,
clomipramine, broiled foods, fluvoxamine progesterone,
phenacetin, tacrine, cruciferous quinidine, rapamycin,
tamoxifen, vegetables, nitonavir, saquinavir,
theophylline, warfarin omeprazole spironolactone,
sulfamethoxazole,
2A6 Coumarin, tobacco Rifampin, Tranylcypromi- sufentanil, tacrolimus,
nitrosamines, nicotine phenobarbital ne, tamoxifen, terfenadine,
(to cotinine and menthofuran, testosterone,
2-hydroxynicotine) methoxsalen tetrahydrocannabinol,
triazolam,
2B6 Artemisinin, bupropion, Phenobarbital, Ticlopidine, troleandomycin,
S-mephobarbital, cyclophospha- clopidogrel verapamil
cyclophosphamide, mide
S-mephenytoin
(N-demethylation to Propofol (2B6) - anaesthetic
nirvanol) propofol, Bupropion (2B6) - smoking deterrent
selegiline, sertraline
Taxol (2C8) - anti-cancer
Trimethoprim (2C8) - antibiotic
2C8 Taxol, all-trans-retinoic Rifampin, Trimethoprim
acid barbiturates Celecoxib (2C9) - pain reliever
Ibuprofen (2C9) - pain reliever
2C9 Celecoxib, flurbiprofen, Barbiturates, Tienilic acid, Diazepam (2C19) - anti-convulsant (epileptic)
hexobarbital, rifampin sulfaphenazole Dextromethorphan (2D6) - cough syrup
ibuprofen, losartan, 3A4 - metabolizes almost all of the drugs
phenytoin, tolbutamide,
trimethadione, Smoking and other drugs can still affect controlled-released
sulfaphenazole, drugs.
S-warfarin, ticrynafen
● Types of reaction:
2C18 Tolbutamide Phenobarbital
➢ Hydroxylation (Phenytoin - anti-convulsant)
2C19 Diazepam, Barbiturates, N3-benzylnirva ➢ N-Dealkylation (Morphine - addictive)
S-mephenytoin, rifampin -nol, ➢ O-Dealkylation (Codeine)
naproxen, nirvanol, N3-benzylphen ➢ N-Oxidation (Nicotine)
omeprazole, -enobarbital,
propranolol fluconazole ➢ S-Oxidation (Thioridazine)
➢ Deamination (Amphetamine)
2D6 Bufuralol, bupranolol, St. John’s Quinidine,
clomipramine, wort, rifampin paroxetine
2. Non-CYP 450 Mediated
clozapine, codeine, a. Xanthine Oxidase - metabolizes purines (and causes
debrisoquin, to produce uric acid)
dextromethorphan, ● Purine and pyrimidine are nucleoside bases
encainide, flecainide,
fluoxetine, guanoxan, ● Peanuts contain purine
haloperidol, b. Flavin Monooxygenase (Ziegler’s enzyme)
hydrocodone, c. Amine Oxidases (Epinephrine - is a catecholamine
4-methoxy-amphetami-
ne, metoprolol, and part of the body)
mexiletine, oxycodone, d. Dehydrogenase (Ethanol)
paroxetine,
phenformin, b. Reduction
propafenone,
propoxyphene, ● The addition of hydrogen atoms or gain of electrons.
risperidone, selegiline ● Ketones and aldehydes can be converted to primary
alcohol because of reduction.
2E1 Acetaminophen, Ethanol, 4-Methylpyraz- ● Carbonyl reduction - Acetohexamide
chlorzoxazone, isoniazid ole, disulfiram
enflurane, halothane, ● Azoreduction - Sulfasalazine
ethanol (a minor ● Nitroreduction - Clonazepam (anti-convulsant)
pathway) ○ Azo and nitro groups are reduced to form amines

8
PCOL 211 - Pharmacology 1

○ Chloramphenicol (can cause Gray Baby Syndrome if


● Primary alcohol converts to aldehyde after oxidation to
acid. The acid will be reduced to form an aldehyde to a taken by 2 years old and below. Infants don’t have
primary alcohol. glucoridination and only Phase I metabolism is
● Secondary alcohol - ketones developed)
● Tertiary alcohol ● Sulfation
● Acetylation - N-acetyltransferase enzyme (Sulfonamides,
C. Hydrolysis Isoniazid, Hydralazine, Procainamide)
○ Side effect: Systemic Lupus Erythematosus (butterfly
● Is the splitting up of a molecule with water, the -OH group rash)
of water becoming part of one molecule and the hydrogen ○ Asians are fast acetylators, namemetabolize agad
atom becoming part of the other. yung Isoniazid hindi siya magtetake effect, at ang
● After hydrolysis, carboxylic acid and alcohol forms effect non, hindi pwedeng maging single ang
○ Acid (urine) treatment sa tuberculosis, hindi pwedeng Isoniazid
○ Yung alcohol na iniinom ng tao, after some time lang, kailangan in combination para maiwasan din na
nagiging aldehyde. Kapag naging aldehyde na yon, magkaroon ng resistance. Kaya combination drug o
doon na nakakaramdam na medyo tipsy. Tapos after triple therapy sa Pilipinas kapag TB.
non, yung nararamdaman na kasunod ay naiihi na ● Conjugation with Glutathione (GSH) - S-transferase
kasi na-convert na siya ng metabolism into acid. enzyme (liver, kidney)
● Type of Reactions ○ Detox and cell protective activity
○ Ester Hydrolysis ○ Binds mostly in drugs to detoxify
■ Cocaine - addictive ● Glycine conjugation - found abundant among animals. In
■ Aspirin humans, it metabolizes salicylic acid and nicotinic acid
■ Enalapril ● Methylation - to deactivate, attenuate or terminate
○ Amide Hydrolysis pharmacological acitivty. It takes effect in epinephrine,
■ Lidocaine - local anaesthetic dopamine, and histamine or in drugs containing thiol,
■ Procainamide catechol, phenol, and amines.
■ Indomethacin
Glucuronidation
Phase II or Conjugation Reactions
● In neonates and children, glucuronidating processes are
1. Convert the Phase I metabolites to more polar and often not developed fully, and accumulation of drugs can
water-soluble products by the addition or attachment of lead to toxicity.
small, polar, and ionizable endogenous molecules, such ● Neonatal Hyperbilirubinemia
as: ○ Inability of newborns to conjugate bilirubin with
● Sulfate glucuronic acid
● Glycine ● Gray Baby Syndrome
● Glucuronic acid ○ Inability to metabolize chloramphenicol
● Glutamine
Conjugated products What are the processes involved in the metabolism of
Aspirin and Paracetamol?
○ Relatively water soluble and readily excretable
○ Biologically inactive and nontoxic ● Acetaminophen/Paracetamol
○ Other phase II reactions, such as methylation and ○ In adults:
acetylation do not generally increase water ■ Major metabolite: O-glucuronide conjugate
solubility but mainly to serve to terminate or ■ O-sulfate conjugate formed in small amounts
attenuate pharmacological activity. ○ In infants and young children (ages 3 to 9 years)
■ O-sulfate conjugate is the main urinary product
2. Serve to terminate or attenuate pharmacological activity
3. Detoxification Metabolism of Aspirin Metabolism of Paracetamol
● Can be regarded as detoxifying pathways
● The role of GSH (glutathione conjugation) is to - The main metabolite is a - Paracetamol undergoes
glycine conjugate glucuronidation and
combine with chemically reactive compounds to
sulfation
prevent damage to important biomacromolecules such
as DNA, RNA, and proteins. - With increasing dose the - With increasing dose both
● Based on some journals, may additional na vitamin C glycine conjugation these systems become
sa GSH. Yung vitamin C yung nakakboost ng katawan system become saturated saturated
kasi yung liver nagpoproduce ng sariling glutathione. - Glucuronide conjugation - Then the drug is
Yung mga supplements, konti lang yung laman na then becomes important conjugated with
glutathione. Kaya nakakaputi ang glutathione kasi - With even higher doses glutathione
the glucuronide system
nadedetoxify yung mga cells, nareremove yung mga
becomes saturated
toxic compounds.

Phase II Metabolism - A greater proportion of the - If this pathway becomes


drug appears in the urine saturated a hepatotoxic
● Glucuridination as salicylic acid metabolite accumulates
○ Acetaminophen - The rate of elimination (hence the danger of

9
PCOL 211 - Pharmacology 1

then depends on urinary overdose) Oxidation CYP2C19 Mephenytoin Overdose toxicity


pH (antiepileptic)

● If you add aspirin with alkaline, it faster to eliminate S-Methyla- Thiopurine Mercaptopurines Myelotoxicity
because they have different pH. Same pH = reabsorbed. tion methyl-tran (cancer
● Toxic dose of paracetamol: 8-10 tablets of 500 mg sferase chemotherapeutic)

Factors A ecting Drug Metabolism Oxidation CYP2A6 Nicotine Lesser toxicity


(stimulant)
1. Chemical Structure of Drug - depends if lipid-soluble,
water-soluble Oxidation CYP2D6 Nortriptyline Toxicity
2. Genetic Differences - Caucasians, Asians (antidepressant)
3. Diet and Environmental Factors - eating fatty, salty, and
O-Demeth CYP2C19 Omeprazole (PPI) Increased
sweet foods can lead to the deterioration of organs can -ylation therapeutic
and cause damage to the metabolism. efficacy
4. Physiologic Stress or Disease - long-term effect of
stress: slows metabolism and damages organs (obese) Oxidation CYP2D6 Sparteine Oxytocic
symptoms
5. Age
6. Gender - males have a faster metabolism than females. Ester Plasma Succinylcholine Prolonged apnea
The physiologic function of females is bearing a baby. Mas hydrolysis cholinester (neuromuscular
marami ang body fat dahil mas mabagal ang metabolism. -ase blocker)
Hinihintay ng katawan mo na magbuntis ka at kailangan
Oxidation CYP2C9 S-warfarin Bleeding
may naka reserve na energy for the fetus. (anticoagulant)
7. Drug Interactions - inducer or inhibitor
Oxidation CYP2C9 Tolbutamide Cardiotoxicity
Genetic Factors (hypoglycemic)

Isoniazid Metabolism - Acetylation Diet

Plasma Effect ● Charcoal boiled foods and cruciferous vegetables.


half-life Because benzopyrene (smoked meats) can cause cancer.
○ Induce CYP1A enzymes
Slow Egyptians 140 to Peripheral neuritis ● Grapefruit juice
acetylators Western 200 and drug-induced ○ Inhibits CYP3A
Europeans minutes systemic lupus ○ Decreases metabolism of coadministered drug
erythematosus
syndrome Enzyme Induction

Fast Eskimos 45 to 80 Develop ● Cigarette smoking


acetylators Asians minutes isoniazid-associated ● Workers exposed to some pesticides (induces metabolism)
hepatitis
(acetylhydrazine) Diseases A ecting Drug Metabolism
● Heart problem: Low cardiac output = less oxygen supply
Genetic Polymorphisms in Drug Metabolism ○ Oxygen is used in metabolism
● Liver problem
Defect Enzyme Drug and Clinical
Involved Therapeutic Use Consequences EXCRETION
● The removal of drugs from the body
Oxidation CYP2D6 Bufuralol Exacerbation of
(β-adrenoceptor β-blockade, ● A drug or metabolite must be polar or water-soluble in
blocker) nausea, order for them to be excreted either in lungs, milk, sweat,
hypotensive tears, skin, hair, saliva, liver or urine.

Oxidation CYP2D6 Codeine Reduced Half-Life


(analgesic) analgesia
● The time it takes for one-half of the original amount of a
Oxidation CYP2D6 Debrisoquine Orthostatic drug in the body to be removed.
(antihypertensive) hypotension ● A measure of the rate at which drugs are removed from
the body.
Oxidation Aldehyde Ethanol Facial flushing,
dehydroge- (recreational drug) hypotension,
nase tachycardia,
nausea, vomiting

N-Acetylati N-acetyl Hydralazine Lupus


-on transferase (antihypertensive) erythematosus-lik
e syndrome

N-Acetylati N-acetyl Isoniazid Peripheral


-on transferase (antitubercular) neuropathy

10
PCOL 211 - Pharmacology 1

● E.g., Mannitol - a diuretic drug. It produces an extensive


REVIEW OF PHARMACODYNAMICS amount of diuresis because it will promote the osmotic
pressure effect.
Part 1: MECHANISMS OF DRUG ACTION ○ It will attract water to your tubules, then the kidney will
excrete an extensive amount of urine.
○ MOA: To stimulate the osmotic effect on our kidneys to
PHARMACODYNAMICS decrease blood pressure.
● A branch of pharmacology that focuses on the study of the ■ Also decreases the pressure in the brain. Kapag
biochemical and physiological effects of drugs and the masyadong maraming fluid accumulation sa brain
mechanisms by which they produce such effects.
2. Chemical Reaction
○ Mechanism: How do they take effect on our normal
function, physiological function. ● Two drugs are added together, to neutralize their effect, to
■ How do they increase the temperature, decrease prevent the toxic effect, and to enhance their excretion
the blood pressure
2.1 Neutralization
● What the drug does to the body at paano niya ginagawa
iyon ● About on having an equal action or they will antagonize
each other reaction
Mechanism of Drug Action ● Can be local or systemic
A. Non-target Protein-mediated mechanism ● Example:
B. Target Protein-mediated mechanism ○ Acid drug + basic environment = excreted
● DRUG + Receptor = Response (mawawalan ng effect, ma-neneutralized)
○ Drug will bind to a receptor or regulatory ■ If you take amphetamine, a basic drug, and you
molecule, and produce an action or response to will acidify the urine, it will enhance the excretion
target protein-mediated of amphetamine
○ Mechanism: Drugs act either by receptor or by a ■ Yung ibang addict na magpa laboratory umiinom
non-receptor, non-target protein, or by targeting ng orange juice para bumilis ma excrete yung
specific genetic changes in our body. amphetamine para kapag nag pa laboratory na
● Ang drug pwedeng directly mag-take effect sa isang sila negatve na yung result
part ng katawan or organ, or si drug magbi-bind muna ■ It is not advisable to stimulate diuresis of basic
siya sa receptor then saka siya magpo-produce ng drugs because it might destroy the kidneys.
response or action. ○ Acid drug + acid environment = absorbed
● Drugs produced their effect to interacting with some (non-ionized)
chemical compartments of living organism receptors. ○ Basic drug + acid environment = excreted
and receptors are macromolecules that so receptors ○ Basic drug + basic environment = absorbed
or regulatory molecules are macromolecules most are
2.2 Chelation
proteins present either in the cell surface of our body
or in the cytoplasm of a cell for in the nucleus of our ● The formation of a large complex molecule which called as
set another organism or regulatory molecules or chelates
macromolecules like most proteins are present in ● If the substance is large in particle size, it will not be
either cell surface cytoplasm rna nucleus where drugs absorbed in the body and will be excreted in the body.
can bind to them and produce an action. ● Digoxin + binding resins like cholestyramine
○ Cholestyramine can form a chelate with digoxin
● Receptor-Mediated - drugs produce their effect (digoxin will not have a pharmacological effect;
through interacting some chemical compartments of
magfo-form ng chelate, magiging large molecule at
the living organism receptors.
● Receptors - or regulatory molecules, are ma-eexcrete)
macromolecules, most are proteins, present either in Others
the cell surface of the body, in the cytoplasm, or
nucleus of the cell, where drugs can bind to them and ● CN + Sodium thiosulfate
produce an action. ○ Cyanide is a toxic substance; to prevent toxicity, give
sodium thiosulfate to enhance excretion of cyanide.
A. Non-target protein-mediated mechanism That is a chemical reaction.
○ We will add two drugs together to neutralize their
● Exert effects without binding to proteins
effect to prevent their toxic effect or to enhance their
● They may interact with other drugs or they can directly act
excretion.
in our organs
3. Counterfeit Incorporation Mechanism
1. Colligative Mechanism
● Counterfeit - fake
● Osmotic effect or Mass effect ● Carmustine will cause alkylation with insert incorrect DNA
○ Osmotic pressure - from low concentration gradient that will cause inhibition of DNA replication, which leads to
to high concentration gradient. cell death, apoptosis or cell lysis.
● Colligative properties depend only on the number of ○ Some drugs will insert incorrect message to the dna
dissolved particles in a solution. para mawalan ng kopya hindi na makaka paggawa ng
○ “Mas maraming concentration dito, dito tayo pumunta”

11
PCOL 211 - Pharmacology 1

protein yung isang cell or isang bacteria or virus na ● Hormones - enhance the effects of other substances and
mag-cacause ng cell death help our body to grow. Includes: Channel transport, carrier
molecules, enzymes, and receptor

2.1 Channels/Transport
● Proteins that take part in transmembrane signaling and
regulate ionic composition.
● It includes sodium channels and calcium channels. Voltage
gated because they cause depolarization, they cause
action potential response through ionic charges.

Voltage-gated Na Channel
● Na channel blockers:
○ Local anesthetics, CBZ, Phenytoin
B. Target protein-mediated mechanism
● Permits the Na influx (pag pasok ng Na papunta sa
● Target protein - biologic site of action of drugs (also known intracelular) if there is an excess Na ion it can cause
as regulatory molecules, receptors) toxicity, shaking, seizure, over contraction of muscle.
a. Structural Proteins ● Treatment: Na channel blockers like: Carbamazipine,
b. Regulatory Proteins Phenytoin , Antiepileptic, Anticonvulsant, and Anti Anxiety
1. Structural Proteins drugs.
● MOA: Inhibit or block the Na channel to prevent influx of
● Constitute “cytoskeleton” Na to inhibit depolarization and action potential.
● Microtubule - Important site of action ● Lidocaine - local anesthetics
○ Important part of the cytoskeleton in the mitotic ○ Bakit walang nararamdaman kapag binibigyan ng
spindle, and helpful in cell division local anes? Because walang pumapasok na Na ion,
● Drugs that inhibit microtubule synthesis/spindle protein walang transmembrane signaling (walang nagsasabi
○ Griseofulvin - Best added with fatty meals to increase sayo na masakit iyon, or hinihiwa ka)
its absorption ● Lidocaine + epinephrine (vasoconstrictor) - to enhance
■ MOA: Not fully defined but it can inhibit the localization of the anesthetic effect.
microtubule synthesis and key to prevent cell
division of the fungi. Voltage-gated Ca Channel
○ Vinca alkaloids - includes vincristine,vinblastine from ● Ca++ channel blockers:
chichirica ○ L-type blockers (“-dipine”)
■ They will prevent tubulin polymerization into ○ T-type blocker - ethosuximide
microtubules and it will lead to mitotic arrest in ● It is an ion channel which shows selectivity permeability to
metaphase which stops the cell division that eats Ca ions only.
to cell death or apoptosis. Used for cancer so this ○ Ca ion - Take effect more on cardiac muscles
is used as an anticancer drug. ● If Ca goes inside the cell or goes in the intracellular
○ Colchicine - Colchicum autumnale membrane, it will cause action potential by increasing
cardiac conductivity and increasing the movement of
■ For acute gout
muscles. That's why kailangan i-inhibit yung Ca because it
■ MOA: It binds to microtubules preventing can cause tachycardia
polymerization into microtubule in leading to the ● Treatment: Ca channel blockers like:
inhibition of leukocyte migration and phagocytosis ○ L- type Blocker (dipine)
process ■ MOA: block the L- type Ca channel blocker to
■ Kapag walang cell division it will not attract prevent influx of Ca to decrease cardiac
leukocytes and phagocytes.yung mga white blood contraction to prevent hypertension and
tachycardia
cells and phagocytic cells they produce ○ T- type calcium channel Blocker (ethosuximide)
inflammation ■ MOA: Block T- type Ca channel to prevent too
○ Etoposide - from Podophyllum peltatum (American much muscle contraction (Muscle relaxing drugs)
Mayapple)
● PISO - Potassium in, Sodium Out
■ Inhibits microtubule synthesis ● MISO - Magnesium in, Sodium out
2. Regulatory Proteins ● Sodium and calcium should be found in the
extracellular part of membrane. Dapat mas madami
● Mediates the transmission of endogenous chemical sila sa extracellular.
signals such as neurotransmitters, autacoids, and ● Potassium and magnesium should be more sa inside
hormones. part dahil mas less yung effect nila
● Neurotransmitters - substances in our body that send ● There should be an exchange between the sodium and
potassium and magnesium and sodium chloride para
signals into our brain to the pheriperies going to the CNS. balance.
includes norepinephrine, epinephrine, dopamine, ● Sodium ions and Calcium ions helps you to move and
serotonin, histamine think because they send electrical signals electrical
● Autacoids - inflammatory molecules that cause allergic impulses movement transmembrane signaling which
cause of reaction we call Depolarization.
● Depolarization - is a change within a cell during which

12
PCOL 211 - Pharmacology 1

prevent absorption of sodium and reabsorption of


the cell undergoes a shift with electric charge water
distribution, resulting in less negative charge inside the ○ Found in thick ascending limb of Henle
cell but more and positive. If sodium influx happens or
when sodium goes inside the cell or goes into the ○ Helps in the Na absorption, and the tubuloglomerular
intracellular membrane it will change the electric feedback in the kidney
charge inside the cell which causes depolarization ■ Nephrons - basic functional unit of kidneys
which causes action potential. ○ Because it helps in the sodium absorption, it can
○ Depolarization is essential to the function of many cause reabsorption of water, which can lead to
cells, communication between cells, and overall hypertension
physiology of an organism (kaya ka nakakagalaw,
■ Kasi kapag na-reabsorb ang sodium, tubig,
nakakayo, and kung bakit tumitibok ang puso
natin ay dahil sa mga electrical charge) mag-iincrease yung blood volume that would lead
to hypertension.
2.2 Carrier Molecules ■ What we need to give is a diuretic in the kidneys

2.3 Enzymes
Na+ -K+ ATPase pump
● are protein catalysts
● Sodium potassium ATPase enzyme is a solute pump that ● Helps to breakdown anything that you eat into smaller
pumps sodium out of this cell while pumping potassium particles, helps to metabolize
into the cell so both against their concentration gradient.
(NA OUT, K IN) Xanthine Oxidase
● For every ATP molecule the pump uses 3 Na ions that are ● Can catalyze or fasten up metabolism of certain
exported and 2 K ions are imported in one ATP, there's a substances like purines from proteins and dietary products
export of a single positive charge per pump cycle which and food, like legumes, coffee or tea that we take
will lead to resting potential. ○ Xanthine oxidase will convert protein to uric acid
● Resting potential in order to maintain the cell membrane ■ Uric acid - cause of gout
potential cell keep a low concentration of Na ions and high ● INHIBITOR: Allopurinol
concentration of K ions ○ MOA: Inhibits xanthine oxidase to prevent uric cid
○ Kapag mas madaming K sa loob ng cell, mas formation
mabagay yung action potential or depolarizing effect ○ Allopurinol - for chronic gout
ng K. ■ Colchicine is for acute gout
● INHIBITOR: DIGOXIN
○ Inhibits sodium potassium ATPase pump so it will
maintain sodium and if there's increase and sodium
ions inside the cell it will activate the sodium-calcium
exchanger increasing now the calcium levels inside
the cell which causes calcium binding that stimulates
cardiac contractility or increasing cardiac contractility
again.
○ MOA: Increasing the cardiac muscles leads to
intracellular accumulation of calcium which increases
contraction of the heart by inhibiting sodium potassium
ATP
■ Pipigilan nya lumabas si Na, para kapag
maraming Na sa loob ng cell, ma-activate ang
sodium calcium exchanger,papasok ang Ca,
dadami ang calcium resulting to increase
contraction of heart
H+ -K+ ATPase pump
● Xanthine oxidase - converts hypoxanthine to xanthine,
● In the gastric part of the body HKATPase which functions to uric acid
to acidify the stomach. It aids in the transport of hydrogen, ○ Nakukuha ang hypoxanthine sa mga kinakain or
to inc the HCl ini-intake na gamot
● High HCL → ulcer, hyper acidity, GERD, heartburn ■ meat products that contains a lot of proteins
● INHIBITOR: PPI or purines,
○ Omeprazole - Inhibit HKATPase pump to dec the ■ 1,3,7-trimethylxanthine (caffeine) coffee
■ 1,3-dimethylxanthine (theophylline) tea
acidity of the stomach
■ 3,7-dimethylxanthine (theobromine) chocolate
Na+ -K+ -2Cl- cotransport ● Because uric acid is a byproduct, it can accumulate
inside our body that can cause gout.
● Found in the cells of the thick ascending limb of the loop of
henle (inside the kidney). They help to balance, excrete, or
Cyclooxygenase
absorb to maintain homeostasis.
● Too much Na+ -K+ -2Cl reabsorbed → hypertension ● Part of cyclooxygenase pathway enzyme that can cause
● INHIBITOR: FUROSEMIDE prostaglandin synthesis, which can lead to inflammation
○ Furosemide (diuretic) is a loop diuretic which is used ● INHIBITOR: NSAIDs
as antihypertensive. ○ MOA: Inhibits cyclooxygenase enzyme to prevent
○ MOA: It will inhibit Na+ -K+ -2Cl- cotransport to

13
PCOL 211 - Pharmacology 1

prostaglandin synthesis (to prevent pain, fever or


vasoconstriction, sodium and water reabsorption, due
inflammation) to aldosterone stimulation that causes high blood
pressure

● MAO (Monoamine oxidases)


○ Metabolizes certain neurotransmitter like serotonin,
dopamine norepinephrine, epinephrine.
○ Inhibitors: MAOI (used as antidepressants)
■ MOA: Inhibits monoamine oxidase enzyme to
prevent metabolism of neurotransmitter like
serotonin and dopamine
■ Phenelzine
■ Isocarboxazid
■ Tranylcypromine

● Cyclooxygenase (COX) converts arachidonic acid to


certain substances like prostaglandin (PG) and
thromboxane (TX)
● Two pathways: COX-1 and COX-2
● COX-1 synthesizes PGI2
● COX-2 synthesizes PGE2
● PGI2 enhancing gastric protection by increasing
mucus production
○ Mucus production in stomach protects the wall of
stomach from acid
● TXA2 stimulates platelet function which is clot
formation ● MAO-A can metabolize serotonin, dopamine,
● PGE2 synthesis causes pain, bone formation and fever norepinephine, epinephrine, and tyramine
● In some cases, non-selective sila. Parehas nilang ● MAO-B can metabolize dopamine, norepinephrine,
epinephrine, and tyramine, but more on dopamine
ini-inhibit si COX-1 at COX-2. Kapag ganon ang
● Kapag na-metabolize and serotonin, which is our happy
pagkakataon, like aspirin, mag-cacause siya ng hormone, nalulungkot yung pasyente, leads to
bleeding, because it will inhibit synthesis of platelet, depression, kaya binibigyan ng mga antidepressants na
and can also cause gastric disturbance kaya nakaka mga MAO inhibitors
sikmura kasi it also inhibits PGI2 (and it will decrease
mucus production) ● ACETYLCHOLINESTERASE
○ Kaya nakaka sikmura yung ibang NSAIDs. Kaya ○ Metabolizes acetylcholine (the neurotransmitter in
pinapa-take and ibuprofen 1 hour after kumain, or parasympathetic nervous system)
after mag-take ng ibuprofen, kakain ka after 30 ○ Inhibitor: Edrophonium
mins ■ MOA: Inhibits acetylcholinesterase enzyme, thus,
● Kapag selective naman, like COX-2, they only inhibit making it your myasthenia gravis kinda good
prostaglandin synthesis ● Dadami yung acetylcholine, magkakaroon na
ng enough muscle contraction, kaya
ma-ttreat niya or magiging mabuti,
ACE ma-eenhance yung condition kapag may
● Angiotensin-converting enzyme myasthenia gravis ka
● Is a part of Renin-angiotensin-aldosterone (RAA) system ■ Edrophonium is part of Tensilon’s test which
● Converts Angiotensin I to Angiotensin II which causes determines if you have myasthenia gravis
hypertension (weakness in skeletal muscle)
● INHIBITORS: ACE inhibitors
○ MOA: Inhibits ACE to prevent conversion of
angiotensin I to angiotensin II, useful for hypertension
○ Captopril, Enalapril
○ Side effects: It can cause dry cough because ACE
inhibitors can also inhibits bradykinin metabolism
■ Bradykinin - inflammatory substances that can
cause irritation
■ Ayaw ng mga pasyente na uubo ubo sila, kaya
nag-ARBS na lang sila (Angiotensin Receptor
Blockers)
● Angiotensinogen from the liver is converted to
Angiotensin I, by the renin coming from the kidney
● Angiotensin I is converted to Angiotensin II by ACE
● The bad effect of Angiotensin II is that it causes

14
PCOL 211 - Pharmacology 1

○ Affinity - ability of the drug to get bound to the


receptor
○ Intrinsic activity or efficacy - ability to produce a
hpharmacologic response after breaking a drug
receptor complex
● Receptor site or recognition site - a specific region
of a receptor molecule to which a drug binds
○ Parang yan yung pinakataas na part, parang
doorknob na lalagyan ng susi
● Effector - component of a system that accomplishes
the biologic effect after being activated by an agonist;
often a channel or an enzyme molecule.
○ For example, adenylyl cyclase
● Spare receptor - Receptor that does not bind drug
when the drug concentration is sufficient to produce
maximal effect
○ Kapag okay naman yung mga receptor na
● Acetylcholine will be metabolized into choline and available at nag-cacause sila ng effect, hindi na
acetate by acetylcholinesterase kalangan ng spare receptor na ma-combine pa sa
○ It can lead to muscle weakness, kasi less na yung ligand or sa ibang substance
contraction ng muscle ● Drug + Receptor = Drug-Receptor complex that will
○ Can lead to tachycardia produce a response
○ Kumonti yung acetylcholine para sympathetic ang
action TYPES OF RECEPTORS
● COMT (Catechol-O-methyltransferase) 1. Type I Receptor
○ Metabolizes neurotransmitter like norepinephrine, 2. Type II Receptor
dopamine, epinephrine (catecholamines) 3. Type III Receptor
■ Can lead to some diseases like Parkinson’s 4. Type IV Receptor
disease. Kasi kapag kulang yung mga
1. TYPE I RECEPTOR
neurotransmitter mo, mawawalan ka na ng
voluntary control sa mga muscles mo, and 1.1 Ionotropic Receptors
kokonti yung intellectual capability mo, kaya
kusang gagalaw na yung ibang part, nanginginig ● A receptor protein that forms a part of a ligand-gated ion
nginig na lang channel, so that binding of ligand to the receptor causes
○ Inhibitors: Entacapone, Tolcapone the opening of channel, permitting ions to flow through it.
■ MOA: Inhibits Catechol-O-methyltransferase, Mga ions ang papapasukin niya kapag nakapag bind na
preventing the metabolism of catecholamines like yung isang ligand
dopamine, useful for Parkinson’s disease ○ Controls movement of ions
○ Found in cell membranes
2.4 Receptors ○ Stimulated in milliseconds
● or regulatory molecules ■ Kapag naisip mong gusto mong igalaw yung
● Functional macromolecular components of cells with kamay mo, gagalaw yan. Kapag gusto mong ipikit
specific stereochemical configuration and in which a ligand yung mata mo, mapapagalaw mo yan
interacts
● Characteristics of receptor:
○ SPECIFICITY
○ SELECTIVITY
○ AFFINITY
● Ligand - any chemical substance that has the ability to
bind to a receptor
○ Either neurotransmitter or a drug
● Drug-Receptor interaction - selectivity, affinity, intrinsic
activity, or efficacy
○ Selectivity - degree of complementary correlation
between drug and receptor
■ Example: Adrenaline selectivity for alpha-beta ● In ionic receptor, when a ligand bind to a receptor
(green), it will open the channel and will permit the ion
receptor. Si epinephrine, mag-bibind lang siya kay
passage (sodium, calcium chloride)
alpha-beta receptor kasi nasa sympathetic
nervous system sila, sila yung nandon talaga.
1.2 GABA Receptors
Selective, kung sino lang yung kapamilya nila.
● Alpha-beta receptor - receptor of sympathetic ● are inhibitory type of receptors
nervous system ● Gamma-aminobutyric acid (GABA)
● Adrenaline (epinephrine) - neurotransmitter ● GABA receptors control Cl- ions stimulated by:
in sympathetic

15
PCOL 211 - Pharmacology 1

■ Benzodiazepines - will bind to GABA receptor, it will


Examples of G-proteins
open the channel and will permit the influx of chloride
ion, which are negatively charged. Dahil negatively ● They can activate or inhibit secondary messenger
charged, magkakaroon ng inhibitory effect, ma-rerelax ○ Gs - stimulates adenylyl cyclase (activating)
yung ating cell. ○ Gi - inhibits adenylyl cyclase (inhibits)
■ Barbiturates - and benzodiazepines, they are ○ Gq - increases IP3, DAG
anticonvulsant, antiseizure, antiepileptic, antianxiety ■ Inositol trisphosphate (IP3)
(pampakalma, calming effect, humihina at nagiging ■ Diacylglycerols (DAG)
malumanay)
● Gs (stimulatory G-protein) activates β-Adrenergic,
1.3 Nicotinic Receptors histamine, serotonin and glucagon, stimulates adenylyl
cyclase, increasing cAMP.
● Stimulating effect ○ It opens adenylyl cyclase pathway, ATP is used,
● Control the entry of Na+ (a cation) and it activates cAMP. cAMP will be converted to
○ Effects are polarization, action potential, contraction protein kinase, leading to phosphorylation, which
● For example, bakit naninigarilyo yung mga nasa call is a function of protein.
○ Common na gumagana ang Gs protein kapag
center? Kasi nagigising daw sila, and pampabilis daw ng
merong glycogenolysis, lipolysis, calcium channel
pagbaba ng kinain, pampabilis ng metabolism. Cigarette activation
smoking induces your enzymes. ● Gi is related to alpha-2 adrenenergic receptor,
○ Nicotine will bind to a nicotinic receptors, will open the acetylcholine receptor
channel and will permit the sodium ions. Kapag may ○ Inhibits/decreases adenylyl cyclase, and
sodium ion sa intracellular, it causes the polarization decreases cAMP, which leads to opening of
action potential, magiging stimulated yung isang tao, potassium channel
■ Magkakaroon ng relaxation
magigising siya and kapag sumobra, too much muscle
● Gq - acetylcholine receptor, phospholipase C
constraction, yung iba nag-seseizure, yung iba ○ Increases IP3, leads to the production of calcium,
nanginginig- nginig kasi may anxiety na, sumobra na leads to activation
kasi yung ions sa loob ○ It has an effect on smooth muscle contraction
● Inhibited by neuromuscular blocker because of production of calcium, and glandular
○ MOA: Inhibits nicotinic receptor to prevent entry of secretion
sodium and to prevent polarization, to inhibit action ○ When Gq activated, DAG opens protein kinase C,
which can also lead to phosphorylation of enzyme
potential

2. TYPE II RECEPTOR ● Second messengers


○ Cyclic adenosine monophosphate (cAMP)
● G-protein lined receptor / metabotropic receptor ○ Cyclic guanosine monophosphate (cGMP)
● A 7-transmembrane spanning receptor ○ Inositol triphosphate (IP3)
○ The polypeptide chain reverses the membrane seven
times Examples of Type II Receptors
● GTP-binding signal transducer protein is G-Protein ● Where Gs can be found or activated)
● G-protein modulates production of an intracellular second 1. Adrenergic receptors (Adrenoceptors)
messenger ● Found in sympathetic nervous system
○ Location: Cell membrane ● Alpha receptors (I, II, III)
○ Onset: In seconds ● Beta receptors (I, II, III)
● When a chemical (like hormone or pharmaceutical agent), 2. Muscarinic receptors (I, II, III, IV, V)
binds to a receptor (like metabotropic receptor) on the ● Found in parasympathetic nervous system
outside of the cell, this triggers a series of chemical ● Acetylcholine will bind to muscarinic receptor,
reaction including the movement and binding of G-protein activating Gi protein, will decrease adenylyl cyclase,
transformation of GTP into GDP, and activation of second will decrease cAMP, leading to relaxation
messengers (like the cAMP). Secondary messengers stars 3. Phosphodiesterase receptors
a cascade of enzymatic reactions leading in cellular ● Drugs: MOA: They will inhibit phosphodiesterase type
response. This signaling method is quite fast and more III receptor, used for heart failure by increasing cardiac
importantly, it amplifies the signal. contractility and lowering peripheral vascular
○ Guanosine triphosphate (GTP) resistance
○ Guanosine diphosphate (GDP) ○ Bipyridine
○ Cyclic adenosine monophosphate (cAMP) ○ Inamnirone (Old name: Amrinone)
G-protein coupled receptors ○ Milrinone
● G-protein composed of one alpha, beta, and gamma
● These are found in the brain, and autonomic nervous
subunit
system
● 2 primary signaling cascades: cAMP or
phosphatidylinositol pathways
● Pathway activated depends on alpha subunit type 3. TYPE III RECEPTOR
○ (Gαs, Gαi/o, Gαq/ I I, Gα12/13)
● Tyrosine kinases-lined receptors
○ GDP bound to α when inactive
● These are highly-affinity cell surface receptors for many
polypeptide growth factors, cytokines (inflammatory agent),
and hormones.

16
PCOL 211 - Pharmacology 1

○ Location: nucleus ● Drugs that act on Type IV receptors


○ Onset: hours ○ Corticosteroids
● Tyrosine kinase - catalyze phosphorylation of tyrosine ○ Mineralocorticoids
residues to modulate a number of biochemical processes. ○ Sex steroids
○ Kinase is an enzyme that transfers phosphate group ○ Vitamin D
from high-energy donor molecules such as ATP, to ○ Thyroid hormone
specific target molecule substrate in a process known ○ Retinoids
as phosphorylation
Kapag nag-iisteroid yung mga tao, lumalaki yung katawan
○ Enzymatic receptors are directly link to tyrosine kinase nila kasi nakaka-aid din sila sa protein synthesis
○ Receptor binding domain present in extracellular site
○ Produce conformational changes in intracellular (e.g.,
Receptor Interactions
insulin receptor)
● Examples: 1. Lock and Key Theory
○ Imatinib - gastrointestinal stromal tumors (GIST) ● The drug molecule must “fit into a receptor” like a “key
■ It will block the type III receptor inhibiting fits into a lock”.
phosphorylation (kasi para siyang continuous ● Agonist and Receptor has the same shape
building or binding of substances para makabuo
ng mas larger cell or substances)
● Because of phosphorylation, lumalaki nang
lumalaki ang cell hanggang sa maging tumor
● It will stop continuous growing of the cell
○ Gefitinib - epidermal growth factor receptor
■ It will stop phosphorylation useful for epidermal
growth
○ Erythropoietin receptor
■ Receptor for RBC production
○ Receptor for Insulin - utilization of glucose 2. Induced-Fit Theory
● As the drug approaches the receptor, it alters the
Glucose ===> Glucokinase ===> glucose-6-phosphate conformation of its binding site to produce a
- Will be used to make energy for building of cell drug-receptor complex. (Ipinilit)
- Kaya si insulin, mag-bibind ng glucose, ma-aactivate
yung kinase (glucokinase)
- Glucose-6-phosphate can be stored an used as
reserved energy

Theories of Drug-Receptor Reaction


● Occupancy Theory - drug effect is directly proportional to
the number of receptors occupied.
○ Kung merong tatlong receptor para magkaroon ng
effect, lahat ng tatlo na yon kailangan ma-bind-an or
ma-occupy
● Si kinase i-hehelp niya na madagdagan ang phosphate ● Ariens and Stephenson Theory - occupational theory of
group yung isang substance para merong energy, response.
which will be use for phosphorylation later on, or yung ● Rate Theory - the activation of receptors is directly
pag build up ng isang thing proportional to the total number of encounters of the drug
with its receptors per unit of time.
4. TYPE IV RECEPTOR ○ Kung gaano kadalas na-eexpose yung isang receptor
● Gene transcription-linked receptors sa isang gamot
● Location: nucleus / cytosol (cytoplasm) ■ Kaya umiinom 3x a day, kase after mo uminom,
● Onset: hours magkakaroon ng response. Unti-unting nawawala
● Acts on the central dogma yung response kaya kailangan ulit-ulitin yung pag
○ Applicable to humans and microorganisms (sa inom
bacteria or virus, malalaman kung paano mapipigilan
dumami)
■ Replication (DNA copied into complimentary
DNA)
■ Transcription (DNA template is copied to RNA)
■ Translation (RNA synthesize protein)

17
PCOL 211 - Pharmacology 1

Types of Receptor Action


Part 2: DOSE-RESPONSE RELATIONSHIP
Upregulation / Sensitization
● The prolonged continuous use of receptor blockers DOSE-RESPONSE RELATIONSHIP
● Inhibition of synthesis or release of hormone
● a.k.a “Exposure-Response Relationship”
neurotransmitters will lead to denervation (loss of nerve
● Describes the change in effect on organism caused by
supply) because we are continuously exposed to a blocker
differing levels of exposure (dose) to a stressor (chemical)
● Hyperactivity and supersensitivity occurs when target
after a certain exposure time.
cells are subject to long term exposure to receptor
● Naexpose ka kaya nagkaroon ng response yung katawan
antagonist or to receptor blockers
mo, uminom ka ng gamot kaya nagkaroon ng effect sa
● Meron ka pa ring sensitivity or meron pa ring effect sayo
katawan mo
yung gamot which leads to hyperactivity and
a. Graded Dose-Response Curve
supersensitivity
b. Quantal Dose-Response Curve
● Kapag masyadong madalas kang gumamit ng mga
blockers, nato-tolerate naman. “Lagi mong pinipigilan, a. Graded Dose-Response Relations
kakayanin kong magkaroon ng action”
● It is a graph of response versus the logarithm of the dose
● Sa mga unang beses, may epekto. Then ma-aadapt ang
that yields the efficacy (Emax) and potency (ED50).
katawan. “Dahil lagi mo akong bino-block, magpaparami
● It is the magnitude or the intensity of the pharmacological
ako” until maging hyperactive at supersensitive na ang
response that increases as the dose administered
katawan ng tao
increases.
● Upregulation through synthesis of new receptors.
○ Habang tumataas yung iniinom mong dose, tumataas
Padadamihin nila yung sarili nila para sa susunod na
din yung epekto, which can lead to toxicity
i-block, wala nang epekto, mas naging strong sila
● It describes the relationship between the magnitude of the
Downregulation effect of a drug in an individual, and the doses of the drug
● It allows comparison for drug efficacies and potencies.
● Caused by continuous prolonged exposure of receptors to
drugs that disrupt the homeostatic equilibrium due to Efficacy - refers to the ability of the drug to elicit a

altered levels of receptor response at the molecular, cellular, tissue, or system
● Desensitization is the result of downregulatio, or the effect level.
of subsequent exposure of the receptors to the same ○ It is a measured by its maximum effect.
concentration of drug that leads to the reduction of ○ a.k.a. “Intrinsic Activity”
sensitivity ● Potency - refers to the concentration (EC50) or dose
● Nawawalan ka na ng sensitivity, wala ka nang pakiramdam (ED50) of a drug required to produce 50% of the drug’s
after matagal kang umiiinom ng gamot. maximal effect.
● Parang microbial resistance, dahil paulit-ulit mong iniinom ○ Is a measure of drug activity expressed in terms of
yung antibiotic, pagdating ng panahon wala na siyang the amount required to produce an effect of given
epekto sayo, nag-dodownregulate intensity.
● Dahil nga maraming receptor, tas lagi kang umiinom ng ○ Absolute amount of drug required to produce and
gamot, laging na-ooccupy yung receptor hanggang sa effect
kumonti na siya nang kumonti at mawalan ka na ng ○ Potent - in small amount it can elicit
response pharmacologic action
○ More potent drug is the one that requires lower
dose to cause the same effect
● Ceiling Dose - is the maximum amount of drug that can
induce biological effect by a given drug.

● Graded dose-response curves for four drugs, illustrating


different pharmacologic potencies and different maximal
efficacies.
○ Nag-cocompare ng dose at response. Gumaling ba
yung pasyente, anong dose siya gumaling, anong
dose nakita yung pagbabago sa kaniyang katawan
● This parameter reflects the limit of the dose response
relation on the response axis
● The stiffer the slope, the more readily the drug will bind to
a receptor

18
PCOL 211 - Pharmacology 1

○ Why don’t we use a drug with a therapeutic index of


less than 1?
■ It can be lethal.
○ TD50
ED50
● Median effective dose (ED50) - the dose at which
50% of individuals exhibit the specified quantal effect.
● Median toxic dose (TD50) - the dose required to
produce a particular toxic effect in 50% of animals.
● Median Lethal Dose (LD50) - the dose that can
produce death in 50% of the animals.

Variation in Drug Responsiveness


● Potent - Drug A and Drug B (Drug A is more potent)
● Idiosyncrasy (or idiopathic) - unusual drug response.
○ At small dose they produce an effect
● Drug A, C and D have equal maximal efficacy and greater Caused by genetic differences in metabolism of the drug or
than Drug B (higher than 50% response, almost all of them by immunologic mechanisms, including allergic reactions
have a 100% response) ○ From unknown cause
● Drug D may have important clinical consequences ● Hyporeactivity - the intensity of effect of a given dose of
(comatose) if the upper portion of the curve represents an drug is diminished.
undesirable extent of response (not that safe) ● Hyperreactivity - the intensity of effect of a given dose of
○ Has a extreme stiff slope, it can be toxic
○ It readily binds to a receptor. May action kaagad drug is increased in comparison to the effect seen in most
pagka saksak pa lang individuals.
● Stiff dose response curve in patients could result from ● Hypersensitivity - refers to allergic or other immunologic
cooperative interactions of several different actions of responses to drugs.
drugs ○ Overreacting
○ Effects in the brain, heart, peripheral vessels,
contributing to loewring of blood pressure Hypersensitivity Reactions (Allergic Reactions)
b. Quantal Dose-Response Curve TYPE OTHER NAMES EXAMPLES MEDIATORS
● All-or-none response
I Allergy ● Atopy IgE
● It is a graph of the number of patients that responds by a (Immediate) ● Anaphylaxis
specified dose ● Asthma
● Used to generate information regarding the margin of
safety to be expected from a particular drug used to II Cytotoxic, ● Autoimmune IgM or IgG
Antibody Hemolytic Anemia
produce a specified effect dependent ● Thrombocytopenia
● Determines the magnitude of drug effect and the number ● Erythroblastosis
of individuals within a population of response is quantified fetalis
as the dose is increased
III Immune Complex ● Serum sickness IgG
○ Depende sa number ng gumaling, anong dose sila
Disease ● Systemic Lupus
gumaling, sino yung pinaka maraming napagaling Erythematosus
(SLE)

IV Delayed-Type ● Contact Dermatitis T-cells


Hypersensitivity ● Mantoux Test
● Chronic Transplant
Rejection
● Multiple Sclerosis

V Autoimmune ● Grave’s Disease IgM or IgG


Disease ● Myasthenia Gravis

● Autoimmune disease - your own immune system attacks


your own cells and body
● Anaphylaxis or anaphylactic shock symptoms:
● Shaded boxes indicate the frequency distribution of ○ Dizziness, headache, rashes, inflammation, and that can
doses of drug required to produce a specified effect lead to suppressed breathing and asthma, continuous
coughing, excess mucus production
● The open boxes indicate the cumulative frequency
○ Kaya nakakamatay dahil kapag namaga pati yung
distribution of response, which are log normally smooth muscle sa throat area, magkakaroon ng
distributed blockage at hindi magpa-pass through yung hangin at
● Drug A has narrow therapeutic index hindi makakahinga. Difficullty in breathing hanggang
mamatay yung pasyente, seconds to minutes lang yun
● Therapeutic Index ○ DOC: Epinephrine - a vasoconstrictor
■ Kasi kapag meron tayong allergic reaction,
○ Is a comparison of the amount of a therapeutic agent
inflammed at dilated yung blood vessels kaya
that causes drug toxicity nababara yung hingahan
○ The dose of a drug required to produce a desired ● Cytotoxic - your antibodies will attack your own cell
effect to that which produces an undesired effect ● Hemolytic anemia - bursting of RBC
● IgM - pentamer (five binding site)
○ a.k.a. “Therapeutic Ratio”

19
PCOL 211 - Pharmacology 1

● IgG - largest
● Systemic Lupus Erythematosus (SLE) - butterfly rash Part 3: CONCEPTS OF AGONISM,
● Mantoux test - skin test to determine if you have tuberculosis
● Chronic transplant rejection - after operahan nung ANTAGONISM, ALLOSTERIC
pasyente (e.g., heart transplant). Kapag hindi binigyan ng
immunosuppressant, dahil hindi talaga part ng katawan mo
yung heart na galing sa ibang tao, aatakehin yon ng immune
MODULATION
system mo.
● T-cells - killer cells
● Grave’s Disease - hyperthyroidism, thyrotoxicosis (excess in CLASSIFICATION OF DRUGS ACCORDING TO
thyroid hormone) RECEPTOR INTERACTION
● Myasthenia Gravis - caused by too much
acetylcholinesterase (decreases acetylcholine, leads to ● Agonists - drugs that occupy receptors and activate them.
muscle weakness) (able to bind)
● Tolerance - it refers to a decreased responsiveness to the ○ With affinity - ability to bind to a receptor
drug, a consequence of continued drug administration ○ With intrinsic activity - full activation, full response
○ Dahil paulit ulit, na-totolerate mo na and you need to ■ Other name: Efficacy
increase the dose for you to be sensitive again to a ■ They have pharmacologic action that is direct
drug ● Antagonists - drugs that occupy receptors but do not
● Tachyphylaxis - rapidly acting tolerance. Responsiveness activate them. Antagonists block receptor activation by
diminishes rapidly after the administration of a drug. agonists. (Prevents binding of agonist, kontrabida))
○ Kapag sinaksak, sa susunod wala nang epekto sayo. ○ With affinity
Na-tolerate na agad ○ Without intrinsic activity (No activation)

CELL SURFACE RECEPTORS


Factors Influencing Drug E ects
1. WEIGHT - decreased adipose tissue (less weight) has
greater toxic effects, kasi less yung binding site
2. AGE
● Infants - underdeveloped liver enzyme system =
increase toxic effect (because they cannot easily
metabolize the drugs)
○ Chloramphenicol - Gray Baby Syndrome
(because they don’t have glucuronidation
metabolism)
● Geriatric patients - deteriorating body functions; has
less or diminished metabolism
3. GENDER - for IM injection
● Drugs, nutrition, ions, and many other more
● Male > female in terms of vascular muscles
substances, if they pass through the cell membrane
○ Effects faster in male than female
and they go to intracellular compartment, they will
○ Males have more proteins and muscles
produce a response
○ Females are more fatty
● Agonist - if they bind to a receptor and activates it,
4. PATHOLOGICAL FACTORS
they can pass through the channel, and prodcuce a
● Liver disease = decrease biotransformation of drugs =
cellular response.
increase toxic effects
● Antagonist - will block the receptor, and block the
5. GENETIC FACTORS
agonist from binding to the receptor and will inhibit
● Asians are fast acetylators
intrinsic activity caused by agonist
○ When we take isoniazid, increased acetylation =
increased metabolism = increased excretion =
can lead to diminished antitubercular effect Types of Agonist
○ Combination drugs for TB in PH: RIPE (Rifampin, 1. Full Agonist - causes full response, stimulate all different
Isoniazid, Pyrazinamide, Ethambutol) variant of receptor even the new receptor means
● Acetylation (metabolizing INH) = decrease ● Lahat babind-an nya to have a full response
antitubercular effects ● Disadvantage will be if it continuously bind to a
receptor, it can become saturated and later on baka
magkaroon na ng downregulation
2. Partial Agonist - less than the expected response. It
competitively inhibit the responses produce by full agonist.
● They will bind to a receptor, can produce intrinsic
activity
● Hindi lahat ng receptor ba bind-an nya pero
magkakaroon pa rin ng effect. Parang nakikipag
compete: “Ako kahit konti lang yung bind-an ko,
magkakroon pa rin ako ng effect”
● Partial agonist many drugs used clinically as
antagonist are weak partial antagonist.

20
PCOL 211 - Pharmacology 1

○ Maraming gamot ang antagonist pero ang ● Antagonist drug has high affinity with receptor they will
talagang action nila, they are weak partial agonist bind to the receptor and will block the agonist
○ At small dose, at small binding capacity mayroon ● “Kasi ako na yung naka kabit bawal ka na dumikit kay
na kaagad syang napproduce na effect receptor” so nagkakaroon ng antagonist receptor complex,
3. Inverse Agonist - a drug that binds to a same binding site the blocker or antagonist now will inhibit the intrinsic
as an agonist for that receptor and reverses constitutive activity or response.
activity of receptor
ANTAGONIST
● Si inverse agonist, mag ba bind sya kay receptor and
magkakaroon sya ng reverse activity. ● TYPES OF ANTAGONISM
● Magbabind pero opposite effect 1. Pharmacologic Antagonism
● Example: Ro15-4513 inverse agonist of 2. Physiologic Antagonism
benzodiazepine class of drugs 3. Chemical Antagonism

1. Pharmacologic Antagonism
● Inhibits the activity of an agonist by reacting with the
receptor or other part of the effectors’ mechanism

1.1 Pharmacodynamic Antagonist


● Produce an effect opposite of agonist by binding to the
same receptor
● Beta blocker (Propranolol) and Beta agonist (Epinephrine)
○ Beta blockers will bind to the receptor and they will
block the agonist
● Propranolol: Antihypertensive
○ Propranolol will bind to the receptor, which is the beta
receptor, and it will block epinephrine so that it will be
useful for hypertension.
○ Si epinephrine na beta agonist (beta receptor name
ng receptor) kapag nag bind sya sa receptor
mag-cacause siya ng vasoconstriction, tachycardia na
nag cacause ng hypertension.
● Muscarinic - receptor of parasympathetic
○ Acetylcholine - agonist
○ Atropine - blocker
■ Atropine will block binding of acetylcholine to
muscarinic receptor to prevent parasympathetic
action.

1.2 Pharmacokinetic Antagonist


● Receptors are inactive at first and if drugs, called as
ligands or neurotransmitters, can bind to a receptor so ● Reduce the effect of one drug by alteration of ADME
from inactive, ma-activate siya kapag nakapag bind na ● E.g., Digoxin + Cholestyramine
si drug kay receptor, it can produce intrinsic activity ○ Cholestyramine is a binding resin
● Anything that can bind to a receptor called ligand ○ When you add digoxin and cholestyramine, they will
● Antagonist - ayaw natin yung intrinsic activity na form a chelate or large complex. So, digoxin will not
ni-cacause ng agonist drug. So antagonist drugs has be absorbed and it will be excreted out from the body,
high affinity to receptor and they will block the binding making digoxin less effective.
of other drugs or ligand, dahil one at a time lang. ○ Cholestyramine prevents absorption and fastens the
Kapag may naka bind na sa receptor hindi na siya excretion of digoxin.
tatanggap ng iba.
2. Physiologic Antagonist
● Occurs when the drugs act independently at different
RECEPTOR INTERACTIONS
receptor sites, often yielding opposing actions
● Physiologic: action of the body/ biologic activity of the body
● Examples:
a. Histamine + Epinephrine
● Histamine is an endogenous substance inside our
body. Common siya, part ng physiologic response and
substance ng ating katawan.
● Histamine is the one that causes allergic reaction,
kaya kapag na expose ka sa alikabok, uubuhin ka,
sisipunin ka, kasi normal yon kasi na expose ka sa
isang antigen

21
PCOL 211 - Pharmacology 1

● Because of hyperreactivity and hypersensitivity, ● Kahit walang agonist dyan basta may kailangan
histamine can trigger anaphylactic shock, which can i-antagonist, pwede silang gumana
be deadly.
Competitive Antagonism
○ DOC: Epinephrine
● Physiologic antagonist, so diba kapag may ● Effects of antagonist can be overcome/reversed by
inflammation, vasodilated kaya hindi makahinga yung increasing the concentration of agonist
patient kasi kapag sa lalamunan yan at vasodilated ● Kapag masyadong madalas i-antagonist yung isang bagay,
yung sa blood vessel sa lalamunan, sumisikip yung baka naman magkaroon ng bad effect. To reverse that,
daanan ng hangin, mamamatay yung patient kasi di you can add more agonist, to reverse the action.
makakahinga. Dahil vasodilation ang effect ng
Non-Competitive Antagonism
histamine, bigyan natin ng vasoconstrictor, physiologic
antagonist, kasi yung normal na dilation na ni-cacause ● It is also known as irreversible antagonism
ng sarili nating katawan ng sarili nating substance like ○ Permanent action/activity
histamine i-aantagonize ng epinephrine kasi ○ Kapag nag-antaginize siya, permanent yung activity
mag-cacause sya ng vasoconstriction. Same with ● Example:
other examples: ○ Phenoxybenzamine
b. Histamine + Salbutamol ○ DNA-alkylating agents
c. Glucocorticoid + Insulin
ALLOSTERIC MODULATION
● Insulin lowers the blood levels that is caused by
glucocorticoid ● Is the regulation of an enzyme or other protein by binding
an effector molecule at the protein’s allosteric site.
3. Chemical Antagonist ● TYPES OF ALLOSTERIC MODULATION
● No receptor is involved 1. Positive Allosteric Modulation occurs when the binding
● Occurs when two drugs bind with each other to form an of one ligand enhances the attraction between substrate
inactive compound molecules and other binding sites. (They can enhance
● Examples: each other)
a. Paracetamol + N-Acetylcysteine ● a.k.a “Allosteric activation”
● Just like a toxin (toxic material) and antidote ● Example: Hemoglobin + Oxygen
● Paracetamol toxic amount: 8-10 tablet or 4000 mg 2. Negative Allosteric Modulation occurs when the binding
○ Toxic dose can cause liver damage of one ligand decreases the affinity for substrate at other
○ Antidote: N-Acetylcysteine active sites
● Both of them are drugs that will attack each other to ● a.k.a “Allosteric inhibition” (negative inhibition)
treat a patient ● Example: Glycerine + Strychnine (spinal convulsant)
b. Warfarin + Vitamin K ● Strychnine blocks Glycine receptor which is
● Warfarin (anticoagulant) + Vitamin K (coagulant) = numerous in the spinal cord which can lead to
Chemical antagonism convulsion
● Warfarin can cause bleeding, and Vitamin K can stop
the bleeding
c. Protamine SO4 + Heparin SO4 → forms a complex
devoid of action
● Protamine - (+) charged at physiologic pH
● Heparin - (-) charged at physiologic pH
● Heparin can also cause bleeding and protamine can
stop it
● Ma-neneutralize yung kanilang effect and maiiwasan
yung bleeding
d. Chelating agents ● So negative allosteric modulation, non-competitive
● Deferoxamine + Iron (Fe) inhibition so, when agonist bind to a receptor and it will
● Iron toxicity can be deadly so give deferoxamine para be block by an antagonist, inhibited yung receptor na
ma-treat yung toxic effect ng Iron ma-activate.
● May agonist na gustong mag bind pero dahil may
Nature of Antagonism
antagonist, hindi nya ibblock mismo yung binding site,
● It depends on whether or not they reversibly compete with but it will just bind to the receptor and change the
agonists for binding to receptors confirmation of the receptor that will deny now the
agonist drug, leading to inhibition of action.

22
PCOL 211 - Pharmacology 1

studies. Identify
Part 4: BASIC & CLINICAL EVALUATION (So may tatlo ka na na target organs of
icocompare na dose, na toxicity.
OF NEW DRUGS i-checheck mo para sa 2
species so ilang testing yon? (Eto need i-check
So kung 3 trial tas may 3 natin sa subacute
dose ka, may 9 ka na toxicity)
DRUG SCREENING kailangan tapos 2 specie
multiply by 2 so 18, tapos 4
● It involves a sequence of experimentation and weeks to 3 months pa kaya
characterization of drugs mahal magtest)
● Determination of the following:
1. Pharmacologic profile of the drug Chronic Rodent and non-rodent Goals of subacute
toxicity species. 6 months or longer. and chronic tests
2. Effects on cell function Required when drug is are to show which
3. Pharmacologic activity and selectivity of the new intended to be used in organs are
compound in comparison with reference compounds humans for prolonged susceptible to drug
periods. Usually run toxicity.
● Drug screening happens of kapag mag-didiscovered tayo
concurrently with clinical trial. Tests as noted
ng gamot and dadaan na sya sa mga test natin above for
subacute. 3 dose
Pharmacologic Profile Tests level plus controls.
● Experimental Method or Target Organ: Systems - blood
Effect on Effects on animal mating Examines fertility,
pressure reproductive behavior, reproduction, teratology,
● Species or Tissue: Dog, cat (anesthetized) performance parturition, progeny, birth perinatal and
- To check antihypertensive drugs gagamit tayo ng defects, postnatal postnatal effects,
development lactation
specimen like dog and cat kasi large animals sila
● Route of Administration: Parenteral Carcinogenic Two years, two species. Hematology,
● Measurement: Systolic-diastolic changes potential Required when drug is histology, autopsy
- Using sphygmomanometer or any mechanical intended to be used in studies.
humans for prolonged Tests in transgenic
machine to determine the blood pressure
periods. mice for shorter
periods may be
➢ Experimental Method or Target Organ: Respiratory permitted as one
effects species.
- Example: Drugs for lungs, asthma, pneumonia
Mutagenic Effects on genetic stability Increasing interest
➢ Species or Tissue: Dog, guinea pig potential and mutations in bacteria in this potential
- Guinea pig kasi yung anatomy nila similar sa mga tao (Ames test) or mammalian problem
➢ Route of Administration: Parenteral cells in culture: dominant
lethal test and clastogenicity (Macheheck na
➢ Measurement: Effects on respiratory rate and amplitude, in mice. don baka kasi
bronchial tone nagcacause sila
ng further defects
PRECLINICAL SAFETY & TOXICITY TESTING sa tao)

Type of Test Approach Comment Investigative Determine sequence and May allow rational
toxicology mechanisms of toxic action. and earlier design
Effects of large single doses up to the lethal level: Discover the genes, proteins, and identification
and pathways involved. of safer drugs.
Develop new methods for Possibly run at
Acute toxicity Acute dose that is lethal in Compare with assessing toxicity. higher compound
approximately 50% of therapeutic dose throughput.
animals and the maximum
tolerated dose. Usually two
species, two routes, single Goals of Preclinical Studies
dose.
● Identifying all potential human toxicities;
(For example 5 mg lang
○ Kasi hindi na pwedeng ituloy sa tao kung toxic na pala
toxic ba sya or effective ba
sya, so kung yung 5 mg na sya sa animals yung kung namamatay na pala sila
yon namatay yung limang lahat)
animal sa sampo, syempre ● Designing tests to further define the toxic mechanisms;
lethal yon, hindi mo na
pwedeng ituloy sa susunod ● Predicting the specific and the most relevant toxicities to
or babawasan mo na yung be monitored in clinical trials.
dose)
Quantitative Estimates are Determined:
Effects of multiple doses, which are especially important if the drug
is intended for prolonged use in humans: 1. “No-effect” dose - the maximum dose at which a
specified toxic effect is not seen
Subacute Three doses, two species. 4 Clinical chemistry, ● Example, ang sinabi niyo, dapat na toxic dose is 50
toxicity weeks to 3 months may be physiologic signs, tapos hindi sya nagcause ng toxicity.
necessary prior to clinical autopsy studies,
trial. The longer the duration hematology,
2. The minimum lethal dose - the smallest dose that is
of expected clinical use, the histology, electron observed to kill any animal
longer the subacute test. microscopy

23
PCOL 211 - Pharmacology 1

3. The median lethal dose (LD50) - the dose that kills


approximately 50% of the animals Part 1: AUTACOIDS - HISTAMINE
● So kung may 10 na test animal tapos namatay yung
kalahati don ichcheck kung anong dose yung namatay
sa kalahati ng population AUTACOIDS
● a.k.a. “Local hormones”
○ Because they are endogenous, they are part of our
body
● Endogenous substances with biological activity which is to
trigger allergic reaction
○ Main purpose of allergic reaction is to warn us, para
maalarma tayo na mayroon tayong na take na
antigenic substance
● Not released or stored in glands
● Not circulated in blood
● Are formed at the site of action where the antigen is being
administered or transferred so it triggers allergic reaction
● Produce localized action
● Autacoids - histamines are inflammatory agents and can
cause allergic reactions.

Classification of Autacoids
● Study of drugs takes a lot of time, tumatagal ng up to 20 1. Biologically active amines
years ang paggawa ng isang gamot kaya nag papa potent ● They contain amine structure
din yung mga discoverer para sila lang ang makapag ● Histamine - causes allergic reaction like rashes,
benta at kumita sila para mabayaran nila yung mga runny nose, watery eyes, itchyness, irritation and in
ginamit nila sa research some cases it can lead to anaphylactic shock,
● In vitro studies - 0 to 2 years average lead compound is ○ Anaphylactic shock - it is an an allergic reaction
being determined, so yung lead compound hahanapin for that can lead to death
example gusto mo na ang pinaka lead compound mo ● Serotonin a.k.a happy hormone - mood enhancer or
muna ay flavonoids, san mo sila gagamitin anti cancer ba stimulant.
antioxidant ba sila ganon, then chemical synthesis, isi ○ Maraming serotonin sa mga foods na maraming
synthesized mo kung ano ba yung specific na pwedeng tryptophan like, saging, balat ng saging,
gamitin na part or anong isang specific na flavonoid ang chocolate, it enhances serotonin release. At mga
magiging anti cancer or antioxidant. sweets, kaya kapag kumakain tayo ng sweets
● In animal testing - check kung toxic ba sya then after the nakakasaya and nattrigger yung release ng
animal testing if it is proven that it is safe in animals you serotonin
can now pass the IND to FDA don ka palang makakapag 2. Lipid derived autacoids (A.k.a Eicosanoids) - which
pass kapag complete na research mo sa animal, so after causes inflammation, pain, fever, clotting
passing of IND you will wait for the approval of FDA and ● Prostaglandins - triggers inflammation, pain, fever
kapag na approve doon palang mag proproceed sa ● Leukotrienes - also trigger/stimulate inflammation
clinical testing and it has 4 phases: ● Thromboxanes - triggers release of platelets which
● Phase 1 checks for the safety and pharmacokinetics of the cause clotting
drug product, san ba na absorb kelan nag liliberate, paano 3. Ergot Alkaloids - can come from natural sources like
madidistribute ano ba nag memetabolise sa kanya anong mushroom
enzyme, kelan sya ma eexcrete sa urine ba sa pawis ba or 4. Vasoactive polypeptides
sa feces. ● Kinins - like bradykinin, it can be seen on the lungs, it
● Phase 2 will check for the effectiveness of the drug, so triggers dry cough or irritancy of the lungs that causes
also throughout the clinical trials you will check for the cough
metabolism safety assessment ng isang gamot nga, ● Angiotensin - Angiotensin 1 and 2, that can trigger
naging effective ba sya vasoconstriction, trigger water and salt reabsorption
● Phase 3 does it work double blind? itest natin sa parehas (can cause pamamanas)
yung researcher and yung pinaka test human specimen, ● Endothelin - vasoactive peptides
yung human na kasali sa trial parehas nilang hindi alam ● Natriuretic peptide - affects salt reabsorption
yung pinapainom sa kanila so yun yung double blind in ● Vasopressin - also known as antidiuretic hormone, it
phase 3 para iwas bias. reabsorbed salt and water
● After the clinical trial, NDA will be passed again to the FDA ● Substance P
so it takes 8 to 9 years kasi matagal yung phase 3. If 5. Endothelium derived autacoids
approved ready na sya, ready na for market so phase 4 it ● Nitric oxide - other name EDRF (Endothelium derived
will further check if there are any adverse drug relaxing factor)
reactions/side effects to or in a large or many people.
Phase 4 at least 5000 human sample/ minimum of 5000.

24
PCOL 211 - Pharmacology 1

● Ano yung mangyayari sa blood vessels kapag natrigger


HISTAMINE
yung mast cells?
● Amine containing autacoids ○ Nagvavasodilate sila so yun yung effect ng autacoids
● Endogenous substance that triggers allergic reactions (histamine). Nagvavasodilate yung blood vessels,
● It comes from histidine. L-histidine decarboxylase enzyme lumalaki. Kapag lumalaki ang blood vessels,
converts histidine to histamine and this is a reaction that napupuno siya ng water. Nag eenhance kasi yung salt
requires pyridoxal phosphate and water reabsorption dumadami yung water kaya
nag-didilate talaga yung blood vessels. Kaya kapag
nag lead yung allergic reaction sa anaphylactic shock,
na very deadly type dahil sa pag vasodilate ng blood
vessels specially sa throat area, nagcclose yung air
way, hindi makakahinga si patient hanggang sa
ikamatay niya yon. Kapag naramdaman na ng patient
na nahihirapan na sya huminga, dapat within 5
minutes masaksakan na siya ng epinephrine (DOC for
anaphylactic shock) para magvasoconstrict yung
Distribution of Histamine
blood vessel sa part ng ating throat para makahinga
● Histamine is found in many tissues, including the brain. yung patient.
○ many tissues like, skin, respiratory tract, digestive
system, urinary tract Two processes by which histamine is released
● It is stored and found in the highest amounts in mast 1. Energy and Ca2+
cells and basophils. ● dependent degranulation reaction (Immmunologic
○ These are inflammatory cells that release histamine, Release)
thromboxane, interleukins that precipitate ● with the help of ATP (Adenosine Triphosphate) and
inflammation Calcium
○ Calcium can cause contractions of the muscles/
heart muscles
● In this type of reaction mast cells is synthesized by
surface IgE, so magbabind yung IgE na antibody natin
kay mast cells and it leads to degranulation. Kapag
nag-bind si antibody IgE kay mast cells marerelease
ang histamine (histamine release and histamine
degranulation). Then it will now cause allergic reaction
like vasodilation, rashes, itchy/runny nose that can
lead to a detrimental anaphylactic shock.
2. Energy and Ca2+
● Independent release (displacement)
● Chemical & Mechanical Release
● Paano ba naactivate ang mast cells/basophils? ○ Occurs following chemical or mechanical injury to
● In picture, makikita yung parang receptor, so yan yung mast cells
mga immunoglobulins specially IgE. IgE will attach to ○ Displacement is induced by drugs such morphine,
allergens, allergens are antigenic substances that tubocurarine, guanethidine, and amine antibiotics
triggers allergic reaction. Sila antigen ba-bind sila kay ○ Does not require energy
IgE, si IgE magbabind sa mga mast cells and it will ○ Nagkakaroon sila ng direct effect sa body natin,
trigger release of histamine that’s why kapag na expose especially the histamine
ka sa alikabok, ng isang allergen at na activate si IgE
Types of Histamine Receptors
and mast cells, nagkakaroon ka ng runny and itchy
nose, watery eyes because of histamine release.
H1 Receptor
● Mast cells are abundant in the following; ● Distribution:
○ Respiratory tract - kaya nga bigla kang uubuhin, ○ Smooth muscle (lungs)
manangangati lalamunan mo, skin allergies specially ○ Endothelium
hands and feet so kung na try mo na ma allergy ○ Brain
nangangati yung palad at talampakan mo and unti unti ○ Also found in blood vessels
na nagkakaroon ng rashes because of histamine ● Post Receptor Mechanism
release from the mast cells. ○ Increases IP3, DAG; Activates Gq protein
○ Gastrointestinal tract - in other cases, nagkakaroon ng ■ IP3 - inositol trisphosphate
increase peristalsis, nagdadiarrhea ang patient or ■ DAG - Diacylglycerol
nagiging hyperacidic. ● Partial Agonists
● Blood vessels store histamine in a granule bound in a ○ 2-(m-Fluorophenyl)-histamine
complex with heparin, adenosine monophosphate, and an ■ Histamine released from mast cells
acidic protein. ○ Smooth muscle can cause bronchoconstriction,
vasodilation because of endothelium, and can cause

25
PCOL 211 - Pharmacology 1

headache (kasi nagva-vasodilate yung mga meninges ● Partial Agonists


sa brain, kaya nahihilo kapag na-allergy) ○ Clobenpropit
● Partial Antagonists ○ Imetit
○ Mepyramine ○ Clozapine
○ Triprolidine ● Partial Antagonists
○ Mawawala yung constriction, vasodilation, headache ○ Thioperamide

H2 Receptor H1 RECEPTORS
● Location: Found in brain, heart, bronchi, gastrointestinal
● Distribution: tract, and vascular smooth muscles
○ Gastric mucosa (especially the stomach that ● Increases wakefulness vasodilation and an increase in
increases acid or HCl) permeability
■ H2 purpose is to increase HCl ○ Effects can cause headache, in heart increase
○ Cardiac muscle contractility, in bronchi bronchoconstriction kaya
○ Mast cells hindi makahinga patient, in GI tract it will increase
hyperacidity, release of acid kaya sinisikmura, and
○ Brain vascular smooth muscles magiging vasodilated
● Post Receptor Mechanism ○ Nag iincrease yung permeability kasi habang nag
○ Increases cAMP; Stimulate activation of Gs protein didilate sya numinipis yung wall kaya permeable
■ Increase in adenylyl cyclase sya kung mayroon mga ibang ions, or ibang
■ Increase of HCl acid production or secretion in substance na pumapasok sa blood vessels
gastric mucosa (hyperacidity) ○ So activation of H1 typically stimulates
non-vascular smooth muscles
■ Cardiac muscle - it can trigger contraction
■ It will trigger mast cells. It will release a lot of H2 RECEPTORS
histamine ● Location: Found in the brain, heart, vasculature and
■ It can cause headache; vasodilation of meninges parietal cells of the stomach
● Partial Agonists ● Increase acid production
○ Dimaprit ○ It is also a membrane bound, parietal cells is the
○ Impromidine one that releases HCl.
○ The response of H2 receptor is coupled by cyclic
○ Amthamine
amino monophosphate / amine monophosphate,
● Partial Antagonists activation of H2 receptors are nerve cells causes a
○ “-tidine” decrease in a histamine release
○ Ranitidine ○ The activation of H3 receptor on the vagal nerve
○ Tiotidine may cause decrease acetylcholine release yun
yung nagiging effect naman ng H3 sa utak.
H3 Receptor
● Histamine is released by your mast cells, mast cells
● Distribution:
should be first triggered by an allergen or an antigen.
○ Presynaptic: brain ● Pagkakasunod sunod:
○ Myenteric plexus ○ Maeexpose ka sa isang allergen or antigen na
● Post Receptor Mechanism tinatawag, si allergen magbabind kay IgE
○ Decrease cAmp, Ca2+; Activates Gi protein ○ IgE magbabind kay mast cell
■ Presynaptic: brain - Decreases sending of ○ Si mast cell magrerelease ng histamine
messages or decrease neurotransmission in the ○ Si histamine mag babind to the receptor either H1,
H2, H3, H4 so it will bind to the receptor and it will
brain (kaya yung iba hihimatayin kapag
cause different types of reactions or biologic
nagkakaroon ng allergy. Kumokonti na rin yung activities like increases wakefulness vasodilation
oxygen kasi humihina yung pag-flow ng air and and an increase in permeability or increase gastric
nagkukulang yung sending ng signal sa utak) acid production.
● Partial Agonists ● Membrane bound and coupled to G proteins ang ating
○ R-a-Methyl Histamine mga histamine and histamine receptor and they
activation causes an increase in the phospholipase C
○ Imetit
(PLC) activity leading to increase diacylglycerol (DAG)
○ Immepip and intracellular calcium and activation of H1 receptors
● Partial Antagonists in the brain increases wakefulness, kasi masakit yung
○ Thioperamide ulo mo.
○ Iodophenpropit
○ Clobenpropit
The Body’s Response to Histamine Release
H4 Receptor ● Blood clots (kasi nagrerelease ng thromboxanes)
● Distribution: ● Gastric acid secretion (kaya nagiging hyperacidic yung
○ Eosinophils - white blood cells that attacks parasitic patient)
microorganisms (worms) ● Blood vessels to dilate (that leads them to increase
○ Neutrophils - white blood cells that attacks bacteria permeability kasi lumalaki blood vessels, numinipis wall)
○ CD4 T-cells - killer cells ● Bronchoconstriction (nahihirapan huminga)
■ Kapag hindi na kinaya ng mga white blood cells ● Increases the permeability of the capillaries
● Post Receptor Mechanism ● Adrenaline is released (kasi kailangan mag compensate
○ Decrease cAMP, decrease Ca2+; Activates Gi protein pa rin ang katawan)

26
PCOL 211 - Pharmacology 1

● Swelling and inflammation (because of water and


Classes of Histamine (H1)-receptor antagonist
sodium reabsorption)
● Frequent heartbeat (because of increase contractility)
1. 1st Generation Antihistamines
HISTAMINE AGONISTS
a. Ethanolamines (Antiemetics)
● Histamine ● Will prevent emesis (vomiting)
○ R-alpha methylhistamine is an H3-specific agonist ● Can induce sleep
○ MOA: of R-alpha methylhistamine, it will bind to your ● Used for motion sickness
H3 receptor, it will activate your GI protein, decreasing ○ Kaya nasusuka or nahihilo kapag sumasakay sa
cAMP and calcium. bus, or nati-trigger ito kapag nakaamoy ng
● Betazole (Histalog) something pungent or mabaho, nakakahilo.
○ Tenfold greater activity at H2 receptors than at H1 ○ Yung mga mababaho or nakakahilong amoy,
receptors pwede silang maging allergen or antigen. It also
○ MOA: Will bind to your H2 receptor and will activate triggers histamine release kaya ka mahihilo and
your Gs protein increasing adenylyl cyclase, magsusuka
increasing cAMP, increasing release of histamine and ● Carbonoxamine, Dimenhydrinate, Diphenhydramine
increase of acid secretion. (Benadryl), Doxylamine (used as sleeping aid)
● Impromidine ○ MOA: It will block H1 receptor, preventing the
○ a investigational agent; its ratio of H2:H1 activity is binding of histamine and allergic reaction.
about 10,000 ○ Side effects: Causes dizziness and sleepiness
○ Diphenhydramine (Benadryl) - kilala sa market.
Uses of Histamine Agonists
25mg and 50mg.
● Used in allergy testing to assess histamine sensitivity ■ Dahil sleeping aid, yung iba nagiging
● In the test of gastric secretory function addiction. Ginagamit as sleeping pills,
○ Para ma-test kung gaano kadami yung nai-rerelease dinadamihan yung dose.
mo from normal ■ Yung 50mg na diphenhydramine, prescription
● Function (they have been largely supplanted for this use drug na. But before, yung mga antihistamine,
by pentagastrin) OTC drugs lang. Yung 25mg pwede pang
○ Especially yung mga histamine na nagbabind sa H2 maging OTC pero depende sa situation ng
patient.
Adverse E ect of Histamine Agonists
b. Ethylaminediamines:
● Flushing (pamumula because of vasodilation, pag nag ● Pyrilamine, Tripelennamine, Antazoline
vasodilated ang blood vessels marami rin ang nag fflow na ○ Can produce moderate sedation (sleepiness)
blood) ○ Can cause gastrointestinal upset
● Hypotension (will be triggered,bababa bigla yung blood c. Piperazine: Antiemetics, anti-motion sickness
pressure because of vasodilation pero pwedeng ● Hydroxyzine, Cyclizine, Meclizine
magkaroon ng reflex, tachycardia, kasi hypotensive ka ○ Meclizine - available in the Philippine market as
dahil nag vasodilate yung peripheris mo na blood vessels, Bonamine. It causes sedation. Iniinom 1 hour-30
bumaba yung blood pressure mo your heart will mins bago bumyahe
compensate for that. Tachycardia magiincrease yung d. Alkylamines:
contractility kaya kapag naactivate yung ibang receptor, ● Brompheniramine, Chlorpheniramine
komo nagiincrease ng contractility nga ng puso hindi lang ○ Agents which produce slight sedation
naman isa lang ang maaactivate pwedeng sabay sabay ○ Available in cold and cough preparation
silang lahat kaya maraming effects/adverse effects ang ■ Chlorpheniramine - bioflu, alaxan
histamine agonist) e. Phenothiazines:
● Tachycardia (compensation for vasodilation) ● Promethazine (antiemetic)
● Headache (because of the vasodilation ng mga meninges ○ Weak alpha-adrenoceptor antagonist. It can
sa mga utak natin, dumidikit na kasi yung mga meninges decrease blood pressure
sa skull kaya sumasakit sila, manipis lang kasi yung skin f. Miscellaneous:
or yung membrane ng meninges kaya kapag dumikit sa ● Cyproheptadine (antihistamine, anticholinergic, and
skull masakit, kaya ka nahihilo or sumasakit yung ulo) antiserotonin activities)
● Wheals (pantal-pantal, bumps) ○ Dahil anticholinergic, mayroong sympathetic
● Bronchoconstriction (mahihirap ang huminga patient) action. For example, pwedeng magkaroon ng
● Gastrointestinal upset (because of increase release of slight tachycardia, mag increase yung heart rate.
HCl sa stomach) ○ Magbo-bronchodilate yung patient, makakahinga
siya ng mabuti.
Histamine (H1) - Receptor Antagonists
○ Antiserotonin - magkakaroon ng slightly pressure
● Aka: Classical ANTIHISTAMINES in your mood
● Competitive inhibitors at the H1-receptor
When dispensing antihistamine, tatanungin niyo kung ano
● Antihistamines will block the H1 receptor to prevent yung trabaho ng patient. “Yung gamot like chlorpheniramine,
binding of agonists, to prevent intrinsic activity nakakaantok, pwedeng mahilo kapag naglalakad. Iwasang
○ Para hindi mag-cause ng allergic reactions magmaneho ng sasakyan or pumunta sa matataas na lugar,
kasi pwedeng maaksidente or mahulog sa hagdanan yung

27
PCOL 211 - Pharmacology 1

○ They will inhibit muscarinic receptor, preventing


taong nakainom ng antihistamine”
Ask them kung ano ang gagawin nila sa maghapon, and release of acids in the stomach, decreases peristalsis
advise them to stay at home and avoid driving and operating (kaya hindi ka magsusuka)
machines. ○ Prevents bronchoconstriction (kaya makakahinga)
● Block mucus secretion (kaya hindi ka sisipunin, hindi
1st generation are lipophilic. They can pass through the maluluha luha) and sensory nerve stimulation (kaya
blood brain barrier, that is why they can cause sedation. nakakaantok) because of anticholinergic activity
● Effective local anesthetics
2. 2nd Generation Antihistamines ○ Sinasama sa mga general anesthetics para kapag
a. Piperidines: ooperahan, decrease yung mucus secretion at acid sa
● Terfenadine (Seldane) stomach, para habang hinihiwa habang inoopera,
○ Metabolized (CYP450) to the active metabolite hindi magsusuka bigla at hindi maghhyperacidic yung
fexofenadine tyan, hindi uubuhin, hindi kikislot, at nakakaantok
● Fexofenadine (Allegra) ○ Eg. Dimenhydrinate and promethazine
○ Approved as safer antihistamine of this class ● Relax histamine-induced contraction of bronchial smooth
● Astemizole (Hismanal) muscle and have some use in allergic bronchospasm.
○ A/E: ventricular tachycardia ○ Sila yung magca-cause ng bronchodilation para
b. Miscellaneous: makahinga na yung pasyente
● Loratadine (Claritin), Cetirizine (Zyrtec) ● Block the vasodilator action of histamine.
○ Zyrtec - not associated with cardiac ○ Magca-cause ng vasoconstriction para hindi malaki
abnormalities, it has poor penetration in CNS, yung blood vessels, para lumuwag yung airway,
less sedating, ineffective for motion sickness especially sa throat area
○ Not used for motion sickness and antiemesis ● Inhibit histamine-induced increases in capillary
permeability.
Advantage of 2nd generation: no sedation. Pwede silang ○ Para hindi nakakapasok yung mga ions. For example,
i-dispense sa mga working.
sodium, hindi niya madaling mape-penetrate yung
Less penetration in blood brain barrier, that is why they are blood vessels dahil na babalik sa normal size yung
less sedating. Their common use is for allergic rhinitis capillaries
● Frequently, it causes CNS depression or sedation
Pharmacokinetic Properties of H1 Blocking Drugs ○ Marked by sedation, decreased alertness, and
decreased appetite
A. Well absorbed after oral administration. ● In children and some adults, these agents stimulate the
● Lipophilic CNS.
● Normal effects seen in 30 minutes (with maximal ○ Depende kung anong receptor ang na-bind-an
effects at 1-2 hours)
● The duration of action is: THERAPEUTIC USES - H1-receptor Antagonists
○ 1ST generation compounds: 3-6 hours ● Treatment of allergic rhinitis and conjunctivitis.
○ 2ND generation compounds: 3-24 hours ○ Allergic rhinitis - runny nose
■ Longer action but less sedating ○ Conjunctivitis - red eyes, teary eyes
B. Lipid-soluble = cross the blood-brain barrier (that can ● Treat the common cold based on their anticholinergic
cause sedation) properties
C. Metabolized in the liver; ○ Diphenydramine also has an antitussive effect not
● many induce microsomal enzymes and alter their own mediated by H1-receptor antagonism.
metabolism ● Treatment of urticaria and atopic dermatitis, including hives
● Sa utak natin, sa hypothalamus, nandon yung chemo ○ Uticaria - teary and itchy eyes
trigger zone ○ Atopic dermatitis - rashes
● If chemo trigger zone is activated, it will also activates ○ Hives - cold, cough and itchiness
your vomiting center, na nagti-trigger ng emesis kaya ● Sedatives
nasusuka ka. Kapag nagkakaroon ng imbalance sa ○ Because it can penetrate the CNS, its lipophilic, and
katawan natin (kapag nagtatravel ka, minsan matagtag
causes sedation
ang daan, minsan smooth lang, or pataas)
● Yung nagiging allergen is yung motion na ○ Several (doxylamine, diphenhydramine) are marketed
nararamdaman ng external body natin. as over-the-counter (OTC) sleep aids.
● Then si histamine ay mare-release, magti-trigger niya ● Prevention of motion sickness
yung sa brain, maa-activate ang chemo trigger zone at ○ Because it will inhibit chemo trigger zone, the vomiting
vomiting center, masusuka ka. center
○ Nagkakaroon ng GI upset due to release ng acids ● Appetite suppressants
sa tiyan kaya nagsusuka.
ADVERSE EFFECTS - H1-receptor Antagonists
PHARMACOLOGIC ACTIONS - H1-receptor Antagonists (Significantly reduced with second-generation agents)

● Ethanolamine, phenothiazines, and ethylenediamines ● Sedation (synergistic with alcohol, other depressants,
○ have anticholinergic activity. dizziness, and loss of appetite)
○ CAUTION: drivers and machine operators

28
PCOL 211 - Pharmacology 1

○ Avoid taking antihistamine together with alcohol and


Inhibitors of histamine release characteristics
depressants, can lead to comatose
● Gastrointestinal upset, nausea, constipation and diarrhea. 1. Poorly absorbed salts;
○ Nausea - sedation, nakakaantok. Ang una mong ● ROUTE of administration: inhalation
mararamdaman doon aya medyo mahihilo ka 2. They inhibit the release of histamine and other autacoids
○ Constipation is caused by anticholinergic activity of from the mast cell.
antihistamine. Nape-prevent yung peristalsis, kasi ● Increase influx of Chloride ions (negatively charged,
kumokonti yung acids sa tiyan kaya hindi causing relaxation of the intracellular, relaxation of the
mabe-breakdown nang mabuti yung kinakain smooth muscles, preventing bronchospasms
● Anticholinergic effects (dry mouth, blurred vision, and urine especially in asthmatic patients)
retention). 3. Prophylactic agents in asthma
○ Stops secretion 4. Nedocromil sodium - appears to be more effective in
○ Stop release of saliva kaya nagiging dry ang mouth reducing bronchospasm caused by exercise or cold air.
○ Stop signaling in brain kaya nakakaantok at lumalabo 5. Adverse effects: sore throat and dry mouth.
ang paningin kasi nahihilo ka na
○ Stop production of mucus at fluids kaya nagca-cause
ng urine retention, parang natutuyot, hindi ka naiihi
Part 2: AGENTS USED IN PEPTIC ULCER
H2-receptor Antagonists
DISEASE
1. Histamine (H2)-receptor antagonists include
● Cimetidine (Tagamet)
● Ranitidine (Zantac) - part of emergency drugs in PHYSIOLOGY OF GASTRIC ACID SECRETION
hospitals
● Famotidine (Pepcid AC)
● Nizatidine (Axid)
2. Competitive antagonists at the H2receptor, which
predominates in the gastric parietal cell.
● H2 receptor antagonist is more on inhibiting HCl acid
release. More on hyperacidity, ulcer, gastrointestinal
upset, gastroesophageal reflux disease (GERD),
heartburn
● H2 receptors are found in stomach (parietal cells)
○ Parietal cells release HCl
● MOA: They will block H2 receptors, preventing binding
of histamine to parietal cells, preventing the release of
HCl in the stomach

Uses

● Treatment of gastrointestinal disorders: ● Apical surface - Meal stimulated (GASTRIN)


● Heartburn and acid-induced indigestion ○ Mga taong may GERD ay lalong mag-aacid reflux
○ Heartburn - caused by GERD (reflux - bumabalik) pag uminom ng gatas since may calcium yon,
○ Kapag sinisikmura, sumasakit yung dibdib. Kasi yung mag-aactivate yung calcium ions tas mag-pproduce
pa lalo ng acid or HCl
acid, it goes to esophagus. Yung acid in the stomach,
● H+ - binds to chloride ions that results to the production
dahil sobra sobra na siya, umaakyat sia papuntang of HCl
esophagus. ● Gs - paracrine
○ If you are hyperacidic, hindi ka makakakain at hindi ● Histamine - triggered by food allergen
matutunaw nang mabuti kasi nagfo-flow siya nang ● Ca2+ (acetylcholine) - neurocrine
pataas, kaya nagca-cause ng pagsusuka ● Ca2+ (gastrin) - endocrine
● Promote the healing of gastric and duodenal ulcers ● Acetylcholine - produced when parasympathetic
nervous system is activated (during night) rest and
because they will lower down HCl acid release
digest
○ Kapag pinindot mo yung center (bandang taas), tapos
sumakit, duodenal ulcer
Peptic Ulcer
○ Kapag sa medyo gitna (right side), gastric ulcer
● Hypersecretory states such as Zollinger-Ellison syndrome ● A circumscribed loss of the mucous membrane of the GIT
(composed of different symptoms: hyperacidity, vomiting, system exposed to gastric juices containing acid and
diarrhea) pepsin
● Other causes are Helicobacter pylori
Inhibitors of histamine release

● For asthmatic reaction caused by cold air, allergens


○ Cromolyn (Intal)
○ Nedocromil sodium (Tilade)

29
PCOL 211 - Pharmacology 1

● Produces alkaline environment enabling survival in


stomach
● Almost all duodenal and ⅔ gastric ulcers pt’s infected with
HP
● Considered class 1 carcinogen gastric cancer

NSAIDS

● Damage to the cytoprotective role of PGs - PGE2 AND


PGI2

Duodenal Vs Gastric Ulcers

Duodenal Gastric

● Age: 25-75 years ● Age: 55-65 years old


● Gnawing or burning ● Relieved by food but pain
upper abdomen pain may persist even after
relieved by food but
eating
reappears 1-3 hours after
meals ● Anorexia, weight loss,
● Worse pain when vomiting
stomach empty ● Infrequent or absent
● Bleeding occurs with remissions
deep erosion ● Small % may erode
- Hematemesis - through stomach wall
nagsusuka ng dugo
● Severe ulcers may erode
- Melena - black stool
● Sinisikmura after kumain through stomach wall

Why Ulceration Occurs?

● Imbalance primarily between Aggressive factors and


Defensive factors:
● Aggressive factors: (nagttrigger para magka ulcer)
○ Acid
○ Pepsin Differentiating between H. pylori and NSAID-induced
○ Bile ulcer
● Defensive factors:
○ Mucus Ulcers associated with H. Ulcers associated with
○ HCO3 pylori NSAIDS
○ PG
● More often in duodenum ● Ulcers associated with
What may contribute to imbalance? ● Often superficial NSAlDs
● Less severe Gl bleeding ● More often in stomach
● Helicobacter pylori
● Often deep
● NSAlDs
● More severe Gl bleeding
● Ethanol
● Sometimes
● Tobacco
asymptomatic
● Severe physiologic stress (Burns, CNS trauma, Peptic
Ulcer Disease Surgery, Severe medical illness)
● Steroids Signs and Symptoms

H. pylori ● Gnawing, burning or aching pain (tends to worsen at night


and occurs 1-3 hours after meals)
● Gram (-) rod with flagella ● Nausea, vomiting, belching and significant weight loss.
● H. pylori is most common cause of PUD
● Transmission route fecal-oral
● Secrets urease conver urea to ammonia

30
PCOL 211 - Pharmacology 1

○ Prevent aspiration pneumonia


Complications of PUD
○ Hypersecretory states
1. Bleeding Clinical Comparison of H2-Receptor Blockers
● occurs in 25-33%of patients
Drug Relative Dose to achieve > Usual Dose Usual Dose Usual
● most frequent complication and maybe lifethreatening
Potency 50% Acid for acute for Dose for
● account for 25% of ulcer deaths Inhibition for 10 duodenal or Gastroeso- Prevention
● may be the first indication of an ulcer hours gastric ulcer phageal of Stress
2. Perforation Reflux Related
Disease Bleeding
● occurs in about 5% of the patient Cimetidine
● accounts for 2/3 of ulcer deaths 1 400-800 mg 8OO mg 800 mg 50 mg/h
HS or bid continuo
3. Obstruction from edema or scarring 400 mg us
● often due to pyloric channel ulcers bid Infusion
● occur with duodenal ulcer
Rantidine
● causes incapacitating, crampy abdominal pain 4-10 150 mg 300 mg 150 mg 6.25
HS or bid mglh
● may lead to total obstruction with intractable vomiting
150 mg continuo
4. Intractable pain bid us
infusion
Goal of Therapy with Anti-ulcer Drugs 50 mg
IV every
1. Reduce gastric acid production 6-8 h
2. Neutralize gastric PH
Nizatidine
3. Protect the walls of the stomach from the acid and pepsin 4-10 150 mg 300 mg 150mg Not
released by the stomach MS or bid availabl
150 mg e
4. Treat peptic ulcer and reflux esophagitis bid

Classification Famotidine
20-50 20 mg 40 mg 20 mg 20 mg
HS or 20 bid IV every
1. Acid Neutralizing agents: (ANTACIDS)
mg bid 12 h
● Systemic: Sodium Bicarbonate and Sod. Citrate
● Nonsystemic: Magnesium hydroxide, Mag. Treisilicate,
Aluminium hydroxide gel, Magaldrate and calcium A. Cimetidine
carbonate
● Reduces acid secretion by 70% for 4-5 hours (300mg qid)
2. Reduction in Gastric acid secretion:
● Bioavailability is reduced by antacids
● H2 antihistamines: Cimetidine, Ranitidine, Famotidine,
● Decreases the absorption of ketoconazole
Nizatidine and Roxatidine
● Major adverse effects include :
● Proton pump inhibitors: Omeprazole, Lansoprazole
● Thrombocytopenia
Pantoprazole, Rabeprazole and Esomeprazole
● Gynecomastia and impotence
● Anticholinergics: Pirenzepine, Propantheline and
○ (androgen receptor antagonist) mental confusion in
Oxyphenonium
the elderly
● Prostaglandin analogue: Misoprostol
● Low incidence of mild gastrointestinal upset, headache
3. Ulcer protectives:
● Potent inhibitor of CYP450
● Sucralfate, Colloidal Bismuth sudcitrate
● IV: dementation and bradycardia (elderly)
4. Anti-H. pylori Drugs:
● Amoxicillin, Clarithromycin, metronidazole, tinidazole B. Ranitidine
and tetracycline
● Five to ten times more potent than cimetidine
H2 Receptor Antagonists ● Does not bind to androgen receptor
● Secreted in milk therefore it should not be given in
● - “tidine”
lactating mothers
○ Cimetidine
● Low incidence of headache and cutaneous rash
○ Ranitidine
● Hepatotoxic
○ Nizatidine
○ Famotidine C. Famotidine
● Mechanism of action:
● Approximately twice as potent as ranitidine
○ decrease gastric acid secretion through competitive
● Tachyphylaxis compromise its long term use
inhibition of H2 receptors
● has a longer duration of action
A. INHIBITORS OF GASTRIC ACID PRODUCTION ● Produces fewer side effects similar to those of ranitidine
● Most potent H2 blocker
1. H2 Receptor Antagonist ● Mild cardiotoxic

● Uses: D. Nizatidine
○ GERD
● As effective as ranitidine and may be administered once
○ Duodenal and gastric ulcer
daily
○ Non-ulcer dyspepsia
● May produce hepatotoxicity but it does not inhibit drug
○ Prophylaxis for recurrent ulcers in patients
metabolism in the absence of liver damage
○ Control of reflux esophagitis and bile reflux gastritis

31
PCOL 211 - Pharmacology 1

● Does not bind to androgen receptors


PPI - Dosage Schedule
Comparison of H2 Antagonists
Omeprazole 30 mg o.d
Cimetidine Ranitidine Famotidine Nizatidine
Lansoprazole 30 mg o.d
Bioavailabi 80 50 40 >90
lity
Pantropazole 40 mg o.d
Relative 1 5 - 10 32 5 - 10
Potency Rabeprazole 20 mg o.d

Half life 1.5 - 2.3 1.6 - 2.4 2.5 - 4 1.1 - 1.6 Esomeprazole 20 - 40 mg o.d
(hrs)

Duration of 6 8 12 8
action
(hrs)

Inhibition 1 0.1 0 0
of CYP
450

Dose mg 400 150 20 150


(bd)

Antiandrogenic effect, prolactin secretion and gyncomastia

2. Proton Pump Inhibitors (PPIs)

● - "prazole"
○ Omeprazole
○ Lansoprazole
○ Rabeprazole
○ Pantoprazole
○ Esomeprazole 3. Anticholinergics
● USES: ● Includes propantheline, isopropamide and scopolamine
○ GERD ● Decrease ACh-stimulated secretion and motility in the
○ Duodenal and gastric ulcer. H.pylori ulcer, gastrointestinal tract
NSAlD-induced ulcer ● Required doses produce systemic anticholinergic effects
○ Prevention of rebleeding from peptic ulcer ● They are rarely used alone but they are useful as adjuncts
○ Non-ulcer dyspepsia in patients resistant to H2 blockers
○ Prevention of stress-related mucosal bleeding
○ Gastrinoma and Hypersecretory states B. CYTOPROTECTIVE AGENTS

A. Omeprazole A. Sucralfate
● Given as delayed release capsule because of acid lability
● A salt of sucrose complexed to sulfated aluminum
● Antisecretory effects occurs within 1 hour with the
hydroxide
maximum effects occurring within 2 hours
● has an affinity for exposed proteins in the crater of peptic
● It may produce abdominal pain, nausea, diarrhea,
ulcer
vomiting, rash, constipation, headache, asthenia and back
● Protects ulcerated areas from further damage and
pain
promotes healing
● May inhibit the metabolism of warfarin, diazepam, and
● MOA: Stimulates mucosal production of prostaglandins
phenytoin
and inhibits pepsin
● It inhibits the absorption of ketoconazole
○ It may produce constipation and nausea, gastric
● Contraindicated in pregnancy
discomfort, indigestion, dry mouth, rash, pruritus, back
B. Lansoprazole pain, dizziness, sleepiness and vertigo,
hypophosphatemia
● Acid labile and administered as an enteric coated tablet
● Prodrug that requires protonation for activation
● Most effective when given 30 to 60 mins before meals
● Its acid inhibitory effects is greater than 24 hours
● Adverse effects include abdominal pain, nausea and
diarrhea
● It can increase theophylline clearance
● It is contraindicated in asthma

32
PCOL 211 - Pharmacology 1

● MOA: Coats ulcers and erosions, creating a protective


layer against acid and pepsin.
○ It may also stimulate prostaglandin, mucus, and
bicarbonate secretion

Bismuth

● has direct antimicrobial effects and binds enterotoxins,


accounting for its benefit in preventing and treating
traveler's diarrhea.
● Direct antimicrobial activity against H pylori
● Clinical Uses:
○ Nonspecific treatment of dyspepsia and acute
diarrhea
○ Prevention of traveler's diarrhea (30 mL or 2 tablets
four times daily)
○ For the eradication of H pylori infection
● Taken on empty stomach 1 hr. before meals ● Adverse Effects:
● Concurrent antacids, 1-12 antagonist avoided (as it needs ○ Causes blackening of the stool
acid for activation) ○ Liquid formulations may cause harmless darkening of
● Uses: the tongue.
○ NSAID induced ulcers ○ Bismuth agents should be used for only short periods
○ Patients with continued smoking and should be avoided in patients with renal
○ ICU insufficiency.
○ Topically - burn, bedsore ulcers, excoriated skins ○ Bismuth toxicity resulting in encephalopathy (ataxia,
● Dose: 1 gm 1 Hr before meals headaches, confusion, seizures).

B. Misoprostol CARBENOXOLONE

● A prostaglandin E1 analog ● Synthetic derivative of glycyrrhizic acid


● More water soluble and it has a half-life than naturally ● Heals both gastric and duodenal ulcers
occurring prostaglandin ● MOA: Increases production, secretion and viscosity of
● Inhibits gastric acid secretion and increases mucosal intestinal mucus
resistance ● A/E: Aldosterone effect
● MOA: Increases mucus and bicarbonate secretion by the C. ANTACIDS
gastric epithelium by increasing epithelial regeneration and
by enhancing mucosal blood flow, thus enhancing mucosal ● Weak bases that are taken orally and that partially
protection neutralizes gastric acid and reduce pepsin activity.
● Produces uterine contractions therefore contraindicated in ● Reduce the pain associated with ulcers and may promote
pregnancy healing
● High doses are required for healing: 40 mEq of base seven
times daily

Types of Antacids

● Systemic antacids
○ Sodium bicarbonate
○ Calcium carbonate
● Nonsystemic antacids
○ Aluminum hydroxide (Amphojel)
○ Dihyroxyalumium sodium (Rolaids)
○ Calcium carbonate (Tums)
○ Magaldrate (Riopan)
○ Magnesium hydroxide and aluminum hydroxide
(Maalox, Mylanta, Gelusil)

MUCOSAL PROTECTIVE AGENTS

● Colloidal Bismuth Compounds


○ Bismuth subsalicylate
○ Bismuth subcitrate
○ Bismuth dinitrate

33
PCOL 211 - Pharmacology 1

ANTACIDS

● SODIUM BICARBONATE
○ Absorbed systemically and should not be used for
long-term treatment
○ Contraindicated to hypertension due to its high sodium
content
○ Potent neutralizing capacity and acts instantly
○ ANC: 1 gm 12 mEq
○ NOT USED ANYMORE FOR ITS DEMERITS:
■ Systemic alkalosis
■ Distension, discomfort and belching - CO2
■ Rebound acidity
■ Sodium overload
● CALCIUM CARBONATE
○ Partially absorbed from the gastrointestinal tract and
have some systemic effects
Di erences in the types of Antacids ○ Should not be used for long term use
○ May stimulate gastrin release and thereby cause
● Cation content rebound acid production
● Neutralizing capacity ○ Contraindicated in renal disease
● Duration of action ● ADR:
● Side effects ○ Hypercalcemia, Alkalosis
● Cost ○ Renal failure (milk-alkali syndrome)
Neutralizing Capacity ● MAGNESIUM HYDROXIDE
○ Not absorbed in the GIT therefore produces no
● ANC = number of mEq of HCI required to maintain 1 mL of systemic effects
an antacid suspension at pH 3 for 2 hr in vitro. ○ Can be used for long term therapy
● Factors that can cause variation in the rate of ○ May produce diarrhea
neutralization. ● Magnesium containing preparation
○ Degree of comminution ○ Diarrhea
○ Crystal form ○ Hypermagnesemia
○ Precipitants used ● ALUMINUM HYDROXIDE
○ Presence of reactive suspending agents ○ Has no systemic effects and causes constipation
○ Also hypophosphatemia and osteomalacia
Dosing Interval
● Prolonged Aluminum use
● Ideal antacid should be rapid in onset and provide a ○ Phosphate depletion
continuous buffering action ○ Osteoporosis, osteomalacia„ neurotoxicity
● Rapid onset: Mgo, CaC03
Combination Products
● Slow onset: Mg trisilicate and aluminum compounds
● Duration of buffering action: ● Various preparations that combine magnesium hydroxide
○ determined by the administration of antacid and aluminum hydroxide
○ With food: action will last for 2 hr ○ To achieve a balance between agents adverse effects
○ An additional 3 hr meals will extend the buffering time on the bowel.
by 1 hr ● Examples of which are Maalox, Mylanta and Gelusil.
○ Ideal dosing interval: 1 and 3 hr after meals and at 1. Combine fast and slow reacting antacids to obtain a
bedtime product with a rapid onset and relative even, sustained
action.
Side E ects
2. Lower the dose of each component and minimize the
● Systemic antacid: Sodium bicarbonate possibility of certain ADR
○ Soluble and readily absorbed 3. Use one component to antagonize one or more side
○ Can cause electrolyte disturbance and alkalosis effects of another component
● Non- systemic antacids: Al, Ca, Mg
Simethicone
○ Form insol compounds in the GIT

The Patient ● Not an antacid


● Used to defoam gastric juice to decrease the incidence of
● Patients with increase risk of antacid toxicity gastroesophageal reflux
○ Heart failure = excess sodium intake (inc. toxicity)
○ Renal failure = should not use magnesium containing Somatostatin
antacids ( can cause hypermagnesemia) or sodium ● Is a 14 amino acid peptide that is released in the
bicarbonate ( systemic alkalosis) gastrointestinal tract and pancreas from paracrine cells,
■ Given with aluminum containing antacids for their
phosphate lowering effect

34
PCOL 211 - Pharmacology 1

D-cells, and enteric nerves as well as from the


hypothalamus
● Short half-life in the circulation (3 minutes) when it is
administered by intravenous injection
● It is a key regulatory peptide that has myriad physiologic
effects:
1. It inhibits the secretion of numerous hormones,
including gastrin, cholecystokinin, glucagon, growth
hormone, insulin, secretin, pancreatic polypeptide,
vasoactive intestinal peptide, and 5- HT3.
2. It reduces intestinal fluid secretion and pancreatic
secretion.
3. It slows gastrointestinal motility and inhibits
gallbladder contraction.
4. It induces direct contraction of vascular smooth
muscle, leading to a reduction of portal and splanchnic
blood flow.
5. It inhibits secretion of some anterior pituitary
hormones. Triple Therapy

Octreotide
Some other Triple Therapy Regimens are:
● Is a synthetic octapeptide with actions similar to
somatostatin. Bismuth subsalicylate 2 tab qid
● When administered intravenously, it has a serum half-life of
1.5 hours. Metronidazole 250 mg qid
● It also may be administered by subcutaneous injection,
Tetracycline 500 mg qid
resulting in a 6- to 12-hour duration of action.
● A longer-acting formulation is available for once-monthly
depot intramuscular injection. Rantidine Mismuth citrate 400 mg bd
● Adverse Effects:
Tetracyclind 500 mg bd
○ Impaired pancreatic secretion may cause steatorrhea,
which can lead to fat-soluble vitamin deficiency.
Clarithromycine / Metronidazole 500 mg bd
○ Alterations in gastrointestinal motility cause nausea,
abdominal pain, flatulence, and diarrhea.
○ Acute cholecystitis
○ Prolonged treatment with octreotide may result in
hypothyroidism.
○ Can cause bradycardia.
Triple Therapy

The BEST among all the Triple Therapy regimen is:

Omeprazole / Lansoprazole 20 / 30 mg bd

Clarithromycin 500 mg bd

Amoxycillin / Metronidazole 1 mg / 500 mg bd

Given for 14 days followed by PPI for 4 - 6 weeks


Short regimens for 7 - 10 days not very effective
Nonpharmacologic Measures

● Avoid tobacco
● Avoid alcohol
● Weight loss
● Avoid hot, spicy, and greasy foods
● Take any NSAIDs including aspirin and oral glucocorticoids
with food or in deceased dosage
● Sit upright
● Do not eat before bed
● Wear loose-fitting clothing

35
PCOL 211 - Pharmacology 1

PLATELETS
PHARMACOLOGY OF SEROTONIN
● Component of the platelet clotting process

SMALL INTESTINE (Enterochroma n cells)


SEROTONIN
● Approximately 90% of serotonin is found in the
● (5-Hydroxytryptamine)
enterochromaffin cells of the gastrointestinal tract.
● An important neurotransmitter
● Increases GIT motility
○ Regulates mood, sleep, appetite, and temperature
regulation, as well as the perception of pain, the RECEPTORS OF SEROTONIN
regulation of blood pressure, and vomiting
● A local hormone (Autacoid) in the gut
● It is a componentofthe plateletclotting process
● Tryptophan - amino acid precursor of Serotonin

Most of serotonin receptors are Type I l (G -protein linked)


Biosynthesis and Metabolism of Serotonin
except 5HT3 are coupled to a Na+-K+ ion channel, Type I
● BIOSYNTHESIS (Ionotropic receptor)
Trytophan (from diet)
SEROTONIN AGONISTS
↓ Tryptohan hydroxylase
5 Hydroxytryptophan Serotonin Details Brands

↓ Aromatic L-A.A decarboxylase agonist

5-Hydroxy tryptamine Buspirone - is a relatively specific Buspar


(Buspar) 5HT1a-receptor agonist.
↓ Monoamine oxidase (MAO) - Is useful for the
managüment of anxiety
5 Hydroxyindole acetaldehyde disorders.

↓Aldehyde dehydrogenase Dexfenfluramine - 5HT2c agonist


5 Hydroxyindoleacetic acid (5 HIAA) - Used as appetite
suppressant
● Metabolized by MAO - Withdrawn: cardiac valve
● Intermediate product = 5-hydroxyindoleacetaldehyde, is toxicity
further oxidized by aldehyde dehydrogenase to
5-hydroxyindoleacetic acid (5-HIAA). Triptans - a 5HT 1B/1D-receptor Sumatriptan
● 5HIAA ● Sumatriptan agonist. (Imigran)
○ Excreted in urine (Imitrex, - promotes the contraction
○ Can be used as a diagnostictest for tumor = Imigran) of vessels in carotid
● Naratriptan circulation. This reduces
especially carcinoid tumor
● Zolmitriptan the rebound vasodilation
● Rizatriptan associated with migraine
SEROTONIN (5-Hydroxytryptamine) headaches.
- used for the treatment of
acute migraine.
BRAIN (Neurotransmitter)
- A/E: Hypertension
● Regulates mood, sleep, appetite (low levels result to
anxiety, depression and sleep trouble) Cisapride - 5HT4 agonist Propulsid
● temperature regulation (Propulsid) - speeds gastric emptying
● Perception of pain and increases lower
● Regulation of blood pressure esophageal pressure.
● Vomiting - Use: Treat nocturnal
● METATONIN heartburn due to reflux,
constipation, and anorexia
○ N-acetyl-5-methoxytryptamine methoxylated and
nervosa.
N-acetylated product of serotonin found in the pineal
- Not available in the
gland. Philippines. In USA, it is
○ Regulate sleep-wake behavior of human "compassionate use"

36
PCOL 211 - Pharmacology 1

Tegaserod - 5HT4 partial agonist Withdrawn due Neuroleptic Malignant Syndrome


- used for irritable bowel to
syndrome with cardiovascular ● Precipitating Drugs
constipation toxicity ○ D2-blocking antipsychotics
● Clinical Presentation
Ergot alkaloids - 5HTID agonist ○ Acute severe parkinsonism; hypertension
(Claviceps - ERGOTAMINE: Used for hyperthermia, normal or reduced bowel sounds, onset
purpurea) prevention and treatment over 1-3 days
of migraine.
● Therapy:
- ERGONOVINE- Used in
the management of
○ Diphenhydramine (parenteral), cooling if temperature
post-partum hemmorhage is very high, sedation with benzodiazepines

Malignant Hyperthermia
Selective Serotonin Reuptake Inhibitors ● Precipitating Drugs
● The most common antidepressants in clinical use. ○ Volatile anesthetics, succinylcholine
● MOA:Inhibition ofthe serotonin transporter (SERT) ● Clinical Presentation
● Examples: ○ Hyperthermia, muscle rigidity, hypertension,
1. Fluoxetine(Prozac) tachycardia; onset within minutes
2. Sertraline (Zoloft) ● Therapy
3. Citalopram (Celexa) ○ Dantrolene, cooling
4. Paroxetine (Paxil)
SEROTONIN ANTAGONISTS
5. Fluvoxamine (Luvox, Faverin)
6. Escitalopram (Lexapro)
Seretonin Details Brands
7. Dexfenfluramine (Redux) antagonist
Notes:
Trazodone - 5HT2 antagonists (peri Desyrel
● Fluoxetine → norfluoxetine
(Desyrel) pheral)
● Fluoxetine and paroxetine are potent inhibitors of
- Serotonin antagonist
CYP2D6 Nefazodone and reuptake inhibitor
● Fluvoxamine is an inhibitor of CYP3A4 (Serzone) (SARI) (central)
ADVERSE EFFECTS: - Use: Antidepressant
1. Gastrointesti nal effects (nausea, gastrointestinal - A/E: Priapism,
upset, dia rrhea) Hepatotoxicity
2. Sexual effects - dec. loss of libido, delayed orgasm,
or diminished arousal Cyproheptadine - a potent HI-receptor Periactin
3. Increase in headaches and insomnia or (Periactin) antagonist of the
phenothiazine class; it
hypersomnia
blocks both 5HT1- and
4. Discontinuation syndrome (dizziness, paresthesias)
5HT2receptors.
5. Paroxetine is a category D agent - Use: diarrhea and
intestinal spasms
SEROTONIN SYNDROME (serotonin - secreting
● diagnosed on the basis of a history of administration carcinoid tumors and
of a serotonergic drug within recent weeks and postgastrectomy
physical findings dumping syndrome)
● It has some characteristics in common with
neurolepticmalignant syndrome (NMS) and malignant
Ketanserin - 5HT2-receptor Sufrexal
hyperthermia (MH)
(Sufrexal) antagonist
Serotonin Syndrome - it also antagonizes
alpha-adrenergic, H1-,
● Precipitating Drugs: and dopamine receptors.
○ SSRls, second-generation antidepressants, MAOIs, - Use: Antihypertensive
linezolid, tramadol, meperidine, fentanyl, ondansetron, (Europe),
Antihypertensive and for
sumatriptan, MDMA, LSD, St. John’s wort, ginseng
vasospastic cond (USA)
● Clinical Presentation:
○ Hypertension, hyperreflexia, tremor, clonus, Clozapine - 5HT2A- and 5HT2C - Clozaril, Risdin
hyperthermia, hyperactive bowel sounds, diarrhea, (Clozaril receptor antagonist.
mydriasis, agitation, coma: onset within hours - Clozapine - also block
● Therapy Risperidone D4 receptor
○ Sedation (benzodiazepines),paralysis, intubation, and (Risperdal, - Risperidone- also block
Risdin) D2 receptor
ventilation; consider 5-HT2 block with cyprohepta dine
or chlorpromazine Use: Antipsychotic agents

37
PCOL 211 - Pharmacology 1

Setrons - 5-HT3 antagonist Zofran Anti-emetics Details Brands


This includes: - Clinical use: mgt. of
- Ondansetron chemotherapy induced Anticholinergic Scopolamine (Hyoscine) =
(Zofran) nausea and vomiting Hyoscyamus niger or henbane Buscopan
- Granisetron
(Kytril) MOA:
- Palonosetron Antagonize receptor of
- Alosetron acetylcholine receptor thev reduce
the excitability of labvrinth in
muscarinic receptors and depress
the conduction from the vestibular
apparatus to the vomiting center

USE:
Treat motion sickness and in
● Vomiting - the expulsion of gastric contents preoperative operation
○ Causes: motion sickness, viral and bacterial A/E: Drowsiness, dry mouth, and
infection, food intolerance, surgery, PG, pain, blurred vision
shock, effects of some drugs, radiation, and
disturbances of the middle ear affecting equilibrium
● Antiemetics can mask the cause and should not be Antihistamines Diphenhydramine Bonamine
used until cause is determined, unless vomiting is • Meclizine
severe enough to cause dehydration and electrolyte • Cyclizine
imbalance • Dimenhydrinate
• Promethazine
EMESIS
MOA:
● Triggered by "vomiting center" at the medulla act by inhibiting cholinergic
● Four important sources of afferent input (neurons that carry pathways of the vestibular
sensory stimuli) to the vomiting center: apparatus by receptor crossover
1. Chemoreceptor trigger zone
USES:
2. Vestibular system (important in motion sickness)
motion sickness, true vertigo and
3. Irritation of the gastrointestinal mucosa by nausea in pregnancy
chemotherapy, radiation therapy, distention, or acute A/E: sedation and dry mouth
infectious gastroenteritis
4. Central nervous system
Dopamine Dopamine receptor (D2) blockade Plasil
antagonist - blocks the dopamine receptors in
the CTZ USE: Motilium
Metoclopramide
(Plasil) USE:
- Used to treat nausea due to
Domperidone chemotherapy (use of cisplatin
(Motilium) and doxorubicin) and narcotic
induced vomiting
(Metoclopramide)
- Motion sickness (Domperidone)

A/E:
• Sedation
• Diarrhea
• extrapyramidal effects
• elevated prolactin secretion

ANTIEMETICS

● They are useful in the treatment of vomiting associated


with motion sickness (OTC) and chemotherapy (Rx).

38
PCOL 211 - Pharmacology 1

SETRONS MOA: Zofran anti-emetics


This includes: 5HT3 Antagonists
Ondansetron Kytril A/E:
May cause adrenal
Ondansetron - more effective than
suppression and metabolic
(Zofran) metoclopramide against nausea disturbances
induced by high dose cisplatin.
Granisetron (Adv: No extrapyramidal effect) Benzodiazepene ● Lorazepam (Avitan) Valium
(Kytril) ● Diazepam (Valium)
Granisetron
Palonosetron - has a greater affinity for 5-HT3 MOA:
receptors than ondansetron GABA agonist
Alosetron - longer acting and more potent
USES:
than ondansetron or - Acts as anxiolytic agents to
metoclopramide reduce anticipatory emesis
- most common adverse effect is - Diazepam is useful as a
headache treatment for vertigo and it
controls symptoms of
Cannabinoids ● Dronabinol (Marinol) Marinol Meniere’s disease
(Synthetic THC ● Nabilone (synthetic derivative)
A/E:
Analog) Anterograde amnesia which
MOA: lasts for 4-6 hours
THC: May act by inhibiting the vomiting
psychoactive center Aprepitant MOA: Aprepitant
component of - Neurokinin receptor Capsules,
Marijuana USE: antagonist USP
Chemotherapy induced nausea - Central blocker of area
postrema
administered either as an oral
- It also inhibits substance P
preparation or smoked as a
cigarette (if other drug is ineffective USE:
only) Combination with 5HT3 blocker
and corticosteroids for acute
A/E: and delayed N and V
Sedation, psychoactive effects, dry associated with
chemotherapeutic agents
mouth and orthostatic hypotension
Enzyme inhibitor (CYP3A4)

EMETICS

● Agents that induce reflex vomiting

1. IPECAC

● Mixture of alkaloids including emetine and cephaeline


derived from the ipecacuanha plant.
○ Carapichea ipecacuanha
● MOA: Induces vomiting by stimulating the CTZ and by
causing GIT irritation and afferent input to the vomiting
center
● Administered orally
● Fast acting, causing vomiting in 85% of the patients within
20 minutes
● Useful in removing unabsorbed toxins from the stomach
● CONTRAINDICATIONS:
○ Coma
○ Absence of the gag reflex
○ Ingestion of caustic agents and petroleum distillates
Anti-emetics Details Brands
● Adverse Effects:
Glucocorticoids Example: Solu-Medrol
○ cardiac toxicity caused by emetine in ipecac is noted
● Dexamethasone and in abusers such as bulimics
Betamethasone ○ Persistent vomiting (eg, acetylcysteine)
● Methylprednisolone
○ Hemorrhagic gastritis or Mallory-Weiss tear
USE: ■ Mallory-Weiss tear - a tear of the tissue of your
Can be effective as a treatment lower esophagus. It is most often caused by
for vomiting caused by highly violent coughing or vomiting.
emetic agents. High doses are
given as an intravenous bolus
or orally for delayed nausea
often combined with other

39
PCOL 211 - Pharmacology 1

2. APOMORPHINE ● Descending colon - descends transverse colon to


sigmoid colon
● The only opioid derivative not classified as narcotic ● Sigmoid colon - hold feces until it is time to defecate
analgesic. ● Rectum and Anus
● MOA: ○ Rectum - place where residual waste accumulates
○ Dopamine agonist ○ Anus - port of exit
○ Directly stimulates the CTZ ● Cecum - absorbs digestive fluids passing out the ileum
of the small intestine
● Administered parenterally (SQ) ● Ascending colon - push up any undigested food
● More toxic than ipecac - it causes CNS and respiratory products from the cecum
depression., therefore it is not widely used.

DRUGS FOR MOTILITY DISORDERS

Physiology of Defecation

● It is controlled by neurological reflexes and sphincters;


thus, it ordinarily occurs on ly in appropriate circumstances
and times.
1. Filling of meal triggers gastrocolic reflex.
When you wake after sleep, stretching and sitting and
then standing - orthocolic reflex.
This reflexes results to muscle contraction in the
colon.
2. Muscle contraction also results to stretch of rectum
called defacation reflex - account to urge to defacate CONSTIPATION
3. Internal defacation reflex - controlled by Enteric
Nervous System (myenteric plexus) ● Accumulation of hard fecal material in Constipation the
Parasympathetic defecation reflex - controlled by large intestine
Parasymphatetic Nervous System (transmitted to the ● It is a major problem of among the elderly (reduce muscle
spinal cord) tone and weaker peristalsis)
● Pathophysiology:
○ Constipation commonly results from a diet low in fiber
or from use of constipating drugs such as opiates.
○ Constipation may sometimes be psychogenic in origin.

Causes

● Poor H20 intake & poor dietary habits


● Ignoring the urge
● Fecal impaction
● Bowel obstruction
● Chronic laxative use
● Neurologic disorders (paraplegia)
● Lack of exercise
● Selected drugs
● Neurogenic constipation
○ (Head trauma, CNS tumors, spinal cord injury, cerebrc
accidents, Parkinson's disease)
● Pregnancy
The Large Intestine ● Inappropriate bowel habits
● Aka Colon Drugs Causing Constipation
● 3 - 5 ft long
● Muscle contraction - cause motility, helping the stool ● Analgesics
move through and out the body ○ Inhibitors of prostaglandin synthesis
● Functions: ○ Opiates
○ Absorbs salt and water from waste ● Anticholinergics
○ Moving waste/feces ○ Antihistamines
○ Storing waste ○ Antiparkinsonian agents (e.g, benztropine or
○ Evacuating the waste through defecation trihexyphenidyl)
○ Phenothiazines
● Transverse colon - majority of water reabsorption take ○ Tricyclic antidepressants
place

40
PCOL 211 - Pharmacology 1

● Antacids containing calcium carbonate or aluminum 2. Nonspecific stimulants or irritants (with effects on fluid
hydroxide secretion and motility)
● Barium sulfate ● Dipheny|methanes (bisacodyl)
● Calcium channel blockers ● Anthraquinones (senna and cascara)
● Clonidine ● Castor oil
● Diuretics (nonpotassium-sparing) 3. Prokinetic agents (acting primarily on motility)
● Ganglionic blockers ● 5-HT, receptor agonists
● Iron preparations ● Opioid recepior antagonists
● Muscle blockers Otubocurarine, succinylcholine)
BULK-FORMING LAXATIVES
● Nonsteroidal antiinflammatory agents
● Polyswrene sodium sulfonate ● Insoluble and non-absorbable substances that expand on
taking up water in the bowel
Signs and Symptoms
● Consist of polysaccharides and cellulose derivatives that
1. Infrequent bowel movements are undigestible.
2. Stool of insufficient size ● Because they absorb water, they increase the bulk of stool
3. Hard, small dry stools and, in so doing, provide a physiological stimulus to
4. Feeling of full ness(bloated stomach) defecation:
5. Abdominal pain and discomfort
6. Difficulty and pain on passing stool
7. Fatigue and headache
8. Nausea and vomiting

Diagnosis

● Proctoscopy, sigmoidoscopy, colonoscopy


Examples and Uses
● Barium enema - necessary to determine presence of
colorectal pathology A. Dietary fiber (10-15g/day)
● Thyroid function test - determine presence of metabolic - Bran, linseed
disorder that may cause the constipation B. Hydrophilic colloids
- Psyllium (C-lium), karaya gum
NON-PHARMACOLOGIC TREATMENT
C. Synthetic fiber
● Diet that contains fiber (whole grains, vegetable such as - polycarbophil
broccoli, carrots and sweetcorn and fruits such as berries, D. Cellulose derivatives
pears, orange and pineapple) - methylcellulose (Citrucel)
● Water ● USE:
● Exercise ○ Management of chronic constipation.
● Routine bowel habits (normal can be 1-3/day or ○ Produce laxation after 2-4 days and adequate
3/wk-varies from person to person) The freq. is secondary hydration is required.
to consistency - feces hard & dry.
OSMOTIC LAXATIVES
● Stretching after waking up
● Eating breakfast (30-45 mins after waking up) ● Soluble but non-absorbable particles that retain water in
the bowel by virtue of their osmotic action and cause a
LAXATIVES
reflex increase in peristalsis.
● Promote and facilitate bowel evacuation by:
A. Salt Containing
○ Acting locally to stimulate intestinal peristalsis
○ To soften bowel contents or both ● Also known as saline purgatives
● Uses: ○ Magnesium sulfate - Epsom salt
1. Ease defecation in patients with painful hemorrhoids ○ Magnesium citrate - Citrate of Magnesia
or other rectal disorder. ○ Sodium phosphate - Fleet Enema
2. Avoid potentially hazardous rises in blood pressure ○ Sodium sulfate- Glauber's salt
during defecation in patients with hypertension, ○ Potassium sodium tartrate- Rochelle's salt
cerebral arterial diseases. ● Elicit a watery bowel discharge 1—3 hours after
3. Relieve acute constipation. administration
4. Remove solid material from the intestinal tract prior to
certain roentgenographic studies.

Classification of Laxatives

1. Luminally active agents


a. Hydrophilic colloids; bulk-forming agents (bran,
psyllium, etc.)
b. Osmotic agents (nonabsorbable inorganic salts or
sugars)
c. Stool-wetting agents (surfactants) and emollients
(docusate, mineral oil)

41
PCOL 211 - Pharmacology 1

Side effects:
● Magnesium containing- Hypermagnesemia
○ Caution: renally impaired patient
● Sodium containing- Hypernatremia
○ Caution: Patients with CVD
● Dehydration- adequate water intake is required

B. Salt-free osmotic agents

● These agents maybe administered rectally (glycerin) or


orally (lactulose)
○ Lactulose
○ Mannitol
○ Sorbitol
○ Polyethylene glycol - electrolyte solutions

Laxative Details

Mannitol ● Osmotic laxative in oral doses larger than 20 g


Osmotic laxative Details
● Mannitol I.V. (mannitol injection) is a diuretic
used to increase urine production, and to treat or
Magnesium sulfate The laxative action probably results from prevent medical conditions that are caused by an
Bitter salt; Epsom salt two factors: increase in body fluids/water (e.g., cerebral
1. SO4 ion - Not absorbed from the Gl edema, glaucoma, kidney failure)
tract and, thus, draws water into the
lumen of the bowel to make an isotonic Mannitol: Injection - 5 % ( 5mg/100ml),
solution and 10% (10mg/100ml), 15% (15 mg/100ml),
2. Magnesium ion - Release of 20% (20mg/100ml), 25% (12.5mg/50ml)
cholecystokinin-pancreozymin, which
causes an accumulation of fluid and Lactulose ● Cannot be hydrolyzed by digestive enzymes
electrolytes within the small intestine Cholac, Lilac ● Fermentation of lactulose by colon bacteria to
lactic acid leads to acidification of bowel contents
Oral - dissolve in iced water or orange and a reduced number of bacteria.
juice to mask nauseous taste ● Depends on its breakdown by colonic bacteria to
carbon dioxide, lactic acid, and small amounts of
IV - (125/500mg/ml) not as laxative, used acetic and formic acids, which acidify the
to manage pre-eclampsia contents of the colon.
● The acidic environment converts ammonia to
ammonium ion (NH4+), which cannot be
Sodium biphosphate - One of the most palatable of the saline
absorbed
(NaH PO4. H2O) laxatives
● Used in liver failure to forestall hepatic coma by
- Its major use, however, is for diagnostic
preventing bacterial production of ammonia and
procedures such as proctoscopy,
its subsequent absorption (absorbable NH3 + -4
colonoscopy, or barium enema
nonabsorbable NH4+).
- It also is used in the form of the oral
solution as an antihypercalcemic
Adverse Effect:
- Used also in cystitis (combined with
Flatus & cramps- gas generated in the stomach or
Methenamine)
bowels. due to fermentation in the gut producing
CO2
Rectal solution - also known as Fleet
Enema
Drug Interaction:
● Neomycin- eliminate colonic bacteria reducing
Oral solution - Fleet Phospho-Soda
the fermentation of lactulose to lactic acid thus
decreasing the acidification of colon
Sodium sulfate - Used also as laxative to evacuate bowel
Glauber's salt prior to surgery Contraindication:
- Used also electrolyte replenisher and is Used with caution to patients with diabetes (it
used in isosmotic solutions so that contains monosaccharide 1 molecule of galactose
administration does not disturb normal and 1 molecule of fructose
electrolyte balance and does not lead to
absorption or excretion of water and
ions STIMULANT LAXATIVES

● Also known as Irritant Laxatives


General Uses: A. DIPHENYLMETHANES
1. For acute evacuation of the bowel before surgery or diagnostic
● Phenolphthalein, bisacodyl
procedures.
2. For elimination of parasites after antihelmhintic administration.
● Sodium picosulfate
B. ANTHRAQUINONES
● aloe, cascara sagrada and senna
C. CASTOR OIL

42
PCOL 211 - Pharmacology 1

● Mechanism of action:
Melanosis coli is a condition usually associated with
○ Direct stimulation of the enteric nervous system and chronic laxative use in which dark pigment is deposited in
colonic electrolyte and fluid secretion. the lamina propria (one of the lining layers) of the large
○ Have multiple actions on the intestine: Decreases intestine (colon). The pigment deposition results in a
water absorption from the bowel lumen and stimulate characteristic dark brown to black discoloration of the
intestinal secretions. lining of the large intestine.
● USES:
○ Prevention of straining at stool following surgery, ANTHRAQUINONES
myocardial infarction, or stroke; and provision of relief
in painful diseases of the anus, e.g., fissure, Laxative Details
hemorrhoids.
Cascara sagrada ● From dried bark of Rhamnus
purshiana
● Nature's Remedy; Sacred Bark;
Chittem; Dogwood;Bearberry; Bitter
Bark
○ Widely used cathartic.
○ It is used for relief of transient
constipation.
○ It has very little action on the small
intestine but promotes peristalsis in
the large intestine

DIPHENYLMETHANES Sennosides Example: Calcium sennosides


(Senokot)
Limitation to use:
Reabsorbed via enterohepatic circulation and have: A natural complex of anthraquinone
1. Increased risk of abuse and overdose glycosides found in senna, isolated from
2. Cardiotoxic Cassia angustifolia as calcium salts,
contains 55% to 65% of the calcium
Bisacodyl (Dulcolax) Converted by gut bacteria into the salts.
active colon-irritant principle.
Given by the enteral route, bisacodyl Large Bowel Irritant Laxative
is subject to hydrolysis of acetyl ● Chronic use: Melanosis coli (Brown pigmentation of the
residues, absorption, conjugation in colon)
liver to glucuronic acid (or also to ● Onset of action occurs in 6-12 hours (oral) and
sulfate), and biliary secretion into the requires adequate hydration
duodenum.

Oral administration: 6 - 8 hours by


discharge of soft formed stool.

Suppository: Bisacodyl effect (one


hour)

Phenolphthalein It tends to act longer than other


laxatives because it enters the
enterohepatic circulation and is
re-excreted in bile into the intestine

In susceptible individuals,
phenolphthalein may cause allergic
reactions, including SJS and lupus CASTOR OIL
erythematosus. It also may cause a
Bartter's-like syndrome ● Obtained from the seed of Ricinus communis linne (Fam
Euphorbiaceae
The drug has been discontinued ● Brand: Emulsoil; Neoloid; Purge
● Small bowel irritant laxative
● Chronic intake: ● MOA:
○ Disrupts the water and electrolyte balance of the body ○ Hydrolyzed to ricinoleic acid (upper GIT)+ which
and can thus cause symptoms of illness (e. g., cardiac stimulates water secretion in the intestine while
arrhythmias secondary to hypokalemia). decreasing glucose absorption Ricin- toxic component
○ May also result in cathartic colon (melanosis coli) of castor bean

43
PCOL 211 - Pharmacology 1

● Oral administration: 10 - 30 ml of castor oil is followed 2. Bacteria and virus


within 0.5 - 3 hours by discharge of a watery stool 3. Drug reaction (antacids containing magnesium,
● Topical - emollient, stimulate hair growth antibiotics, cholinergic drugs)
4. Laxative abuse, malabsorption syndrome, stress,
STOOL SOFTENER
bowel tumor, inflammatory bowel disease
● Also known as Emollient Laxatives Clinical Presentation of Diarrhea
● Substances included in this category are surface-acting or
wetting agents, which are nonabsorbable and relatively ● General
nontoxic ○ Usually, acute diarrheal episodes subside within 72
● Have a detergent action that facilitates the mixing of hours of onset, whereas chronic disrrhea involves
watery and fatty substances to increase luminal mass frequent attacks over extended time periods
● MOA: Their action is attributed to their surface-active ● Signs and symptoms
property ○ Abrupt of nausea, vomiting, abdominal pain,
● Lowering surface renson → permit the intestinal fluids to headache, fever, chills, and malaise
penetrate the fecal mass more readily → produce soft, ○ Bowel movements are frequent and never bloody, and
easily passed stools diarrhea lasts 12 - 60 hours
○ Intermittent periumblical or lower right quadrant pain
Examples Details with cramps and audible sounds is characteristic of
small intestinal disease
Docusate ● Used to produce short-term laxation
○ When pain is present in large intestinal diarrhea, it is a
(Colace, Surfak) and to reduce straining at defecation
● They are used to prevent constipation gripping aching sensation with tenesmus (straining,
Glycerin ineffective and painful stooling). Pain localizes to the
hypogastric region, right or left lower quadrant, or
Mineral Oil ● Almost non-absorbable and makes sacral region.
feces softer and passed easier and ○ In chronic diarrhea, a history of previous bouts, weight
coats fecal contents and thereby loss, anorexia, and chronic weakness are important
inhibits absorption of water
findings
● USE: Prevent and treat fecal
impaction in children and debiliated ● Physical examination
adults ○ Typically demonstrates hyperperistalsis with
● Decreases the absorption of fat borborygmi and generalized or local tenderness
soluble vitamins (long-term use) and ● Laboratory tests
causes lipid pneumonitis can ○ Stool analysis studies include examination for
develop if aspirated microorganisms, blood, mucus, fat, osmolality, pH,
electrolyte and mineral concentration, and cultures
SEROTONIN AGONIST ○ Stool test kits are useful for detecting GI viruses,
particularly rotavirus
TEGASEROD ○ Antibody serologic testing shows rising titers over a 3
to 6 days period, but this test is not practical and is
● MOA:
nonspecific
○ Stimulation of 5HT4 also activates cAMP dependent
○ Occasionally, total daily stool volume is also
chloride secretion from the colon, leading to increased
determined
stool liquidity
○ Direct endoscopic visualization and biopsy of the
○ Promotes gastric emptying and enhances small and
colon may be undertaken to assess for the presence
large bowel transit
of conditions such as colitis or cancer.
● Taken before meals
○ Radiographic studies are helpful in neoplastic and
● NOT be given to patients with severe hepatic or renal
inflammatory conditions
impairment
● USES: Diarrhea
○ For chronic constipation
○ Irritable bowel syndrome with constipation ● Can cause minor or severe dehydration and electrolyte
imbalance
DIARRHEA ● Nonpharm treatment:
1. Clear liquids and oral solutions (gatorade,
● Excessive fluid weight, with 200 g/day representing the
pedialyte)
upper limit of normal stool water weight for healthy adults
● Intended as soom as intake of usual foods and
● Manifested by:
fluids is discontinued and before serious fluid
○ Frequent liquid stool discharge
looses or deficit occurs
○ Increase frequency and decreased consistency of
● Avoid non-clear liquids such as milk, juices, and
fecal discharge
sodas, because these can actually make the
○ Decrease transit time for gastric emptying
diarrhea worse
● It can be life threatening to the young and elderly
2. IV electrolyte solutions
● Can be mild to severe - Identify underlying causes first
● Ringers Injection USP: (NaCl, KCl, and CaCl2)
● Causes:
● Lactated Ringers Injection USP: (NaCl, KCl,
1. Foods
CaCl2 and Na lactate)

44
PCOL 211 - Pharmacology 1

ORAL REHYDRATION SALTS ○ Also, loperamide may prolong the course


of infectious diarrhea
● Balanced salt solutions containing glucose, sucrose or ● Examples:
1. Atropine
rice powder
2. Scopolamine
● Increase water absorption from the bowel lumen by 3. Methantheline
increasing sodium substrate transport across intestinal 4. Propantheline
epithelial cells ● Agents that inhibit colonic peristalsis by
blocking the receptors of intestinal
● Can remedy 99% of acute cases of childhood diarrhea smooth muscle to cholinergic timulation
● Antispasmodics - used to reduce colonic
cramping (abdominal cramps)
● They have little effect on diarrhea

Adsorbents - Kaolin and pectin and dietary fiber


- Act by adsorbing toxic compounds from
intestinal water
- They are non-toxic but they are less
effective than other agents

KAOLIN
● Light Kaolin; White Bole; Kaolin-Pectin
Suspension
● A native hydrated aluminum silicate

PECTIN
● A purified carbohydrate product obtained
from the dilute acid extract of the inner
ANTIDIARRHEAL portion of the rind of citrus fruits or from
apple pomace
A. Non Specific Antidiarrheals ● Metals, particularly the heavy metals, form
insoluble derivatives
● Decrease fecal water content by increasing solute
Bismuth ● Binds to toxins produced by Vibrio cholerae
absorption and decreasing intestinal secretion
subsalicylate and E. coli
● Decrease motility and increased transit time facilitates ● Can be absorbed across the intestine
water reabsorption ● Inhibits the production of prostaglandins in
the intestine and reduces secretion
Opiates and 1. Opium tinctures
Opioids 2. Camphorated opium tincture Use: Effective for both treatment and
3. Codeine prophylaxis of traveler’s diarrhea

Acts directly on opiate μ receptors Adverse effect: Tinnitus


- Decrease transit rate
- Stimulate segmental contraction
They also stimulate electrolyte absorption B. Antidiarrheals for Specific Cases

A/E:
● Nausea, sedation, vomiting Octreotide ● A synthetic 8-amino acid analog of
● High doses may control refractory diarrhea somatostatin
resulting in a potential for dependence ● Used in cases of severe diarrhea caused
by excessive release of gastrointestinal
tract hormones including gastrin, motilin,
Diphenoxylate ● A synthetic analog of morphine
vasoactive intestinal polypeptide, glucagons
● Combined with atropine to reduce the
and others
potential for abuse and to further reduce
motility (Lomotil)
Tegaserod Serotonin (5HT4) Agonist
USE: Short term treatment of IBS with
Opiates and Opioids
constipation
● Caution should be used in patients with ulcerative colitis and
pseudomembranous colitis who are at increased risk of
developing toxic megacolon Mesalamine & ● Acts within the colon to limit prostaglandin
● Also, it may prolong infectious diarrhea Olsalazine and leukotriene production

S/E: USE:
● Nausea, sedation, vertigo, vomiting, pruritus, skin eruption, ● For the treatment of mild to moderate
insomnia, and abdominal cramps ulcerative colitis
● Numbness of the extremities, headache, blurring of vision, ● Olsalazine is approved for maintenance of
swelling of gums, and general malaise also have been reported remission but not treatment of this disorder

Loperamide ● MOA: BInds to the opiate receptor in the gut Glucocorticoid Prednisone
(Lomotil, wall, acts by slowing intestinal motility and ● Stimulates sodium absorption in the
Imodium, by affecting water and electrolyte movement jejunum, ileum and colon
Diatabs) through the bower ● Used to treat refractory diarrhea →
● Essentially free of CNS effects unresponsive to other agents and chronic
● Caution should be used in patients with: inflammatory bowel disease (Crohn’s
○ Ulcerative colitis disease)
○ Crohn’s colitis
○ Pseudomembranous colitis who are at
increased risk for toxic megacolon

45
PCOL 211 - Pharmacology 1

IRRITABLE BOWEL SYNDROME (IBS) Biosynthesis of Eicosanoids

● Idiopathic (unknown cause), chronic, realpsing ● Arachidonic acid, (5,8,11,14-eicosatetraenoic acid)


disorder (reoccurring) characterized by: ● the most common precursor of the eicosanoids, is formed
○ Abdominal discomfort (pain, bloating, distention, or by two pathways
cramps) ○ Phospholipase A2 - mediated production from
○ Alterations in bowel habit (diarrhea, constipation or membrane phospholipids
both) ○ Phospholipase C in concert with diglyceride lipase
● Types: can also produce free arachidonate
1. Crohn’s disease ● Following irritation or injury Arachidonic acid is released
● Inflammatory bowel disease that causes chronic from cell membranes.
inflammation of the gastrointestinal tract ● Pathways of Arachidonic Acid
● Most commonly affects the colon and the last part ○ Prostaglandin H synthase (COX) pathway
of the small intestine (ileum) ○ Lipoxygenase pathway (LOX)
2. Ulcerative colitis ● Various eicosanoids are synthesized throughout
● An inflammatory bowel disease (IBD) that causes ● the body. Synthesis can be very tissue-specific.
inflammation and ulcers (sores) in your digestive
tract
● Affects the innermost lining of your large intestine
(colon) and rectum. Symptoms usually develop
over time, rather than suddenly.
3. Pseudomembranous colitis
● Swelling or inflammation of the large intestine
(colon) due to an overgrowth of Clostridium
difficile (C. difficile) bacteria
● Clindamycin - antibiotic that causes the
overgrowth
● Vancomycin and Metronidazole - used in the
management

Treatment for Irritable Bowel Syndrome (IBS)

1. Antispasmodics - relieve abdominal pain


2. Diarrhea - antidiarrheal agents Prostaglandin - H - synthase
● Mesalamine & Olsalazine - ulcerative colitis
● Glucocorticoids - Chron’s disease ● This pathway make use of COX
3. Constipation ● This procuses:
● Fibers, milk of magnesia ○ Thromboxane
● Tegaserod - Serotonin (5HT4) Agonist ○ Primary prostaglandins (PGE, PGF, PGD, PGI2)
● SSRIs - Selective Serotonin Reuptake Inhibitors COX-1 enzyme COX-2 enzyme
4. Chronic abdominal pain
● TCA (amitriptyline, desipramine 10-50 mg/d) Distribution This is located in Found in great
● Doses typically begin low and can gradually increase endothelial cells, kidney, abundance in
gastrointestinal tract, and connective tissues.
if needed. These low doses are usually not at a
many other
therapeutic dose to manage depression or anxiety locations

Expression Expressed at fairly It is highly inducible by


PHARMACOLOGY OF EICOSANOIDS constant levels
PGH synthase-1 (COX-1)
numerous factors
associated with
is expressed inflammation.
constitutively in most
Eicosanoids cells.

● Eicosanoids are a large group of autacoids with potent Function Generates prostanoids Current theories
effects on virtually every tissue in the body; for “housekeeping” such suggest that the type 2
as gastric epithelial isoforms is
● These agents are derived from metabolism of 20-carbon, cytoprotection predominantly
unsaturated fatty acids (eicosanoic associated with
● acids).
● The eicosanoids include: ● The eicosanoids all have short plasma half-lives (typically
○ Prostaglandins 0.5-5 minutes). Most catabolism occurs in the lung.
○ Thromboxanes ○ Prostaglandins are metabolized by PDGH to 15-keto
○ Leukotrienes metabolites.
○ Hydroperoxyeicosatetraenoic acids (HPETEs) ○ Thromboxane A2 (TXA2) is rapidly hydrated to the
○ Hydroxyeicosatetraenoic acids (HETEs). less active TXB2.
○ PGI2 is hydrolyzed to 6-keto-PGF.

46
PCOL 211 - Pharmacology 1

Immune PGE2 & PGI2 ncrease blood flow and


cells promote but do not cause,
edema.

Leukotrienes Chemotaxis of eosinophils


PGE2 & TXA2 and neutrophils • Leukocytes
(WBC)- Produce
Leukotrienes- Facilitate T-cell
proliferation and act as
chemoattractants. •
Macrophage- Synthesize
PGE2 and TXA2 Causes
inflammation in rheumatoid
arthritis

Gastro-intes PGF2 increase the rate of


tinal tract longitudinal contraction in the
gut and decrease transit time.

PGE2 Cytoprotectant-inhibit acid


and pepsinogen secretion in
Lipoxygenase Pathway the stomach

● This produces the HPETEs, HETEs, and the leukotrienes. Blood PGE2 & PGI2 Inhibit platelet aggregation
● Additional metabolites of the HPETEs, hepoxillinsm and
lipoxins, have been identified, but their biological roles is TXA2 Induce platelet aggregation
unclear
Histamine 5-HPETE Stimulate release
● This pathway is of great interest since it is associated with Release
asthma, anaphylactic shock, and cardiovascular disease PGD2 & PGI2 inhibits release
● LTC 4 and LTD 4 > primary components of the
slow-reacting substance of anaphylaxis (SRS-A) CNS (Fever) PGE2 Increases body temperature

CNS (Sleep) PGD2 Induces natural sleep

PGE2 Wakefulness

Eyes PGE and PGF Derivatives can lower


intraocular pressure

Synthetic Analogs

Synthetic Details Brands


Analogs

Carboprost PGF2 analogue Used to


(Hemabate) induce labour at second
trimester Therapeutic
abortion- Infusion of
carboprost tromethamine
Pharmacologic Action of Eicosanoids combined with
mifepristone (RU486) to
induce abortion.
SITE OF EICOSANOID RESPONSE
ACTION
Epoprostenol PGI2 analogues:
(Flolan) Epoprostenol (Prototype)
Vascular PGE2 & PGI2 Vasodilation Iloprost (Ventavis)
smooth Treprostinil (Remodulin)
muscle PGF Arteriolar vasodilation and Management of
superficial vein constriction pulmonary HTN

Thromboxane Vasoconstriction Alprostadil PGE1 analogue


(Caverject) Management of erectile
Bronchial PGF2 Bronchoconstriction dysfunction
Smooth Intracavernosal injection
Muscle PGE series Bronchodilation (base of the penis) or
urethral suppository
therapy with alprostadil
LTC4 & LTD4 Bronchoconstriction (SRS-A)
Also used in PDA (Patent
Ductus Arteriosus)
Thromboxane Bronchoconstriction

Uterine PGE2 & PGF2 Increase blood flow and


Smooth promote but do not cause,
Muscle edema.

47
PCOL 211 - Pharmacology 1

Misoprostol PGE1 analogue Used as


(Cytotec) cytoprotectant approved AUTONOMIC NERVOUS SYSTEM
for use in patients taking
high doses of
nonsteroidal
anti-inflammatory drugs AUTONOMIC NERVOUS SYSTEM
(NSAIDs) to reduce
gastric ulceration. A/E: ● The part of our body that controls our involuntary
Uterine contraction movements
(Abortifacient) ○ Kung bakit ka pumipikit, kung bakit tumitibok yung
puso mong mag-isa, bakit humihinga ka mag-isa, or
Dinoprostone PGE2 analogue Used for
(Prepidil, abortion and cervical
bakit nag-iisip o gumagalaw yung mga parts ng
Cervidil) ripening combined with katawan mo even without controlling it
mifepristone (RU486) to
induce first-trimester Neurotransmitters are chemicals that send signals and
abortion. information from brain, spinal cord, and to the site of action
or organs.
Latanoprost PGF2α analogue Used in
(Xalatan) the management of
glaucoma A/E:
NERVOUS SYSTEM
irreversible brown
pigmentation of the iris
and eyelashes

Eicosanoids Synthesis Inhibitors

● Most common Adverse Effect:


● NSAIDs (Non-steroidal Anti-inflammatory Drugs)

● COX-2-Selective Inhibitors “Coxibs”

a. Central Nervous System (CNS)


● The one that controls our whole body
1. BRAIN - receives and processes sensory information,
initiates responses, stores memories, generates thoughts
and emotions
○ Central processing unit of our body
○ Lahat ng may kinalaman sa katawan mo is
kino-control ng brain. That’s why kapag nagkakaroon
ng “brain-dead” person, parang ina-assume na rin na
patay na siya kasi hindi na nag fufunction yung brain.
2. SPINAL CORD - conducts signals to and from the brain,
controls reflex activities
Dietary Manipulation of Arachidonic acid ○ The one that receives signal from the brain and
distribute it to the different parts of the body
● Changing the phospholipid composition of cell membranes
○ That is why kapag nagkakaroon ng damage sa spinal
by replacing arachidonic acid with the dietary fatty acids:
cord, na-paparalyze, hindi na makagalaw yung mga
○ Linoleic acid: corn, safflower, and sunflower oils
extremities (kamay at paa)
○ Eicosapentaenoic (C20:5) and docosahexaenoic acids
(C22:6) b. Peripheral Nervous System (CNS)
■ called omega-3 fatty acids
1. MOTOR NEURONS - CNS to muscles and glands (from
■ from cold-water fish.
brain to extremities).
● Dietary uses:
○ Somatic Nervous System - controls voluntary
1. Reduction in the incidence of myocardial infarction
movements
and sudden cardiac death
a. Sympathetic Division - “Fight or Flight”. The
2. Protect against experimentally induced
brain’s response sending signals to the spinal
arrhythmogenesis, platelet aggregation, vasomotor
cord, muscle in cases of emergency
spasm, and dyslipidemias.
b. Parasympathetic Division - “Rest or Digest”.
Control sleep/wake mechanism (circadian
rhythm) and metabolism

48
PCOL 211 - Pharmacology 1

c. Enteric Nervous System - control the GIT and ○ Yung message mula sa extremities mo, pupunta sa
peristalsis movement. Can be affected by CNS, and the brain will make response
sympathetic and parasympathetic
● If you are in a emergency situation, the
sympathetic nervous system will send signal
to enteric nervous system to stop the
movement, or the peristalsis.
● For example, meron kang nakain na
masama, or it contains bacteria or virus,
siyempre our body aims to expel it out. The
parasympathetic nervous system will help to
fasten up peristalsis para maremove agad
yung bad contents or microbes na nasa loob
ng tiyan.
○ Autonomic Nervous System - controls involuntary
movement ● If you touch something hot, that will be the stimulus
2. SENSORY NEURONS - sensory organs to CNS (from ● There are sensory nerves in the skin (skin is the largest
organ of the body)
extremities to brain). Send signals from the organs to the ○ Kaya masakit tusukin yung dulo ng daliri, kaya
brain. yung skin ma-touch lang or kahit hindi ka pa
hinahawakan ng isang tao is nakakaramdam ka na
DIVISIONS OF NERVOUS SYSTEM
ng feeling.
● Central Nervous System - Brain and Spinal cord ○ All over our body, from head to toe, maraming
○ Functions: to process, integrate, store and respond to neurons at nerves diyan. Isa sa mga sensitive
information from the PNS. nerves ay yung nasa fingers.
● Kapag may na-touch ka na stimulus na hot, it will
○ The one that makes your memory, controls your whole stimulate the receptor, and that receptor will be
body, and receives messages from the peripheries activated sending a message via afferent neuron to your
● Peripheral Nervous System - Spinal nerves, Cranial brain. The brain will process kung ano ba yung
nerves & Ganglia nangyari, ano ba yung nahawaka (mainit or malamig),
○ Functions: transmit information to and receive and what should I do with that.
information from the CNS
Our brain always protects our body from danger (physical,
○ Spinal nerves - send or receive signals/messagaes
mental, psychological)
○ Ganglia - part of the nervous system that receives
messages. It act like a terminal that distribute ● Efferent Division - neurons carry signals away from the
messages from the brain going to the peripheries brain and spinal cord to the peripheral tissues (exit)
(doon maghihintay muna or iipunin yung messages, ● Somatic Nervous System - innervates mostly under
then it will be distributed to the site of action). voluntary control
■ For example, in cases of emergency, sabay ○ Under musculo skeletal division
sabay isesend ang signal. Si ganglia ang ● Autonomic Nervous System - innervates cardiac
magdi-distribute kung saang station siya pupunta. muscles, smooth muscles, vasculature and the exocrine
PNS - 12 Cranial Nerves glands that functions involuntarily

Oh! Oh! Oh! To Touch And Feel A Girl’s Velvet So Heaven! PNS - 1. Sympathetic Nervous System
1. Olfactory - sense of smell (nose) ● “Fight or Flight” response
2. Optic - send signals in eyes (recognizing things) ● It has the property of adjusting in response to stressful
3. Oculomotor - all eye muscle movement situations such as trauma, fear, hypoglycemia, cold or
4. Trochlear - superior oblique muscle exercise (emergency cases)
5. Trigeminal - near ear area (affects sinuses, face, teeth, ○ Hypoglycemia - low blood sugar (within 15-30
muscle mastication) (when damaged: tinnitus, dizziness) minutes, pwedeng mamatay ang pasyente because
6. Abducens - lateral rectus muscle wala ng glucose that gives energy to your brain)
7. Facial - affects face muscle ● For emergency cases
8. Vestibulocochlear/Auditory - controls inner ear ● NEUROTRANSMITTERS:
9. Glossopharyngeal - affects pharyngeal musculature of ○ Norepinephrine or noradrenaline
the pharynx, tongue, tonsils (mga nagagalaw ng paglunok) ○ Epinephrine or adrenaline
10. Vagus - affects movement of major organs: heart, lungs, ■ Adrenaline rush
bronchi, GIT, trachea, larynx, pharynx, external ear
11. Spinal Accessory - sternocleidomastoid and trapezius
muscle (affects the muscle of the neck)
12. Hypoglossal - affects the muscle of the tongue

DIVISIONS OF NERVOUS SYSTEM - continuation


● Afferent Division - neurons bring information from
periphery to the CNS

49
PCOL 211 - Pharmacology 1

Sympathetic Nervous System Effect on Organs ○ To control body tone


○ Cools down the body because of muscle contraction
● Bronhi: Dilation (bronchodilation) para may enough na
space para mag inhale at exhale
● Saliva: Little, viscous (natutuyo yung laway, to reserve
yung natitirang fluid sa katawan)

STOP, STOP, STOP production of fluids.


DRY, DRY, DRY

PNS - 2. Parasympathetic Nervous System


● “Rest and Digest” response
○ Kapag natutulog ka, doon nagii-start na mag-digest
ka. Kaya kapag morning, na-dedefacate na. During
sleep, doon nag cocomplete digestion where your
body converts it na to glucose, to fat, na irereserve ng
katawan and the rest will be eliminated.
● It maintains essential bodily functions (digestive processes
and elimination of wastes) during the sleep/wake
mechanism or circadian rhythm, and is required for life
● For normal base (everyday use: tulog, kain, digestion)
● NEUROTRANSMITTER:
○ Acetylcholine
■ Cholinergic action
Parasympathetic Nervous System Effect on Organs
● Once our sympathetic nervous system is activated by
our own experience or by a drug product, it will release
norepinephrine and epinephrine to our body to cause an
action.
● If the neurotransmitters are released in brain, it
increases the activity of the brain and alertness
● In eyes, pupillary dilation or mydriasis (Para kitang kita
mo or na-aaccommodate mo lahat ng bagay sa paligid)
● Heart: It increases the heart rate, the force of the heart,
the blood pressure. The heart needs to send enough
oxygen to the different parts (organs, cells) para may
energy sila, para makagalaw sila nang maayos.
○ Aside from supplying blood, the heart also supplies
oxygen and nutrition
● In fat tissue, lipolysis (fat breakdown). Nagkakaroon
ng lipolysis so that you will have enough energy.
○ Fatty acids liberation - kapag nagli-lipolysis or
metabolism, may mga fatty acids na naliliberate
which will be used for making energy.
● Urinary bladder: Sphincter (opening) - the sphincter
tone (movement) increases (constriction)
○ If tone is high - constriction
○ If tone is low - dilation ● Eyes: Accommodation for near vision or miosis
○ Sphincter act as reservation. Nirereserve nila yung (pupillary constriction)
natitira mong tubig or food sa katawan. (If ● Saliva: Copious, liquid (malabnaw)
ma-isolate ka sa isang island, hindi ka iihi nang iihi ○ Need ng malabnaw na saliva kapag nag
kasi kulang na yung tubig mo, so your body will mamasticate ka, kapag kumakain ka
retain yung natitirang fluids). ● Bronchi: Constriction, increase secretion
○ Detrusor muscle - the muscle of the wall ○ Kasi bumabagal na yung paghinga kapag
■ The detrusor muscle tone decreases (dilation) matutulog ka na
para enough yung space para i-retain yung ● Heart: Decrease heart rate, decrease blood pressure
water na natitira sa urinary bladder. ● GI Tract: Increase secretion, increase peristalsis, low
● Skeletal muscle: The blood flow increases because sphincter tone (dilate)
you need enough oxygenation and muscle movement ● Bladder: Low sphincter tone (dilate), increase detrusor
para makatakbo ka in a emergency situation. (constricted)
○ Glycogenolysis (breakdown of stored sugar) -
increases LIQUID, LIQUID, LIQUID
● GIT tract: Decrease peristalsis, increase sphincter tone,
decrease blood flow
● Liver: Glycogenolysis, glucose release
● Skin: Perspiration (sweating) - it is a sign of cholinergic
property pero kailangan mag-perspire kasi
nagkakaroon ng muscle movement

50
PCOL 211 - Pharmacology 1

SYNAPTIC NEUROTRANSMISSION

Structure of a Typical Neuron

PNS - 3. Enteric Nervous System


● Organized collection of neurons in the walls of the GIT.
Innervated by the myenteric plexus (auerbach plexus)
and the submucous plexus (meissner plexus)
○ These two causes the peristalsis movement
(movement of GIT) ● Nerves (neurons) - parts of our body that receive
● Affected by sympathetic or parasympathetic nervous electrical impulses, that receives messages, that acts
with the help of the chemicals, neurotransmitters so that
system
they can distribute the messages
○ Para - enhances neurotransmitter production, ○ They receive endogenous substances, like
enhances peristalsis which can lead to defacation, norepinephrine, epinephrine, dopamine, serotonin,
diarrhea and electrical impulse na pinapadala dahil sa mga
○ Sympa - stop neurotransmitter production, stop chemicals na yon.
peristalsis, stops defacation ● Neurons to neurons sending of signals, kaya ka
● NEUROTRANSMITTERS: (VANS) nagkakaroon ng reaction.
● Kaya nakakabuo ng memory, kaya nakakapag isip is
○ Substance P - promotes mucus production, but as
because of neurons.
well enhances pain perception ● As you get older, when your reach 18, 20, and above,
○ Neuropeptide Y - send signals dito na mag start na mag deteriorate yung number of
○ Vasoactive Intestinal Peptide - promotes movement neurons or nasisira because of stress, pollution,
○ ATP radiation, many other environmental factors (external
factors), psychological, mental, and physical stress.
Innervation of the Small Intestine by the Central and ○ That is why it is important to take vitamin B
Enteric Nervous Systems complex, vitamin C, and exercise to avoid getting
● (The movement of the small intestine by the effect of the neuronal problems that may lead to dementia and
central and enteric nervous system) Alzheimer’s,
● The central nervous system is the one that processes ● Dendrites - receive the chemical messengers, and
messages that send a response to our organs. electrical impulses and distribute them to the nucleus.
○ Once it receives a message like in case of ● Nucleus - processes the message
emergency, i-pprocess niya yon, “ano yung dapat ● Cell body - protects the nucleus
kong gawin” ● Axon - way, passage, or pathway of message.
● CNS will send signal to enteric nervous system: “Stop Protected by myelin sheath, Schwann’s cells and node
na muna ang movement mo, stop the peristalsis” para of Ranvier
mareserve yung food, water, para at ease ang ● Myelin sheath - protects axon. Isa sa mga madaling
movement masira kapag exposed ka palagi sa stress, at pollution.
● The CNS innervation of the small intestine, it will Kapag nasisira yan, nasisira rin ang axon kaya
stimulate the vagus nerve (cranial nerve 10), and then nagpuputol-putol yung memory nung iba kasi nakastay
postganglionic parasympathetic neuron can also be lang sa nucleus, hindi marereceive ng axon terminal. I
affected which leads na kapag kailangan mo na ○ If the axon terminal doesn’t receive the message,
mag-defacate. hindi niya ma-isesend sa panibagong neuron para
○ You can control your defecation using your brain, madeliver yung message sa ibang part.
you can also stop defecation using your brain.
○ It can stimulate either para or sympa to stop the
movement of the GIT tract
● The enteric nervous system is the one that sends signal
(normal peristalsis)
○ Can be affected by the para or sympa, and the
brain

51
PCOL 211 - Pharmacology 1

● The norepinephrine and epinephrine are released by an


adrenal gland located on the top of the kidney
● Norepinephrine primary neurotransmitter of the
postganglionic sympathetic adrenergic nervous system
● Synthesized inside the nerve axon, in the adrenal
medulla
○ Norepinephrine and epinephrine are released in
adrenal medulla
○ Cortex releases steroids
● It is released by the nerve when the action potential
travels down the nerve
● Parts of Synapse:
1. PRE SYNAPSE Biosynthesis of Catecholamines
● Site of the synthesis, storage, and release of
neurotransmitters happen. (dulo, papunta sa ● (or the process of neurotransmission)
axon terminal) 1. Biosynthesis
● Receptors: Pre-synaptic receptors 2. Storage
2. SYNAPTIC CLEFT 3. Release
● Space between the two neurons/synapses 4. Binding to receptor (alpha, beta, and delta)
● Para may enough na area para ma-distribute 5. Metabolism (30% inactivation)
yung mga neuro transmitter ● MAO - monoamine oxidase
3. POST SYNAPSE ● COMT - catechol-o-methyltransferase
● Dendrite area ng sumunod na neuron 6. Reuptake (70% of inactivation)
● Majority of the receptors are found in this
location. This also contains metabolizing
enzymes

Norepinephrine
● The neurotransmitter that sends the exact message that
you need
● Under the sympathetic nervous system
● Aka noradrenaline
● Responsible for fight or flight response
● The primary neurotransmitter of postganglionic
sympathetic adrenergic nerves
○ Synthesized inside nerve axon in the adrenal
medulla and stores within vesicles
○ It is released by the nerve when an action potential
travels down the nerve
● Neurotransmission happens when the neurotransmitter
is synthesized from its precursor
1. Norepinephrine is made from its precursor amino
acid tyrosine (synthesized from a tyrosine)
2. It will be stored inside the pre-synapse
3. It will be released in the synaptic cleft
4. Will bind to its receptor (norepinephrine and
epinephrine can bind to alpha, beta, and delta)
5. The rest will be metabolized (by MAO or COMT)

52
PCOL 211 - Pharmacology 1

6. 70% of the neurotransmitter will reuptake ● Cocaine and tricyclic antidepressant - inhibits the
(bumabalik sa pre synapse) sixth step (reuptake)
● With the help of tyrosine hydroxylase enzyme, tyrosine ○ Inhibits the NET (norepinephrine transporter)
○ Leads to the increase of norepinephrine
will be converted to DOPA
● With the effect of DOPA decarboxylase enzyme, it will
be then converted to dopamine Acetylcholine
● Because of the dopamine beta-hydroxylase enzyme, it ● The primary neurotransmitter of parasympathetic
will be converted to noradrenaline or norepinephrine nervous system.
● With the effect of phenylethanolamine and
N-methyltransferase it will be converted to adrenaline
or epinephrine

Metabolism of Catecholamines

● The release of this neurotransmitters depends on the


● Kapag naging VMA na yung norepinephrine or body’s needs (kung ano yung kailangan mo, yun yung
epinephrine, wala na siyang effect, hindi na siya nare-release sa loob ng neurons mo)
makakapag bind kay alpha, beta, or delta ○ Acetylcholine - matutulog, kakain
○ Norepi/epi - fight or flight, nasa emergency cases
● Acetylcholine is synthesized from acetate and choline
○ Because of acetylcholinesterase enzyme
○ AcCoA and choline - precursor of acetylcholine
● It will be stored, and then will be released to synaptic
cleft and will bind to its receptor
● Cholinoceptors - collective term for receptors under
the parasympathetic nervous system
○ Nicotinic and muscarinic receptor - individual
● It is metabolized by acetylcholinesterase enzyme in the
post synapse or synaptic cleft (will be converted to
choline and acetate na walang effect)
● Hemicholiniums - inhibits synthesis
● Vesamicol - inhibits storage
● Botunilum toxin - inhibits release

● Inside the axon, or the pre synapse, the synthesis,


storage, and release happen.
● From the amino acid tyrosine, with the effect of sodium,
activating the sodium channel (will open) so that the
tyrosine can go inside the axon
● Adrenoceptors - collective term for receptors under the
sympathetic nervous system (alpha, beta, and delta)
● Metyrosine - stops the action of norepinephrine
○ It inhibits the first step (synthesis)
● Reserpine - inhibits the second step (storage)
○ Walang magiging norepinephrine, walang
sympathetic action
● Bretylium and guanethidine - inhibits the release of
norepinephrine and epinephrine by preventing the
● Hemicholinium - inhibits the choline transporter
action movement of the calcium channel
(synthesis)
○ Kapag walang calcium, hindi naa-activate ang
● Vesamicol - inhibits the vesicular acetylcholine
VAMP’s and SNAP’s. If it is not activated, the
transporter (storage)
norepinephrine ATP release will be prevented
● Arecoline - is a muscarinic agonist (activates)

53
PCOL 211 - Pharmacology 1

○ Increases the effect of acetylcholine Postganglionic neuron: Postganglionic neuron:


● Atropine and benztropine - are muscarinic antagonist Soma is usually in a Soma is in a sympathetic
○ They block muscarinic receptor and leads to ganglion near the target ganglion, located next to the
decrease of action of acetylcholine organ spinal cord
○ They are drugs under sympathetic
● Bolutinus and tetanus toxins - inhibit ACh release Neurotransmitters released Neurotransmitters released
(which leads to paralysis), whereas black widow spider from postganglionic from postganglionic
toxin stimulates release (nagkakaroon ng seizure) synapse: Acetylcholine or synapse: Norepinephrine
○ Botolinium toxin (Botox) - to relax muscle (para nitric oxide
banat ang face)
● Tacrine, galantamine, and physostigmine - inhibit Target organ: Target organ:
AChE (acetylcholinesterase enzyme), inhibits “Rest and digest” response “Fight or flight” response is
metabolism, leads to the increase of acetylcholine levels is activated activated
○ Effects: naglalaway, nagiihi, at nagtatae
● Curare and mecamylamine are nicotinic antagonists
○ Inhibits the binding of acetylcholine to the nicotinic,
which leads to paralysis
● Varenicline is a partial agonist at nicotinic receptors
○ Smoking deterrent
○ It still gives the kick of the nicotine in the body that
cigarette smoker’s want
○ Partial agonist - have agonistic and antagonistic
effect (balance) hindi fully inaactivate ang receptor.
Still give a hint of the action of the neurotransmitter
to lessen the addiction

COMPARISON OF SNS AND PNS

Criteria SNS PNS

ANATOMIC

1. Roots Thoracolumbar Craniosacral


- Thoracic spine - Sacral (S1-S5)
(T1-T12) - Cranial (10, 9,
- Lumbar 7)
(L1-L5)

2. Location of Near the spinal Near the target Criteria SNS PNS
ganglia cord organ (large and
small intestine) NEUROTRANSMITTERS

3. Length of fibers 1. Pre-ganglionic Acetylcholine Acetylcholine


Pre-ganglionic Short (sending) Long (sending)
Post-ganglionic Long (action) Short (action) 2. Post ganglionic NE & Epi Acetylcholine

RECEPTORS EFFECTS ON SPECIFIC ORGANS

1. Ganglionic Nicotinic Nicotinic Heart Tachycardia Bradycardia


(increase (decrease
2. End organ α, β and D - Nicotinic n contractility) contractility)
(neurons, brain) Tachy - fast Brady - slow
- Nicotinic m
(muscles) Pupils Mydriasis (pupillary Miosis (pupillary
- Muscarinic dilation) constriction)
receptor
Bronchi Dilation (needs Constriction
enough O2) (reduces O2)
Parasympathetic Sympathetic
Bladder wall Relaxation (low tone) Contraction
Preganglionic neuron: Preganglionic neuron: (high tone)
Soma is usually in the Soma is usually in the spine
brain-stem or sacral (toward Bladder Contraction (high Relaxation (low
the bottom) spinal cord spincthers tone) tone)

Neurotransmitter released from the preganglionic synapse: EFFECTS ON SPECIFIC ORGANS


Acetylcholine
Intestinal wall Relaxation (low tone) Contraction
(high tone)

54
PCOL 211 - Pharmacology 1

○ Betanechol
Intestinal Contraction (high Relaxation (low
Spincther tone) tone) ● Alkaloids
○ Pilocarpine
Apocrine Hyperhydrosis Anhydrosis ○ Muscarine - Amanita muscaria (mushroom)
(Sweat glands) (needed to maintain ■ Kapag kinain, malalason
temperature, to cool ■ Its activity is similar to cholinergic or
down the body) parasympathetic activity where, magkakaroon ng
emesis or vomiting, diarrhea, uruination, miosis,
Ecrine Anhydrosis Hyperhydrosis
and na-paparalyze yung iba
○ Nicotine
Uterus Relaxation (low tone) Contraction
(high tone) ○ Varenecline
○ Arecholine

Indirect Acting
● Reversible - the action can be changed, you just need to
PARASYMPATHETIC NERVOUS SYSTEM: add or reduce the amount of substance
○ Quaternary alcohol
Cholinergic Agonist & Antagonists ■ Edrophonium - used as diagnostic agent
○ Carbamates
■ Neostigmine
Parasympathetic Nervous System
■ Physostigmine
● Also known as Cholinergic System ■ Ambenonium
○ The main neurotransmitter under the parasympathetic ■ Demecarium
system is the acetylcholine ■ TDGR
● Irreversible - causes permanent action which can lead to
Cholinergic Agonists
toxicity and even death
● Drugs that bind to the parasympathetic or cholinergic ○ Organophosphates - chemicals, drugs used as
receptors insecticides and pesticides
● Drugs that mimic acetylcholine ■ Parathion
○ Mimics the action of parasympathetic (miosis, ■ Malathion
bradycardia, bronchoconstriction) ■ Ecothiophate
● A.k.a “Parasympathomimetic Agents” and ■ Nerve gases
“Cholinomimetic Agents” ● Nerve gases that is used as warfare (causes
● Two classification of Parasympathomimetic or bronchoconstriction):
cholinomimetic: ○ Sarin
○ Direct acting - directly binds to receptor and mimics ○ Soman
the action of parasympathetic or cholinergic nervous ○ Tabun
system
○ Indirect acting - increases acetylcholine level by ● Neostigmine and physostigmine - treatment if ever
magkaroon ng sympathetic toxicity
inhibiting acetylcholinesterase. They will act on the
● If nagkakaroon ng toxicity from cholinergic agonist,
metabolism and reuptake and inhibits them to produce treatment is a sympathetic drug (Atropine)
the desired action. It blocks those that reduces the
neurotransmitter
CHOLINE ESTERS
● If magkakaroon ng toxicity from cholinergic agonist,
treatment is Atropine. ● Poorly absorbed & distributed in the CNS
● Charged with quaternary ammonium group renders then:
insoluble in liquid
● Hydrolized in the GIT (acetylcholinesterase) - Betanechol
and Meacholine are resistant

A. Acetylcholine
● The prototype drug, first drug under this category
○ Ginawang similar sa endogenous acetylcholine or
yung ginawa ng sarili nating katawan.
● Is quaternary ammonium compound that cannot penetrate
membranes
● Rapidly hydrolyzed
● Larged IV bolus injection has brief effects of 5-20 seconds
Direct Acting compared to IM or SC which has a local effect
● MOA: It will bind to the cholinoreceptor, muscarinic or
● Choline esters
nicotinic receptor to produce parasympathetic activity at
○ Acetylcholine
the site of action (brain, eyes, heart, GIT, reproductive
○ Metacholine
organ)
○ Carbachol

55
PCOL 211 - Pharmacology 1

2. NICOTINE
B. Metacholine
● From Nicutina tabacum
● acetyl-B-methylcholine ● Component of cigarette smoke
○ Mecholyl - brand in the market ● Acute toxicity: 40mg (1 drop of pure nicotine liquid) -
● More resistant to enzymatic hydrolysis than ACh fatal dose of nicotine
● Pharmacological actions: ● Chronic nicotine toxicity (because of cigarette
○ It can bind to muscarinic receptor and produce the smoking)
muscarinic parasympathetic or cholinergic activity ● Dangerous Effects of Nicotine:
○ No nicotinic actions ○ CNS action - convulsion, coma % respiratory
● Clinical use: not used clinically but used as asthma arrest
diagnosis ○ Use in smoking cessation: Nicorette gum or patch
● Binds to muscarinic receptor ○ Skeletal Muscle endplate depolarization -
respiratory paralysis, depolarization blockade
C. Carbachol
■ Depolarization - movement muscle
● Very potent (can lead to toxicity) choline ester both ○ Hypertension & cardiac arrhythmias
muscarinic and nicotinic effects 3. LOBELINE
● MOA: Binds to the muscarinic receptors and produce the ● plant derivative similar to nicotine (Lobelia inflata)
parasympathetic activity. Also, it can bind to a nicotinic 4. MUSCARINE
receptor which can cause muscle contraction and increase ● from mushroom Amanita muscaria (red mushroom)
brain activity ● A quaternary ammonium salt
○ When binds to Nicotinic m = contraction ● Action: Muscarinic action similar to Ach; No nicotinic
○ When binds to Nicotinic n = increased brain activity action
● Pharmacological actions: ● Causes: Mushroom Poisoning (mimic Ach)
○ act predominantly on GIT and urinary bladder once it 5. ARECOLINE
binds to the muscarinic receptor and leads to ● From Areca nut (Areca catechu)
defecation and urination
Mas makulay, mas toxic
○ The nicotinic actions of carbachol > Muscarinic actions
● Use: Treatment for glaucoma
○ Carbachol + Pilocarpine INDIRECT ACTING AGONISTS
● Carbamoylcholine - chemical name ● Also known as Acetylcholinesterase inhibitors
D. Betanechol (Urecholine) ● Increases acetylcholine in the synaptic cleft by inhibiting
acetylcholinesterase
● carbamoyl-B-methylcholine ● Produce their primary effects by inhibiting
● Not hydrolyzed by cholinesterases (resistant to acetylcholinesterase which hydrolyzes acetylcholine.
metabolism, more likely potent) ● Inhibits either metabolism or reuptake to increase
● MOA: It binds to the muscarinic receptor but not that acetylcholine levels
effective in nicotinic receptor
● Pharmacological actions: Mechanism of action
○ Act predominantly on GIT, UB ● Ach binds to the enzyme’s active site & hydrolyzed
○ No nicotinic actions yielding free choline & acetylated enzyme.
● It can increase intestinal motility which promote defecation, ● “All of the cholinesterase inhibitors increase the
and it ca nrelax the bladder sphincter which lead to concentration of endogenous Ach at cholinoceptor by
urination inhibiting acetylcholinesterase.”
Uses of Betanechol: ● TYPES:
A. REVERSIBLE INHIBITORS - Attach to the ACHE
1. Treatment of gastric retention enzyme and are only slowly hydrolyzed (nababago)
2. Treatment of phosphate abdominal distention B. IRREVERSIBLE INHIBITORS - Reacts to form a
3. Treatment of nonobstructive urinary retention stable, or covalent, phosphorylated enzyme, which is
4. Prevention of paralytic ileus essentially not hydrolyzed (permanent action)
ALKALOIDS INDIRECT ACTING AGONIST- REVERSIBLE
● Natural (derived from natural sources)
● Tertiary amine - well absorbed orally except muscarine A. Quaternary Alcohols
● Chlolinomimetics - stimulate receptor of acetylcholine
● 15-30 mins duration of action
1. PILOCARPINE
● Don’t form a covalent bond and bind reversibly at the
● From leaves of Pilocarpus jaborandi
active site. The action is therefore very short-lived.
● MOA: Binds to the muscarinic receptor and produces
● Edrophonium (Tensilon) -Short-acting; for diagnosis of
the acetylcholine or parasympathetic activity
Myasthenia gravis
● Use: Glaucoma - stimulates M3 in the pupil → miosis
○ Myasthenia gravis - An autoimmune disease
→ drains aqueous humor and decrease ocular
(Antibodies disrupt Nm receptors)
pressure
○ Ptosis - medical term for drooping eyelids
○ M3 (muscarinic 3) is located in eyes

56
PCOL 211 - Pharmacology 1

● TYPE OF ANTAGONISM:
B. Carbamates
○ Less than 24-48 hrs of exposure = potentially
● Intermediate-long acting reversible Greater than 24-48 hrs = definitely
● 2-8 hrs duration of action irreversible
● Pyridostigmine (Mestinon)
● Highly toxic compounds that were developed as
● Ambenonium (Mytelase) potential chemical warfare agents (nerve gas) (soman,
○ used for the treatment of muscle weakness and sarin, tabun)
fatigue in people with myasthenia gravis ● Certain agents in this class have therapeutic
applications, but their principal interest is toxicological
Physostigmine (aka eserine) because of their widespread use as insecticides.
● highly lipid soluble , crosses blood-brain barrier
● Can be used as treatment for parasympathetic toxicity Examples
● Pharmacological Action: similar to Ach ● Echothiophate
● USES: ● Isofluorophate – both are used for glaucoma
○ Ophthalmic uses ● Parathion - converted into Paraoxon
○ Treatment of poisonings with anticholinergic agents ● Malathion- converted into Malaoxon Both are used as
Neostigmine (Prostigmin) insecticide
● Sarin, Soman, Tabun - nerve gases, used in biological
● not lipid soluble, does not cross BBB warfare.
● Pharmacological Action similar to Ach ● SIGNS OF TOXICITY:
● USES: ○ Depends on entry
○ Post-operative ileus ■ Ingestion - DUMBBELS (muscarinic effects)
○ Urinary retention - bawal yung may obstruction MTWTHF (nicotinic effects)
○ Reverse NMJ block from curare-like drugs ■ Inhalation - Bronchoconstriction
■ NMJ - neuromuscular junction ■ Eye contact - irritation of the eyes, miosis
○ Trestment for Myasthenia Gravis ● ANTIDOTE
○ Antidote for tubocurarine toxicity ○ Atropine 3-4 mg every 15 min till recovery
C. Demecarium (Humorsol) ○ Pralidoxime - Cholinesterase activator or agonist
■ MOA: promotes metabolism of acetylcholine
● Ophthalmic Solution
● A cholinesterase inhibitor used to treat glaucoma by ● DUMBBELS
○ Diuresis
lowering the pressure inside the eye
○ Urination
● Inc. Ach → stimulates M3 in the pupilmiosis → drains ○ Miosis
aqueous humor and decrease ocular pressure ○ Bradycardia
○ Bronchoconstriction
D. Reversible inhibitors used in Alzheimer’s Disease ○ Emesis
● Tacrine (Cognex®) ○ Lecrination
○ Salivation
● Donepezil (Aricept®)
● MTWTHF
● Galantamine ○ Mydriasis
● Rivastigmine ○ Tachycardia
○ Weakness
Alzheimer’s ○ Hypertension
● a progressive disease, where dementia symptoms ○ Fasiculation
gradually worsen over a number of years.
● In its early stages, memory loss is mild, but with latestage PRALIDOXIME
Alzheimer's, individuals lose the ability to carry on a
● Cholinesterase activator
conversation and respond to their environment.
● Also known as 2-PAM
● Acetylcholine (ACh) is essential for processing memory
● It is used to combat poisoning by organophosphates or
and learning= decreased in both concentration and
acetylcholinesterase inhibitors (nerve gas), in conjunction
function in patients with Alzheimer's disease.
with atropine.
INDIRECT ACTING AGONIST- IRREVERSIBLE ● MOA:
○ It reactivates the cholinesterase by removing the
ORGANOPHOSPHATES phosphoryl group that is bound to the ester group. In
this reaction both the organophosphate and the
● Reacts to form a stable, phosphorylated enzyme, which
pralidoxime are mutually inactivated.
is essentially not hydrolyzed
● Available as: Pyridine aldoxime methiodide
● The pharmacological effects of cholinesterase inhibition
● Administered via IV 1-2 grams, add atropine 2-3 grams
persist until a new enzyme is synthesized.
● Very long-acting The Nerve Gases
● Duration of action: several days
● G-stands for Germany. Developed and stockpiled for use
in Germany during World War II.

57
PCOL 211 - Pharmacology 1

Nerve gases Chemical Name Myasthenia Gravis

Tabun (GA) - 1936 Ethyl ● An autoimmune disease (Antibodies disrupt Nm receptors)


dimethylphosphoramidocyanidate “Anti-ACR antibodies”
● Early manifestations:
Sarin (GB) - 1938 Isopropyl ○ Late afternoon muscle weakness
methylphosphonofluoridate ○ Late afternoon ptosis
● Final manifestation:
Soman (GD) - 1944 (3-3-dimethylbutane-2-ylmethylph ○ Diaphragmatic muscle weakness
osphonofluoridate
○ Associated with thymic hyperplasia/tumor (increase
activity of thymus gland)
Cyclosarin (GF) - 1949 (Cyclohexyl
methylphosphofluoridate ■ Thymus - train antibodies to be strong
● Diagnosis:
● Nerve agents have been used both in battle and in ○ Pharmacologic - Edrophonium (Tensilon Test).
terrorist attacks, such as the 1995 release of sarin on Rapidly relieved ptosis- (+) myasthenia gravis
commuter trains in Japan that killed 12 people and (right)
Iran-Iraq war. ○ Immunologic - detects anti-ACR antibodies
● Treatment:
1. Neostigmine, Pyridostigmine, Ambenonium
2. Thymectomy - removal of thymus gland

Cholinergic Antagonists
● Drugs that block the action of acetylcholine to the
cholinergic receptors.
● a.k.a “Parasympatholytic Agents”, Cholinergic blockers,
Anticholinergics
● ANTIMUSCARINICS
○ muscarinic antagonist
SUMMARY: ○ Aka: Anticholinergics
● Clinical Uses: ● ANTI-NICOTINICS
1. Dx and Tx of Myasthenia Gravis Edrophonium (Dx) ○ nicotinic antagonist
Neostigmine, Pyridostigmine, Ambenonium (Tx) ○ consist of ganglion-blockers & neuromuscular junction
2. Mgt. of Atropine toxicity (Neostigmine) blockers.
3. Mgt. of neuromuscular blocker toxicity (Neostigmine,
Edrophonium)
4. Useful for glaucoma (Pilocarpine, Ecothiopate,
Carbachol)
5. Use in non-obstructive ileus (Physostigmine)
6. For Alzheimer’s dx (TDGR)
7. Used in smoking cessation (Nicotine & Varenicline)
8. Mgt. of Urinary retention (Betanechol)
● Adverse/Toxic Effects:
○ DUMBBELS (muscarinic effects) MTWTHF (nicotinic
effects) Similar effects compare to ADRENERGIC
BLOCKERS (SYMPATHOLYTICS)
■ Adrenergic - similar nicotinic
■ Sympathetic blockers - similar muscarinic
● Contraindications:
1. Urinary or bowel obstructions
2. Coronary disease or hyperthyroidism
3. Asthma
4. Peptic Ulcer
5. Bronchial asthma
● Drug Interactions:
○ Intensifies cardiac depression caused by
beta-blockers.
○ Beneficial effects of parasympathomimetics may be
blocked by drugs that block muscarinic receptors
(atropine, tricyclic antidepressants, antihistamines,
phenothiazine antipsychotics).

58
PCOL 211 - Pharmacology 1

○ Twilight sleep - high ka, gising na happy, parang nasa


heaven
ANTIMUSCARINICS ● MOA: Block cholinergic receptor (Nm) resulting to
reduction of muscle contraction
ANTIMUSCARINICS ○ Nicotinic m - causes muscle contraction
● Transderm Scop - patch for motion sickness (sa likod ng
● Anticholinergic drugs ear nilalagay)
● Antiparasympathetic drugs ○ Balances fluid
1. ATROPINE ○ Relaxes muscles of GIT and uterus

● A sympathetic drug 3. IPRATROPIUM, TIOTROPIUM, OXYTROPIUM


● found in Atropa belladona or deadlynight shade or in ● Used in the management of BA and COPD
Datura stramonium or jimsonweed (Jamestown weed) or (bronchodilators) reduce the accumulation of secretions in
ggthorn apple. the trachea & laryngospasm
● l(-) hyoscyamine –naturally occuring atropine. ● Ipratropium (Atrovent) - more peripheral effects (LUNGS)
○ levo-hyoscyamine than CNS
● more potent than d(+) isomers. ● Ipratropium + Fenoterol (Berodual)
MOA ● Ipratropium + Salbutamol (Combivent)
○ Salbutamol aka Albuterol
● Blocks the cholinergic receptor to prevent the binding of
● Ipratropium - short acting (used for bronchial asthma)
acetylcholine, preventing DUMBBELLS
● Tiotropium - long acting (used for bronchial asthma)
● Atropine causes reversible blockade of cholinomimetic
● Oxytropium - normal acting
actions at muscarinic receptors.
● When atropine binds to the muscarinic receptor, it MOA
prevents the actions of muscarinic agonists such as ● Blocks muscarinic receptor in the lungs to promote
the release of IP3 & the inhibition of adenylyl cyclase. broncho dilation and reduces mucus production
● Prevent actions like DUMBBELLS effect ● Relax bronchial smooth muscle by antagonizing m3
Uses receptor

1. Mydriatic (dilation) / Cycloplegic (blurry vision) 4. BENZTROPINE, BIPERIDEN, TRIHEXYPHENIDYL


2. Mgt. of symptomatic bradycardia ● Management of Parkinsonism
3. Antidote for cholinergic agonist toxicity ○ Parkinson’s symptoms: tremors, bradykinesia (slow
4. Anti-diarrheal (w/ diphenoxylate) movement)
● Diphenoxylate (Lomotil®) ● Also used to manage extra pyramidal symptoms (EPS)
5. Mgt. of diarrhea caused by IRINOTECAN ● Biperiden - Akineton
6. 1st drug used in the therapy of tremor of Parkinson`s ● Benztropine -Cogentin
Disease. ● Trihexyphenidyl - Artane
Atropine Poisoning: “Reverse of Dumbbells” MOA
● Dry mouth ● Competitive antagonism of acetylcholine at cholinergic
● Mydriasis receptors in the corpus striatum, which then restores the
● Tachycardia - increase blood pressure balance.
● Hot and flushed skin
● Agitation 5. HOMATROPINE, CYCLOPENTOLATE, TROPICAMIDE,
● Delirium GLYCOPYRROLATE
Antidote: Physostigmine & Neostigmine ● Mydriatic and Cycloplegic
Anticholinergic Toxidrome ● Opthalmic examination Glycopyrrolate-for peptic disease,
● Mad as a hatter - altered mental status hypermotility
● Blind as a bat - mydriasis ● Homatropine (Isopto homatropine)
● Red as a beet - flushed skin ● Cyclopentolate (Cyclogyl)
○ Tachycardia = increase BP ● Tropicamide (Mydriacyl)
● Hot as a hare - dry skin (anhydrosis)
○ Dahil sa metabolism, gumagamit ng energy MOA
● Dry as a bone - dry mucous membranes
● Block the muscarinic receptor in eyes to cause mydriasis
2. SCOPOLAMINE or dilation of pupils

● occurs in Hyoscyamus niger or henbane, as the l(-) 6. ANTIMUSCARINICS THAT TREAT UROLOGIC
stereoisomer. PROBLEM
● Also known as Hyoscine ● OXYBUTININ - bladder spasm after urologic surgery
● Used for motion sickness and abdominal cramps ● TOLTERODINE- an M3-selective antimuscarinic used for
Morphine + Scopolamine = twilight sleep (an amnesic urinary incontinence in adults
condition characterized by insensitivity to pain without loss ○ MOA: Blocks muscarinic receptor to relax the wall and
of consciousness) contract the sphincter
● IMIPRAMINE - a TCA drug, used to reduce incontinence

59
PCOL 211 - Pharmacology 1

○ TCA - tricyclic antidepressant (urinary incontinence in


NEUROMUSCULAR BLOCKERS
kids)
■ Urinary incontinence - bed wetting, secretion of ● Blocks the cholinergic transmission between motor nerve
fluid endings and the nicotinic receptors on the neuromuscular
○ MOA: Blocks muscarinic receptor to relax the wall and end-plate of skeletal muscle.
contract the sphincter (same as tolterodine) ● Pampa-relax (e.g., kapag nag-iintubate)
○ Para yung muscles, hindi magcontract sa airway
7. PIRENZIPINE & TELENZIPINE
● MOA: Blocks Nm preventing the binding of acetylcholine,
● Used for the management of Peptic Ulcer Disease (PUD) preventing muscle contraction resulting to muscle
● Gastric acid, Pepsin & mucin are all reduced relaxation
● MOA: Blocks the muscarinic receptor at stomach (GIT)
A. NON-DEPOLARIZING
causing the reduction of acid (gastric acid, pepsin, and
mucin) helping in the treatment of peptic ulcer. ● Reversible
● True NM Antagonist
Parietal cells (also known as oxyntic cells) are epithelial cells
● Curare derivatives
in the stomach that secrete hydrochloric acid (HCl) and
intrinsic factor. ● MOA: Combines with nicotinic receptor & prevent the
binding of acetylcholine thus prevent depolarization of the
muscle cell membrane and inhibit muscle contraction
GANGLIONIC BLOCKERS
● Long Acting: d-Tubocurarine (d-TC), Metocurine,
● Most of these drugs are no longer used clinically and are Doxacurium, pancuronium, pipecuronium, gallamine
of historical importance only, because drugs that target the ● Intermediate acting: Atracurium, Cisatracurium,
ganglia usually have a broad range of effects and therefore
Vecuronium, Rcuronium
many side effects
● Stops sympathetic and parasympathetic nervous system ● Short Acting: Mivacurine, Rapacuronium
by blocking nicotinic receptor Curare
● Can block CNS and PNS
● USES: used as adjuvant in anesthesia during surgery
MOA
○ These agents also facilitates intubation for mechanical
● Acts on the nicotinic receptors (Nn), probably by blocking ventilation.
the ion channels of the autonomic ganglia. ● SIDE EFFECTS: induce histamine release and promote
● Blocks the action of acetylcholine and at the nicotinic ganglionic blockade (Bronchoconstriction, hypotension,
receptors of both parasympathetic and sympathetic respiratory paralysis)
autonomic ganglia. ● ANTIDOTE: Neostigmine (cholinergic drug)

EFFECTS B. DEPOLARIZING
1. CNS - sedation, tremor, choreiform movements, & mental ● Irreversible
aberrations. ● True NM Agonist
2. EYE - cycloplegia with loss of accomodation ● Succinylcholine is the prototype
3. Cardiovascular - vasodilation ● MOA: Attaches to the nicotinic receptor and acts like
4. GIT - constipation, decrease acid secretion acetylcholine to depolarize the junction that causes the
5. GUT - urinary retention in men with prostatic hyperplasia opening of the Na channel leading to fasciculations
(lumalaki ang prostate) (contraction) - Visible fast, fine, spontaneous and
intermittent contractions of muscle fibers.
DRUGS
○ Agonize the Nm
1. TETRAETHYLAMMONIUM (TEA) - first recognized ○ Binds to Nm receptor resulting to muscle contraction
ganglionic blocker and later on relaxation
2. HEXAMETHONIUM - first effective drug for management ● Contraction then relaxation
of hypertension. It is now rarely used for HTN due to lack
of selectivity compared to newer agents. Succinylcholine
● Over decreased of blood pressure ● USES: useful when rapid endotracheal intubation is
3. DECAMETHONIUM - An analog of hexamethonium required during the induction of anesthesia.
effective as neuromuscular depolarizing blocking agent. ○ employed during electroconvulsive shock treatment.
4. MECAMYLAMINE - secondary amine studied for possible ○ Used in surgery
use in reducing nicotine craving in patients attempting to ● ADVERSE EFFECT: may caused malignant hyperthermia
quit smoking. when halothane is used as anesthetic.
5. TRIMETHAPHAN - short-acting ganglion blocker, ○ Hyperthermia - over contraction of the muscle, using
occasionally used in the treatment of hypertensive energy
emergencies & dissecting aortic aneurysm. ● ANTIDOTE: Dantrolene
● Hypertensive - high blood pressure with organ failure ○ Kapag walang Dantrolene, mag ice bath or maligo
6. Alkaloids ● MOA: It reduces the release of activator calcium from the
● NICOTINE - initially stimulate then block ganglia. sarcoplasmic reticulum
● LOBELINE - from Indian tobacco (Lobelia inflata)

60
PCOL 211 - Pharmacology 1

pheochromocytoma, which can cause severe hypertension


DRUGS AFFECTING SYMPATHETIC and cardiac arrhythmias.

NERVOUS SYSTEM CATECHOLAMINES


● The endogenous neurotransmitters contain three required
Sympathetic Nervous System structural features. They contain a catechol nucleus (i.e.
● a.k.a. “ADRENERGIC” system. hydroxyl groups in the meta and para position on the
● Responsible for arousing and adrenaline rush or “FIGHT or aromatic ring), a b-hydroxy group and a nitrogen
Flight” response substituent.
● It has the property of adjusting in response to stressful ● Norepinephrine - catechol
situations such as trauma, fear, hypoglycemia, cold or ● Epinephrine - catechol + hydroxyl group
exercise.
● NEUROTRANSMITTER: Norepinephrine (Noradrenaline),
Epinephrine
○ Made from the adrenal medulla, located at the adrenal
gland, on the top of kidney
● RECEPTORS: Alpha, Beta, Delta
● Location: Adrenal medulla
● Expenditure of energy: Ergotropic

NOREPINEPHRINE
● The primary neurotransmitter of postganglionic
sympathetic adrenergic nerves.
● Synthesized inside nerve axon in the adrenal medulla and
stored within vesicles. Catechol Resorcinol Hydroquinone
● It is released by the nerve when an action potential travels
down the nerve. Biosynthesis of Catecholamines
● Fates:
1. Biosynthesis
2. Release
3. Binding to receptor (α, β and D)
4. Metabolism (30% inactivation) – MAO & COMT
5. Reuptake (70% of inactivation

Biosynthesis
● Sodium (Na+) - facilitates the entry of tyrosine
● Tyrosine-DOPA = catalyzed by tyrosine hydroxylase
considered as “rate-limiting step”
● DOPA-Dopamine = catalyzed by dopa decarboxylase.
Removal of carboxylic acid (COOH) group
● Dopamine-Norepinephrine = catalyzed by
dopamine-b-hydroxylase. Addition of (OH) group in C2/ Receptors of Catecholamines
beta carbon. NE will be stored in the vesicle
● NE-Epinephrine = catalyzed by PENMT (Phenyl ● Receptors of catecholamines are example of TYPE II
ethanolamine N-methyltransferase). Methyl conjugation of receptors
amino group ● G-Protein Linked Receptors
● Calcium (Ca+2) - facilitates the release of 1. Gi-inhibitory G-protein of adenylyl cyclase.
neurotransmitter (exocytosis) 2. Gs-stimulatory G-protein of adenylyl cyclase.
3. Gq-the protein coupling α receptors to phospholipase
Inactivation C.
● REUPTAKE Receptor Location Response
○ Reuptake of neurotransmitter
○ Responsible for 70% of inactivation Blood vessel Vasoconstriction
○ Facilitated by NET (Norepinephrine transporter)
● METABOLISM Smooth muscle Contraction
○ Responsible for 30% of inactivation
Pupil Mydriasis
○ MAO (Monoamine oxidase / Phase 1 metabolism / α1
oxidation) and COMT (Catechol-O-methyltransferase /
Pilomotor muscle Hair erection
Phase 2 / methylation)
Vanillylmandelic acid (VMA) - This final product is Bladder sphincter Contraction
measured in urine and plasma in the diagnosis of

61
PCOL 211 - Pharmacology 1

Prostate Contraction Cardiac muscle (+) chronotropic

● BLOOD VESSELS ● LUNGS


● MOA: Linked to the Gq protein (increasing IP3 and ● Increase heart rate
DAG) resulting to the vasoconstriction of the blood
vessel
Receptor Location Response
Receptor Location Response
β3 Adipose cells Lipolysis
Pre-synaptic Exert negative
nerve terminal feedback leading ● FATS
(mas abundant sa to decrease in the ● Lipolysis - breakdown of fats to produce enough
pre-synapse) synthesis and energy; to produce blood sugar (glucose) which is
α2 release of NE useful in fight or flight response
(decrease BP,
vasodilation)
Receptor Location Response
Post synaptic Vasoconstriction
nerve terminal (increase BP) Renal and Vasodilation
D1 splanchnic blood
● Gi protein linked - decrease CAMP vessels
● Negative feedback - reduction of the body’s function up
to normal level BRAIN Modulation of
○ Positive feedback - increases those na kulang. - Substantia nigra motor activity,
Plus lang nang plus hanggang sa sobra na D2-D4
- Vomiting center behavior, mood
and thought
process
Receptor Location Response
RECEPTOR REGULATION
Cardiac muscles (+) inotropism,
dromotropism,
Desensitization
β1 chronotropism
● can be described as the loss of response subsequent to
Juxtaglomerular Renin release prolonged or repeated administration of an agonist.
cells
○ Tolerance - loss of adapting to a substance
● HEART ○ Refractoriness
● Inotropism - increase cardiac contraction (causes ○ Tachyphylaxis - fast tolerance to a drug product (sa
tachycardia) next na inom ng drug, hindi na effective)
● Dromotropism - increase cardiac electric conductivity ● Mechanism: Transcriptional/Translational
(causes tachycardia)
● Chronotropism - increase heart rate ● Desensitize - nawawalan na ng pakiramdam
● Pacemaker channels - SA (sinoatrial) node ● Sensitization - sensitive; with feelings (6:26)
○ Source of electric conduction
○ Nasa right atrium Homologous Desensitization
● Purkinje fibers - pag nakareceive ng message,
magcocontract ● Loss of responsiveness exclusively of the receptors that
● Kidney regulates blood pressure have been exposed to repeated or sustained activation by
● Renin - activates RAAS system a drug.
(renin-angiotensin-aldosterone) ● Both α1 and α2

Heterologous Desensitization
Receptor Location Response ● Loss of responsiveness of some cell surface receptors that
have been directly activated by a drug.
Bronchial Dilation
● α1 lang or α2 lang
Uterus Relaxation / Phosphorylation
Tocolytics
● Major mechanism of desensitization that occurs rapidly by
Intestinal Relaxation members of the G-protein coupled receptor kinases
β2 (GRK’s). GPCR phosphorylation by second messenger-
Vascular Vasodilation dependent protein kinases such as protein kinase (PKA)
and protein kinase C (PKC) was regarded as the principal
Skeletal muscle Contraction mechanism of GPCR desensitization.
● Once G-protein is activated
RBC/ other cells K+ influx ● Kapag may iba pang bagay silang na-aactivate

62
PCOL 211 - Pharmacology 1

○ have a β-hydroxyl group


SYMPATHOMIMETIC AGENTS
● Drugs that mimic the action of epinephrine or
norepinephrine
● A.k.a. “Adrenomimetic Drugs”, “Adrenergic Agonists”,
“Adrenoreceptor Agonists”
a. Direct acting - directly binds to the receptor
b. Indirect acting - increase NE/E by blocking
metabolism/reuptake
c. Centrally acting - directly activate the brain receptors

Chemistry of Sympathomimetic Agents

Direct Acting Sympathomimetic Agents

A. Non-Selective - Stimulate more than 1 general type of


receptors
● Walang pinipili, lahat bibind-an
B. Selective - Stimulate only one general type of receptor
● Isa lang ang bibind-an

Direct Acting (Non-Selective) Sympathomimetic


Pharmacophore for adrenergic agents
A. Natural Catecholamines (natural sa body kaya magbibind
Phenylethylamine sa lahat)
● Epinephrine
● considered as the parent compound from which ● Norepinephrine
sympathomimetic drugs are derived. ● Dopamine
● considered as the parent compound from which B. Isoproterenol
sympathomimetic drugs are derived.
● SUBSTITUTION Catecholamines
○ Terminal amino group ● Pharmacokinetics:
○ Benzene ring ○ All of them have poor oral absorption (EXTENSIVE
○ α or β carbons FPE)
● Catecholamines - substitution by -OH groups at 3,4 ■ FPE - first-pass effect or metabolism
positions. ● Pharmacodynamics:
● SUBSTITUTION ON TERMINAL AMINO GROUP ○ NE: stimulate α1 & β1
○ Methyl substitution on NE, yielding Epi, enhances ○ Epi: stimulates α1, β1 and β2
activity at β2 receptors. ↑ β-receptor activity. ○ Dopamine: stimulates α1 & β1 and Dopamine
○ The larger the substituent on the amino group, the
lower the activity at α-receptor. Epinephrine

● A.k.a “Adrenaline”
● Potent vasoconstrictor and cardiac stimulant
● Gives positive inotropic and chronotropic effect
● CLINICAL USES:
1. Bronchospasm (SQ, inhalational)
2. Glaucoma
● At α-receptors 3. Anaphylactic shock (IV)
○ Norepinephrine ≥ Epinephrine >> Isoproterenol 4. Adjunct in anesthetics (SQ) to minimize systemic
○ Mas nag-bibind si norepinephrine kay alpha absorption & toxicity
○ ↑ hydrogen = ↑ binds to alpha 5. First-line cardiac stimulant (IV)
● At β-receptors ● DOC for anaphylaxis (it promotes bronchoconstriction)
○ Isoproterenol >> Epinephrine >> Norepinephrine ● Epinephrine + Lidocaine (local anesthetic) = increase local
○ ↑ carbon = ↑ binds to beta effect
● Binds to α1 - vasoconstriction
● Substitution on the BENZENE RING ● Binds to β1 - tachycardia (increase cardiac contractility
○ maximal α & β activity and heart rate)
● Substitution on the ALPHA CARBON ● Binds to β2 - bronchodilation
○ Dec. metabolism by monoamine oxidase (MAO) ● MOA: Binds to α1, β1 and β2 receptor resulting to
○ Ex. Ephedrine & Amphetamine vasoconstriction, tachycardia, and bronchoconstriction
● Substitution on the BETA CARBON
○ by direct-acting agonists

63
PCOL 211 - Pharmacology 1

2. Used to overcome hypotension during surgery caused


Norepinephrine
by halothane anesthetics
● A.k.a. “Levarterenol”, “Noradrenaline” ● CLINICAL USES:
● Clinically used in the management of septic shock (SBP 1. Nasal decongestant (PO or intranasal)
<90mmHg) 2. Local vasoconstrictor (+ local anesthesia) (SQ)
○ SBP - Systolic Blood Pressure (low blood = low 3. Management of acute hypotension (IM or IV)
oxygen) 4. Used to terminate supraventricular tachycardia
○ Septic shock - decrease in brain activity due to an ● ADVERSE EFFECTS:
infection ○ Systemic (PO):
● MOA: Binds to α1 and β1 receptors resulting to 1. Tolerance (nmt 5 days)
vasoconstriction and tachycardia 2. Exacerbation of hypertension
3. Urinary retention
Dopamine ○ Topical effects (inhalational)
● immediate metabolic precursor of norepinephrine. 1. Rebound hyperemia (limited up to 3 days use
● USES: only)
1. DOC for septic shock ● A allergic reaction
2. Given in acute oliguric renal failure ● MOA: Binds to α1 receptor resulting to vasoconstriction
3. Management of cardiogenic shock (CHF) (increase blood pressure)

Isoproterenol SELECTIVE A2 AGONISTS

● a.k.a “ Isoprenaline” very potent non selective β-receptor ● Methyldopa


agonist potent bronchodilator ● Clonidine
● USES: ● Guanfacine
1. Management of BA & COPD ● Guanabenz
2. Management of shock states & acute heart failure ● EFFECTS:
(alternative) ○ Pre-synaptic: Vasodilation, Sedation, Depression
● MOA: Binds to α1, β1 and D1 receptors resulting to ○ Post synapse: Transient vasoconstriction
vasoconstriction, tachycardia and bronchodilation

Direct Acting (Selective) Sympathomimetic Agents

I. Alpha 1 agonists
II. Alpha 2 agonists
III. Beta1 agonists
IV. Beta 2 agonists
V. Dopamine 1 agonists

SELECTIVE A1 AGONISTS

● Naphazoline
● Oxymetazoline
● Xylometazoline
● Propylhexedrine
● Phenylephrine (PO, IV, IM, SQ)
○ Present in Neozep

Phenylephrine Clonidine
● Relatively a pure α agonist. it is not a catecholamine, ● Analogues: Brimonidine, Apraclonidine
therefore it is not inactivated by COMT, resulting longer ● Brand name: Catapres - 75 mg, sublingual (90-100%
duration of action. used as nasal decongestant used to bioavailability)
raise blood pressure & to terminate episodes of ● USES:
supraventricular tachycardia. 1. Management of HTN in patients on hemodialysis
○ Supraventricular tachycardia - upper side of the 2. Alternative in the mgt. of ADHD
ventricle (atrium) ● ADHD - Attention Deficit Hyperactivity Disorder
● MOA: Binds to α1 receptor resulting to vasoconstriction, (DOC: Methylphenidate)
useful as decongestant 3. For clonidine withdrawal-induced, HTN and
● Contraindicated with hypertension hypertensive crisis
● Hypertensive crisis - hypertension with organ
Methoxamine
failure (stroke, aneurysm)
● Predominantly a direct acting α1-receptor agonist. can ● Has effects both on pre and post synapse
cause increase in BP ○ Pre synapse - vasodilation
● USES: ○ Post synapse - vasoconstriction (increase BP)
1. Clinically used to relieve attacks of paroxysmal
supraventricular tachycardia.

64
PCOL 211 - Pharmacology 1

4. Tocolytic agent (mgt. of premature labor)


Methyldopa
● Tocolytic - uterine relaxation
● α-methyldihydroxyphenylalanine ● Oxytoxic - uterine contraction
● Is a prodrug (will be activated upon metabolism) 5. Adjunct in the management of hyperkalemia
● Mimics norepinephrine ● A/E:
● Promotes vasoconstriction 1. Salbutamol/Albuterol - tremors (contraction of skeletal
● It is converted to alpha-methylnorepinephrine a false muscles)
neurotransmitter that produces transient vasoconstriction 2. Muscle weakness (hypokalemia)
and long-term vasodilation. 3. Tachycardia (at high doses)
● USE: ● MOA: Binds to β2 receptor and may cause
○ Alternative in mgt. of HTN during pregnancy. bronchodilation, vasodilation, cardiac contraction and
(Nifedipine, Hydralazine, Labetalol) uterine relaxation
■ Eclampsia - hypertension in pregnancy
SELECTIVE D1 AGONISTS
■ It cannot cross the placenta
● A/E:
Fenoldopam
1. Sedation (most common)
2. Hepatotoxicity ● MOA: A D1-receptor agonist that selectively leads to
3. False (+) Coomb’s test peripheral vasodilation
● Coomb’s test - used to determine immune- ● Intravenously administered for the treatment of severe
mediated hemolytic anemia (in newborns) hypertension.
● MOA: Binds to α2 receptor at the pre synapse resulting to
negative feedback causing vasodilation Indirect Acting Sympathomimetic Agents
○ Post synapse - causes vasoconstriction ● Increases the concentration of catecholamines in the
synaptic cleft.
SELECTIVE B1 AGONISTS
● Mechanisms:
1. Increase catecholamine release
Dobutamine
2. Inhibition of reuptake (REUPTAKE INHIBITORS)
● β1-selective synthetic catecholamine.
Ephedrine
● USES:
1. Clinically used as inotropic agent (cardiogenic shock, ● A plant alkaloid, found in Ma-huang.
CHF) ● A non-catechol phenylisopropylamine, it has long duration
2. Pharmacologic Stress Testing (Dobutamine Stress of action & bioavailability
Test). ● Used to treat asthma, as nasal decongestant
● Pinapatakbo sa treadmill (Pseudoephedrine) and mgt. of hypotension
● MOA: Binds to β1 receptor and may activate cardiac ○ Some time before, it is used as energy enhancer of
inotropic and chronotropic activity, and it may activate or athletes
stimulate the release of renin ● A/E: Hypertension, tachycardia, abused by athletes
● MOA: Directly stimulates adrenergic receptors, which
SELECTIVE B2 AGONISTS
stimulates norepinephrine release

SABA (Short-acting LABA (Long-acting Amphetamine


β2 agonist) β2 agonist)
● A phenylisopropylamine
Albuterol Salmeterol ● Marked as CNS stimulant
Terbutaline Formoterol ● Under Schedule 1 drug (no clinical use but addictive drug)
Metaproterenol Bambuterol ● Used in the therapy of depression, hyperactivity (because
Pirbuterol Indacatero it increases focus), narcolepsy (sudden sleep), & appetite
control
Used as reliever for acute Controller or maintenance ○ Alternative for ADHD as it increases CNS stimulation
asthma drug (once or twice lang and focus
ginagamit) ● MOA: Inhibits reuptake of norepinephrine

Tyramine
● CLINICAL USES:
1. Management of bronchial asthma ● Found in fermented foods, such as ripe grapes cheeses &
● SABA - reliever (for acute attacks of asthma) Chianti wine.
● LABA - controllers ● Normal by-product of tyrosine metabolism.
2. Used in COPD (Chronic obstructive pulmonary ● Oxidized by MAO
disease) ● Can precipitate vasopressor episode when taken with
● SABA - for acute exacerbation (biglang hindi MAOI.
makahinga) ● Tyramine + MAOI = high NE = vasoconstriction,
● LABA - maintenance treatment tachycardia = hypertension
3. Management of symptomatic bradycardia
(Terbutaline)

65
PCOL 211 - Pharmacology 1

○ Avoid adding tyramine and MAOI because both of


ALPHA BLOCKERS
them will increase the number of NE which can lead to
hypertensive crisis ● Drugs that block the α adrenoceptors profoundly affect
blood pressure.
Centrally Acting Sympathomimetic
● Reduces the sympathetic tone of the blood vessels
● Phenylpropanolamine resulting to decreased peripheral vascular resistance
● Phentermine (causes vasodilation)
● Phenmetrazine ● A/E: Postural hypotension & reflex tachycardia.
● Amphetamine ○ Postural hypotension - also known as orthostatic
● Methylphenidate hypotension (when you suddenly change position from
● Modafinil lying to upright position, nagkakaroon ng sudden
● USES: decrease in blood pressure due to the help of gravity)
1. Management of ADHD (1st line: Methylphenidate) ○ Reflex tachycardia - Because of the hypotension effect
2. Used to treat NARCOLEPSY (sleeping disorder) of alpha blockers, you heart will compensate with that
● Because of the increase number of NE will (mababa ang blood pressure sa peripheries,
stimulate the brain kailangan padalhan ng dugo ng heart. Bibilis ang
3. Appetite suppressants (Phentermine, Phenmetrazine, takbo ng puso para makapag supply ng dugo kung
Phenylpropanolamine) saan bumaba yung blood pressure)
● A/E:
Non-Selective Alpha Blockers
1. CNS stimulation (risk of addiction)
2. Hemorrhagic stroke (because of vasoconstriction) ● PHENOXYBENZAMINE
3. Primary pulmonary HTN (phenmetrazine & PPA-young ○ Causes irreversible antagonist of alpha receptor
women) (permanent action)
● PPA (phenylpropanolamine) - a vasoconstrictor ○ It has Acetylcholine, serotonin, and Histamine effect
and nasal decongestant ○ MOA: It will block alpha receptor resulting to
○ A/E: Rhinitis medicamentosa or rebound vasodilation and causes allergic reaction
congestion ● PHENTOLAMINE
○ Reversible antagonist of alpha receptor (changeable
SYMPATHOLYTIC AGENTS
action)
● a.k.a. “Adrenergic Antagonists”, “Adrenoceptor ○ MOA: Blocks alpha receptor and produce vasodilating
Antagonists”. effect but it can be reversible if needed
● Sympathetic antagonist (breakdown of sympathetic action)
● MOA: Binds to adrenoceptors but do not trigger the usual Selective Alpha Blockers
receptor-mediated intracellular effects (they will block the ● ALPHA 1: (They will selectively inhibit alpha 1 receptor
effect of the receptor) resulting to vasodilation)
Mimetic - agonist ○ Prazosin
Lytic - antagonist ○ Doxazosin
○ Terazosin
○ Alfuzosin
○ Tamsulosin
● ALPHA 2: (Directly blocks alpha 2 receptor)
○ Yohimbine

Clinical Uses of Alpha Blockers

1. Mgt. of HTN
● Caused by pheocromocytoma
● BPH or Benign prostatic hyperlasia (Selective α1
blocker)
○ Alpha 1 causes constriction in prostate
2. Mgt. of Reynaud’s Phenomenon
● (Non-selective and Selective α1 blocker) to cause
vasodilation, to open blood vessels, to allow blood
flow
3. Mgt. of Urinary Retention
● Beta receptor antagonists (-olol)
● Alpha 1 antagonists (-zosin) ● Alpha agonist has sympathomimetic effect which
● Propranolol (Inderal®) - used for stage fright deceases urine because of the constriction of the
bladder sphincter
● (Selective α1 blocker) - will cause relaxation of the
bladder sphincter to increse urination
4. Mgt. of Carcinoid Syndrome
● Phenoxybenzamine
5. Mgt. of Erectile dysfunction

66
PCOL 211 - Pharmacology 1

● Phentolamine (Inj.) - will cause vasodilation to allow


Two Types of Beta Blockers
erection
● Directly injected into the penile shaft
1. Intrinsic Sympathomimetic Activity (ISA)
● PO: Sildenafil, Tadalafil
● Has partial agonistic effect
Pheochromocytoma
○ Pindolol
● A tumor in adrenal gland that causes increased release of ○ Acebutolol
NE, which increases BP because of vasoconstriction ○ Labetalol
● Diagnosis: ○ Penbutolol
1. VMA assay in serum/urine ○ Carteolol
2. Imaging studies (MRI/CT scan) ● Advantage: Less associated with rebound hypertension or
● Management: tachycardia upon withdrawal.
1. Initial control (alpha and beta blockers)
2. Membrane Stabilizing Activity (MSA)
2. Surgical intervention (removal of the tumor)
3. Tumor/hyperplasia in the adrenal medulla ● Has quinidine-like effect or anesthetic effect.
4. Hypersecretion of NE and Epi (vasoconstriction and ○ Pindolol
increase BP) ○ Acebutolol
● Manifestations: ○ Labetalol
1. Agitation (hindi mapakali, masama pakiramdam) ○ Metoprolol
2. Confusion ○ Propranolol
3. Tachycardia
● With mixed α & β-blocking effect (α & β-blockers)
4. Palpitation
○ Labetalol
5. Paroxysmal HTN ○ Carvedilol
● Phenoxybenzamine and phentolamine can be given ● Dahil mixed yung action, kapag nablock yung alpha, it
causes vasodilation. Kapag nablock yung beta, it
Reynaud’s Syndrome causes decrease ng contractility and conduction, and
● Digital vasospasm in a cold environment decrease renin release
● A disease that causes some areas of the body such as
fingers and toes to feel numb and cold in response to cold Non Selective Beta Blockers
temperature or stress
○ Smaller arteries that supply blood to your skin become Propranolol
narrow, limiting blood flow to the affected areas (kaya
● Inderal®
biglang pumuputi yung dulo ng daliri, sumisikip yung
● Prototype (first drug under this category), non-selective
daanan ng blood, nawawalan ng dugo)
β-blocker.
○ Kapag hindi nagamot, mabubulok yung mga daliri
● Used for hypertension, glaucoma, migraine,
● Vasoconstriction - digital necrosis
hyperthyroidism, angina pectoris, myocardial infarction,
● Management:
and stage fright (mga kinakabahan kapag nasa stage na)
1. Non-selective and selective alpha blockers
● MOA: It will block both β1 and β2 receptor. It decreases
2. CCBs - Calcium channel blockers
the contractility of the heart, but it may cause
3. Tolazoline (Priscoline®)
bronchoconstriction
Carcinoid Syndrome ● Contrindicated and should be avoided to be given to
asthmatic patients
● Neuroendocrine tumors
● Hypersecretion of 5-HT (serotonin) by tumor affecting the Timolol
enterochromaffin cells in the GIT.
● Non-selective agent with no local anesthetic activity (no
● Manifestations:
MSA activity)
1. Severe headache
● Excellent ocular hypotensive effects.
2. Watery diarrhea
● It decreases intraocular pressure which is useful to
3. Flushing
patients with glaucoma
BETA BLOCKERS
Levobunolol
● Drugs that blocks beta receptors and may cause
vasodilation, decreased contractility of the heart, and ● Non-selective agent for glaucoma
bronchoconstriction ● Decreases intraocular pressure in the eyes
● With Intrinsic Sympathomimetic Activity Carteolol
Non Selective (First Generation) ● Have partial agonist effect (ISA activity)
● They will block both β1 and β2 receptor causes decreased ● Effective for glaucoma
contractility of the heart, decreased renin release, and ● Avoids having rebound HTN
bronchoconstriction ● Also decreases intraocular pressure in eyes
● Cannot be given or contraindicated with asthmatic patients Labetalol

● Mixed α & β-blocker

67
PCOL 211 - Pharmacology 1

● Alternative to hydralazine in treatment of pregnancy-


induced hypertension (PIH) or eclampsia
● MOA: Blocks alpha receptors and cause vasodilation, and
LOCAL ANESTHETICS
blocks beta receptors causing decreased contractility,
decreased renin release, which are helpful for HTN Anesthetic Agents
Selective Beta Blockers ● Agents that are used during before or afer surgery

LOCAL ANESTHETICS
Betaxolol
● A drug that causes anesthesia and a loss of nociception.
● β1-selective blocker used for glaucoma. ○ Nociception - pain perception (pakiramdam)
● Selectively inhibits β1 receptor and it decreases ● When it is used on specific nerve pathways (nerve block),
contractility and may also decrease the intraocular effects such as analgesia (loss of pain sensation) and
pressure in the eyes paralysis (loss of muscle power) can be achieved.
Clinical Uses of Beta Blockers ● Reversibly block impulse conduction along nerve axons
and other excitable membranes that utilize sodium
1. Mgt. of HTN in px w/ CAD channels
2. Mgt. of arrythmia (Class II anti-arrythmic agent) ● Affect permeability of nerve membranes to sodium ions –
3. Used to manage Chronic stable angina pectoris prevent sodium ions from entering the nerve - nerve
4. Mgt. of Stable heart failure (responsive to meds) cannot depolarize - particular section of the nerve cannot
Metoprolol, Carvedilol, Bisoprolol be stimulated.
5. Mgt. of Glaucoma (Timolol and Betaxolol) ● May specific site
6. Manage sympathetic symptoms of hyperthyroidism ● MOA: Block the sodium channel preventing depolarization
(Tachycardia) at the specific site
7. Prophylaxis of migraine
8. Mgt. of stage fright (Propranolol) ● Na - may movement na magagawa
○ + ion
Adverse E ects of Beta Blockers ○ Promotes electricity
○ Loss on Na is nanghihina
● Bronchospasm ● K - promotes slowness of movement
● Decreased exercise tolerance ● Ca - more action, more movement
● Bradycardia ● Cl - hyperpolarization (relaxation)
● Heart block
● Dyslipidemia Ideal Characteristics
● Decreased sexual activity
1. Rapid/fast onset
Contraindications of Beta Blockers ● Kailangan mag numb kaagad para malinis at
maoperahan
● Anaphylaxis
● Within 30 mins dapat wala na nararamdaman
● Bradycardia/Heart block
2. Long Duration of Action
● Active bronchial asthma
● To stay for a long period of time during the period of
● Unstable heart failure
surgery
● Patients taking Non-dihydropyridine CCBs
● At least 1-2 hours
● Diabetes melitus
3. Reversible & selective blockade of sensory nerves without
Innovator Brands motor blockade
● Mawawalan ng sensation or feeling pero maigagalaw
● Acebutolol - Sectral mo pa rin yung katawan mo
● Atenolol -Tenormin 4. Minimal local tissue irritation & no systemic toxicities
● Betaxolol - Kerlone (cardiac & CNS)
● Bisoprolol - Zebeta ● Especially pag nag add ng Epinephrine
● Carteolol - Cartrol
● Carvedilol - Coreg Local anesthetics are administered locally or topically.
● Labetalol - Normodyne Kung saan lang yung area sila inadministered, doon lang
dapat yung effect nila.
● Metoprolol - Lopressor
● Nadolol - Corgard
● Penbutolol - Levatol History
● Pindolol - Visken ● Cocaine was the 1st local anesthetic isolated from the
● Propranolol - Inderal Coca plant as an ophthalmic anesthetic.
● Timolol - Blocadren ○ the first agent isolated by Niemann in 1860.
○ It was introduced into clinical use by Koller in 1884 as
an ophthalmic anesthetic.
○ strongly addicting
○ widely used for travelling
○ 1905 - Einhorn synthesized procaine. Since 1905,
many local anesthetic agents have been synthesized.

68
PCOL 211 - Pharmacology 1

○ 1943 - Löfgren synthesized Lidocaine - considered ● Prilocaine


the prototype local anesthetic agent. ● Ropivacaine
■ Emla® - pinaka sikat
METHODS OF ADMINISTRATION
Goals in the synthesis of new local anesthetics
1. To reduce local irritation and tissue damage 1. Surface Anesthesia
2. Minimize systemic toxicity ● direct application of the drug on the surface such as skin
3. Faster onset of action and wounds.
4. And longer duration of action. ● Cream type
Basic Pharmacology of Local Anesthetics ● Lidocaine
● Topical
● CHEMISTRY
○ Weak bases and are usually made available clinically 2. Infiltration Anesthesia
as salts to increase solubility and stability. ● injection in subcutaneous tissue in order to paralyze nerve
● Consist of:
1. lipophilic group - aromatic ring 3. Nerve Block
2. membrane penetration intermediate chain - ester or ● injected in the vicinity of major nerve such as teeth
amide ● Peripheral nerve block - blocks sensory & motor aspects
3. ionizable group - for channel blockade (tertiary amine) of a particular nerve for relief of pain or for diagnostic
● Increase lipophilic character purposes
1. Increase potency ● Central nerve block -involves injecting the anesthetic into
2. Decrease metabolism the roots of the nerves in the spinal cord
3. Increase toxicity ● Epidural anesthesia - injection of the drug into the space
where the nerve endings emerge from the spinal cord
○ Used sa CS na gising yung patient
● Caudal block injection - into the sacral canal, below the
epidural area

4. Field Block
● involves injecting the anesthetic all around the area that
will be affected by the operation
● often used for tooth extractions

5. Sympathetic Block

Two Classes of Local Anesthetics ● inject around sympathetic nerves

● Ester 6. Spinal Anesthesia


○ more prone to hydrolysis ● injection into subarachnoid space in the lumbar region
○ short duration of action ● Spinal block - penetration of the anesthetic into the
● Amide subarachnoid membrane
○ more stable ● Epidural block - placement of the local anesthetics in the
outer covering of the spinal cord
CLASSIFICATION
● Caudal block - inject near the sacrum (dulong buto ng
spinal)
ESTERS
● Saddle block - given at the lower end of the spinal column
● Esters of Benzoic Acid. to block the perineal area
○ Cocaine
7. Intravenous Regional
○ Benzocaine
○ Tetracaine ● Local anesthesia + epinephrine
○ Butacaine ● involves carefully draining all of the blood from an arm or
○ Hexylcaine leg, securing a tourniquet to prevent the anesthetic from
● Esters of p-aminobenzoic acid entering the general circulation, & then injecting the
○ Procaine (pinakasikat, used in minor surgery) anesthetic into the vein of the arm or leg
○ Chloroprocaine
Mechanism of Action
○ Propoxycaine
● Block initiation & propagation of action potential (AP) by
AMIDES
preventing voltagegated Na+ channels
● Articaine
PHARMACOKINETICS
● Bupivacaine - for cesarean section
● Dibucaine 1. Absorption in highly vascular area is > poor perfused
● Etidocaine tissues.
● Lidocaine - popular, used in minor surgeries ● Perfused tissues - blood supply
● Mepivacaine ● High blood supply = high bioavailability

69
PCOL 211 - Pharmacology 1

2. All local anesthetics cross the BBB & the placenta and ○ Convulsions: excessive level in blood. Premedication
enter the blood stream of the developing fetus with benzodiazepines as prophylaxis.
● Has aromatic ring that cross BBB ● for COCAINE
3. Use of Vasoconstrictors (epinephrine/ phenylephrine) ○ widely abuse drug, severe CV toxicity; HTN,
4. Onset of action - can be accelerated by the use of arrhythmia, & myocardial failure
solutions saturated with carbon dioxide ("carbonated").
At higher concentrations
● Vasoconstrictors
○ Slow removal & reduce systemic absorption of LA ● Nystagmus and muscular twitching occur.
from injection site by decreasing blood flow (upto ○ Nystagmus - movement of eyes side by side
30%) & cause higher local tissue concentration. ● Overt tonic-clonic convulsions followed by central nervous
○ reduce CNS & systemic toxicity. system depression and death may occur
5. Repeated Injection
Seizure
● can result in loss of effectiveness (ie, tachyphylaxis)
due to extracellular acidosis. Local anesthetics are ● can also be treated with small doses (given intravenously):
commonly marketed as hydrochloride salts (pH 4.0– ○ Thiopental 1–2 mg/kg
6.0) ○ Propofol 0.5–1 mg/kg
○ Midazolam 2–4 mg total dose or diazepam 0.1 mg/kg.
Metabolism ● The muscular manifestations of seizures can be
A. Amide Local Anesthetics’ primary site of metabolism is the suppressed by a short-acting neuromuscular blocking
liver (hepatotoxicity) agent:
B. Ester Local Anesthetics hydrolyzed rapidly in the blood to ○ succinylcholine, 0.5–1 mg/kg IV)
inactive metabolites
TOXICITY
Duration of Action
Neurotoxicity
SHORT ACTING - Procaine ● HIGH CONCENTRATIONS:
(minor) - Chloroprocaine
○ all local anesthetics can be toxic to nerve tissue.
● Chloroprocaine & Lidocaine
INTERMEDIATE ACTING - Lidocaince
(used for minor surgery) - Mepivacaine ○ are the most neurotoxic when given as spinal
- Prilocaine anesthetic
○ Neurotoxic when used for spinal anesthesia,
LONG ACTING - Tetracaine producing so-called *transient radicular irritation (there
(used for major surgery) - Bupivacaine will be no movement)*
- Ropivacaine
- Dibucaine - 15-20 times Cardiovascular
more potent that procaine
● Depress strength of cardiac contraction, ECG changes &
cause hypotension
Uses of Local Anesthetics ● Block Cardiac Sodium Channels - depress abnormal
1. Since local anesthetics are membrane-stabilizing drugs cardiac pacemaker activity, excitability, and conduction.
they are used to treat patients with neuropathic pain ● At very high concentrations - blockade of calcium
syndromes. channels.
2. Parenteral (eg, intravenous lidocaine) and oral (eg, ○ severe hypotension.
mexiletine, tocainide) ● Cardiovascular collapse and death • usually occur only
3. Topical local anesthesia is often used for eye, ear, nose, after large doses of 0.75% bupivacaine (most cardiotoxic)
and throat procedures and for cosmetic surgery ● Lidocaine - used as antiarrythmic drug
○ Class I antiarrythmic drug
Two Major Forms of Toxicity ● The (S)-isomer, levobupivacaine
● direct neurotoxicity from the local effects of certain agents ○ appears to have a lower propensity for cardiovascular
administered around the cord or other major nerve trunks toxicity than the racemic mixture or the (R)- isomer
● Systemic effects and has recently been approved for clinical use.
○ local anesthetic agents are absorbed from the site of ● Ropivacaine - another newer local anesthetic
administration. ○ similar to bupivacaine
○ If blood levels rise too high, effects on several organ ○ available only as the (S)-stereoisomer, which has
systems may be observed. inherently less affinity for the cardiac sodium channel
○ Mas maganda gamitin pag may sakit sa puso yung
EFFECTS ON OTHER ORGANS patient

Central Nervous System Hematologic

● LOW DOSES: ● Prilocaine (> 10 mg/kg)


○ sleepiness, light headiness, visual & auditory ● Accumulation of Oxidizing Agent (o- toluidine) that convert
disturbances & restlessness Hgb to metHgb (cyanosis & chocolate-colored blood)
● HIGH CONCENTRATIONS: ● Treatment: IV administration Methylene Blue or Ascorbic
Acid

70
PCOL 211 - Pharmacology 1

● Articaine (Ubistesin)
Allergic Reactions
● Bupivacaine (MarcainE, Sensorcaine)
● Ester-type are metabolized to P-ABA derivatives (allergic ● Etidocaine (Duranest)
reactions) ● Lidocaine (Xylocaine)
● Can cause anaphylactic shock ● Mepivacaine (Carbocaine, Mepivastesin)
● p-aminobenzoic acid derivatives: ● Prilocaine (Citanest, EMLA)
○ Tetracaine ● Ropivacaine (Naropin)
○ Procaine
○ Chloroprocaine
■ metabolized by butyrylcholinesterase
GENERAL ANESTHETICS
ESTER LOCAL ANESTHETICS

GENERAL ANESTHETICS
Cocaine Used for topical anesthesia of mucous
membranes ● Reversible loss of consciousness and insensibility to
painful stimuli
Procaine Included in some formulations of penicillin
● Act on the brain, medulla and spinal cord
G to decrease the pain of intramuscular
injection Adjuncts

Chloroprocaine It has a more rapid metabolism than 1. Muscle relaxants - to facilitate intubation & ventilation
procaine 2. Anticholinergics - to decrease body secretions
3. Benzodiazepines & ultra short acting barbiturates - for
Dibucaine 15-20x more potent and 15x more toxic induction of anesthesia
than procaine 4. Opioids - post operative pain

Cocaine Five Primary E ects

● Cocaine's blockade of norepinephrine reuptake results in ● Neurophysiologic state produced by general anesthetics is
vasoconstriction and hypertension. characterized by five primary effects:
● It may also precipitate cardiac arrhythmias. A. Unconsciousness
● Lead to ischemia B. Skeletal muscle relaxation
● Chronic abusers: can cause ulceration of the mucous C. Analgesia
membrane and even damage to the nasal septum when D. Amnesia
"snorted." E. Inhibition of autonomic reflexes
● This vasoconstrictor property of cocaine can be used
Ideal anesthetic drug:
clinically to decrease bleeding from mucosal damage in
the nasopharynx 1. Induce rapid, smooth loss of consciousness
● Rapid and pleasant anesthetic induction and recovery
AMIDE LOCAL ANESTHETICS 2. Be rapidly reversible upon discontinuation
● Induce anesthesia smoothly and rapidly while allowing
Lidocaine The most widely used local anesthetic Also for prompt recovery after its administration is
used as an antiarrythmic drug discontinued
3. Possess a wide margin of safety.
EMLA Combination of lidocaine and prilocaine
Permits anesthetic penetration of the 4. Rapid changes in anesthetic depth
keratinized layer of skin, producing localized 5. Adequate relaxation of skeletal muscles
numbness. 6. Absence of toxic effects or other adverse properties in
normal doses
Lidocaine (Xylocaine) DEPTH OF ANESTHESIA
● rapid onset & long duration of action; ● Stage 1 – Analgesia Stage (Cortical stage)
● causes fever & hypersensitivity reactions more than ● Stage 2 – Excitement
procaine ● Stage 3 – Surgical anesthesia
● With anti-arrhythmic activity ● Stage 4 – Medullary depression
● Most neurotoxic when given as spinal anesthesia causing
transient radicular irritation. STAGE I STAGE II

Bupivacaine ● Analgesia w/out amnesia. ● Delirious


● for peripheral nerve block, caudal & epidural anesthesia ● Later in stage I, both ● Respiration is rapid, and
(amide) analgesia and amnesia heart rate and blood
are produced pressure increase
Brand Names
● Benzocaine (Americaine, United Home Burn Ointment)
● Tetracaine (Pontocaine)
● Procaine (Novocain)
● Chloroprocaine (Nesacaine)

71
PCOL 211 - Pharmacology 1

STAGE III STAGE IV

Slowing of respiration and Severe depression of the


heart rate and extends to CNS (Without circulatory
complete cessation of and respiratory support,
spontaneous respiration death would rapidly ensue)
(apnea).

Stage III (Surgical anesthesia)

● Divided into 4 planes:


○ Plane 1 - Roving eye balls. This plane ends when
eyes become fixed.
○ Plane 2 - Loss of corneal and laryngeal reflexes.
○ Plane 3 - Pupil starts dilating and light reflex is lost.
○ Plane 4 - Intercostal paralysis, shallow abdominal
respiration, dilated pupil.

Stage IV (Medullary depression) Induction


● This stage of anesthesia includes severe depression of the ● Factors to consider:
vasomotor center in the medulla as well as the respiratory 1. Blood: Gas Partition Coefficient
center. 2. Inspired Air Concentrations
● Without full circulatory and respiratory support, death 3. Breathing Rate
rapidly ensues.
Blood: Gas Partition Coe cient
● Cessation of breathing to failure of circulation and death.
● Pupil is widely dilated, muscles are totally flabby pulse is ● a useful index of solubility and defines the relative affinity
thready or imperceptible and BP is very low. of an anesthetic for the blood compared with that of
inspired gas.
Current Levels of Anesthesia
● Low Blood: Gas Partition Coefficient
1. Induction - This level encompasses the administration of 1. Low solubility in blood - reaches high arterial tensions
preoperative medications, adjunctive drugs to anesthesia, rapidly, which in turn results in rapid equilibration with
and the anesthetics required for surgery. the brain
2. Maintenance - This level begins when the patient has 2. Fast onset of action. Eg. Nitrous oxide
achieved a depth of anesthesia sufficient to allow the 3. Necessary for quick removal of anesthetic
surgery to begin and ends upon the completion of the ● In terms of the extent of hepatic metabolism, the rank
surgical procedure. order for the inhaled anesthetics:
3. Recovery - The recovery phase begins with the ○ halothane > enflurane > sevoflurane > isoflurane >
termination of the surgical procedure and continues desflurane > nitrous oxide
throughout the postoperative recovery period until the
patient is fully responsive to his or her environment. Oil: Gas Partition Coe cient

TYPES OF GENERAL ANESTHETICS ● High Oil: Gas Coefficient


● cause quick redistribution into adipose tissue
A. Inhaled Anesthetics ● Redistribution into fat causes the recovery of
B. Intravenous Anesthetics consciousness
C. Rectal (rare) ● Considerations: Weight and fat ratio of a patient
INHALED ANESTHETICS PHARMACODYNAMICS
● Volatile anesthetics ● Impaired excitatory transmission
○ Methoxyflurane, halothane, enflurane, isoflurane, ○ Ach, AMPA
desflurane, sevoflurane (amino-3-hydroxy-5-methyl-4-isoxazol-propionic acid),
○ have low vapor pressures and high boiling points ○ NMDA (N-methyl-D-aspartate)
● Gaseous anesthetics ● Potentiated inhibitory effects
○ nitrous oxide, xenon ○ Chloride channels (GABA and Glycine receptors)
○ Have high vapor pressures and low boiling points ○ Potassium channels
PHARMACOKINETICS MECHANISM OF ACTION
● Depth of anesthesia is dependent upon the concentration ● They increase the sensitivity GABAA receptors. This
of anesthetic in the central nervous system. causes a prolongation of the inhibitory chloride ion current
● Anesthetic potency is currently described by the MAC. after a pulse of GABA release. Postsynaptic neuronal
○ The alveolar concentration of an inhaled anesthetic excitability is thus diminished.
that prevents movement in 50% of patients in ● They block the excitatory postsynaptic current of the
response to a standardized stimulus. nicotinic receptors.

72
PCOL 211 - Pharmacology 1

○ Malignant hyperthermia
HISTORICAL INHALED ANESTHETICS
Enflurane
● Ether
○ Slow onset, recovery, explosive ● Rapid, smooth induction
● Chloroform ● Pungent odor
○ Slow onset, very toxic ● Causes electroencephalographic (EEG) patterns
● Cyclopropane consistent with electrical seizure activity,
○ Fast onset, but very explosive ● CI: seizure disorders

INHALATIONAL ANESTHETICS Isoflurane

● Rapid, smooth induction


Nitrous Oxide
● Pungent odor
● Gas at RT, liquid under pressure in metal cylinders ● structural isomer of enflurane
● This is the least toxic anesthetic ● There have been no reports of seizures
● Also the least potent ● Bronchoirritating, laryngospasm
● rapid onset of, and recovery from its effects ● Coronary steal syndrome
● intermittent dosing for analgesia during dental procedures ● DISADVATAGE:
and the first stage of labor ○ May cause “steal” phenomenon
● Low potency, quick recovery ■ Coronary “steal” phenomenon - diversion of blood
● Needs other agents for surgical anesthesia flow via alternate routes or via reversed flow
● Weak anesthetic, powerful analgesic
Desflurane
● SIDE EFFECTS:
○ Hypoxia ● Very fast onset and offset
○ Inhibits vitamin B-12 metabolism ● Very pungent
○ Hysterical laughing ○ breath holding, coughing, and laryngeal spasm
● DISADVANTAGES: ● Not recommended for induction of anesthesia in children
○ Weak agent, no muscle relaxant activity
Sevoflurane
○ Danger: Hypoxia (if used alone in large amounts)
○ Bone marrow depression (megoblastic anemia) ● Rapid induction
● Second Gas Effect ● Low pungency
● Breakdown by CO2 absorbents generates heat and has
Second Gas E ect
resulted in sporadic operating room fires.
● Causes an increase in the concentration of the other gases
INTRAVENOUS ANESTHETICS
● Reduce the risk of myocardial depression at a given depth
of anesthesia compared to the more potent anesthetic ● Several drugs are used intravenously, alone or in
given alone combination with other drugs, to achieve an anesthetic
● Reduce the anesthetic requirement for the more potent state (as components of balanced anesthesia) or to sedate
anesthetic patients in intensive care units who must be mechanically
ventilated
Xenon
○ Thiopental
● An inert gas ○ Propofol
● Nonflammable and nonexplosive ○ Etomidate
● A potent analgesic ○ Ketamine
○ Opioids
Methoxyflurane
1. Barbiturates (Thiopental, Methohexital)
● Diffuses into fatty tissue
2. Benzodiazepines (Midazolam, Diazepam)
● Metabolism:
3. Opioid Analgesics (Morphine, Fentanyl, Sufentanil,
○ Methoxyflurane → Fluoride Oxalic acid
Alfentanil, Remifentanil);
● Most potent of all inhalational anesthetics
4. Propofol;
● DISADVANTAGE:
5. Ketamine; and
○ Metabolized to a great extent to fluoride - renal
6. Miscellaneous Drugs (Droperidol, Etomidate,
dysfunction
Dexmedetomidine)
Halothane
1. Ultra-Short Acting Barbiturates
● Halogen substituted ethane
● Fast induction of anesthesia and fast recovery potent ● Thiopental, Thiamylal, Methohexital
anesthetic and relatively weak analgesic ● Use: Pre-anesthetic agent for induction of anesthesia
● Halothane → Trifluoroethanol ● Onset of action: 10-20 seconds
● ADR: ● Peak: 40 seconds
○ Hepatic necrosis - there is no specific treatment for ● Recovery: 30 minutes
halothan hepatitis, and liver transplantation may ● Dose: 50mg test dose given with moderate rapidity
ultimatey me required ● Toxicity: Respiratory and circulatory depression

73
PCOL 211 - Pharmacology 1

2. Benzodiazepines Etomidate

● Diazepam, Lorazepam, Midazolam ● Miscellaneous


○ MIdazolam is the only water-soluble form ● Potent ultra short-acting non-barbiturate hypnotic without
● Primary Use: as premedication analgesic property
● MOA: Potentiate action of GABA ● Dose of 0.3 mg/kg will induce sleep that lasts for
● Cause anterograde amnesia approximately 5 minutes
● Toxicity: Antagonized by Flumazenil ● Structure similar to ketoconazole
● Rapid induction
3. Opioid Analgesics
● Metabolized to carboxylic acid
● Mainly used as adjuncts ● Insoluble in water, formulated in 35% propylene glycol
● Serious effect: Depression of steroidgenesis
Drugs Dose Duration of
● SIDE EFFECTS:
Analgesia (min)
1. Myoclonic activity
Morphine 1 - 2 mg 90 2. Nausea and vomiting (50%)
3. Cortisol suppression
Meperidine 10 - 25 mg 45 4. Pain on injection
● MOA: GABA-mimetic action
Fentanyl 0.05 - 0.1 mg 30
Ketamine
Sufentanil 0.005 - 0.01 mg 15 ● Acts as a noncompetitive antagonist at the glutamate,
NMDA receptor, a nonspecific ion channel receptor.
Alfentanil 0.15 - 0.3 mg 20
○ believed to bind to the PCP and block the calcium ion
flow in the channel.
Thiopental
● Norketamine - active metabolite
● Prototype ● Structure similar to PCP
● Highly lipid soluble ● Analgesic
● MOA: Act at GABA receptors (inhibitory), potentiate ● Dissociative anesthesia
endogenous GABA activity at the receptor, direct effect on ○ Cataleptic appearance, eyes open, reflexes intact,
Cl- channel at higher concentrations. purposeless but coordinated movements
○ Rapid Onset, short-acting ● Stimulates sympathetic nervous system
○ Others: Thiamylal, Methohexital ● Produces “emergence” phenomena
○ Build-up in adipose tissue ● Psychomimetic – “emergence reactions”
● SIDE EFFECTS: ○ vivid dreaming extracorporeal (floating "out-of-body")
1. Hypotension experience misperceptions, misinterpretations,
2. Airway obstruction illusions
3. Apnea ○ may be associated with euphoria, excitement,
confusion, fear
Propofol ● Used in low doses for outpatient anesthesia in combination
with propofol and in children undergoing painful
● Notable effects:
procedures (eg, dressing changes for burns).
○ emulsion -
● MOA: Blockade of the membrane effects of the excitatory
○ eduction in cerebral blood flow
neurotransmitter glutamic acid at the NMDA (N-methyl D-
○ respiratory depressant
aspartate) receptor
○ may increase arrhythmogenicity with epinephrine but
by itself does not cause arrhythmias or MI ● Sensation of dissociation in 15 seconds
● Anti emetic effect ● Unconsciousness in 30 seconds
● Rapid onset and short duration of action ● Duration of unconsciousness = 10-15 minutes
● A sedative–hypnotic used for the induction and ● Duration of analgesia = 40 minutes
● Duration of amnesia = 1-2 hours
maintenance of anesthesia or sedation
● An alkyl phenol
● Originally formulated in egg lecithin emulsion “Emergence” phenomena
○ anaphylactoid reactions ● Associated with postoperative disorientation, sensory and
○ Current formulation: 1% propofol in 10% soybean oil, perceptual illusions, and vivid dreams
2.25% glycerol, 1.2% egg phosphatide
● SIDE EFFECTS: Dexmedetomidine
1. Pain on injection ● Alpha 2 adrenergic agonist
2. Hypotension ● Hypnosis is due to stimulation of a2 receptors in the locus
3. Apnea coeruleus
Fospropofol ● Use for short term sedation of intubated and ventillated
patients in ICU
● Prodrug
● Approved for sedation during monitored anesthesia care

74
PCOL 211 - Pharmacology 1

Opioids
ANALGESICS
● Fentanyl, sufentanil, alfentanil, remifentanyl or morphine
● Excellent Analgesic
● SIDE EFFECTS: PAIN
○ nausea, chest wall rigidity, seizures, constipation,
● An unpleasant sensory and emotional experience,
urinary retention
associated with, or described in terms of actual, or
Receptor-Mediated E ects of Narcotics potential tissue damage.
● Aristotle, who considered pain a feeling and a passion of
Receptor E ects the soul, advanced the punishment theory.

PATHOPHYSIOLOGY
(mu) Analgesia, respiratory depression, euphoria,
physical dependence A. Peripheral Stimulation
● The first step leading to pain is the activation of
(kappa) Analgesia, respiratory depression, sedation, nociceptors
miosis B. Neuronal Stimulation
● Afferent, nociceptive pain fibers synapse at the dorsal
(sigma) Dysphoria, hallucinations, tachypnea, horn of the spinal cord along with other
tachycardia non-pain-transmitting neurons.
C. Spinal Cord Transmission
BALANCED ANESTHESIA ● Pain reaches brain through ascending fiber tracts.
● The spinothalamic tract is divided into two ascending
● Combination of IV andinhaled drugs pathways:
● Neuroleptics and Opioids 1. Lateral Pathway
○ Augments the action of GA 2. Ventral Pathway
● Skeletal muscle relaxants
● Anticholinergics TYPES OF PAIN
● Use specific drugs for each component: ● Based on ORIGIN
○ Sensory 1. Neuropathic Pain
■ N2O, opioids, ketamine for analgesia 2. Nociceptive Pain
● Based on DURATION
1. Acute Pain
○ Cognitive: 2. Chronic Pain
■ Produce amnesia, and preferably 3. Breakthrough
unconsciousness, with N2O, .25-.5 MAC of an
According to the International Association for the Study
inhaled agent, or an IV hypnotic (propofol,
of Pain (IASP), pain can be classified according to specific
midazolam, diazepam, thiopental) characteristics:
○ Motor 1. Region of the body involved
■ Muscle relaxants as needed 2. System whose dysfunction may be causing the pain
3. Duration and pattern of occurrence
1. Monitored Anesthesia Care (MAC) 4. Intensity and time since onset
5. Etiology
● For minor superficial surgery or for invasive diagnostic
procedure
● Sedative + Local Anesthetic Nociceptive Pain
● Medications used during MAC include + LA: ● Caused by activity in neural pathways in response to
○ Midazolam (Versed) potentially tissue-damaging stimuli
○ Fentanyl ○ Postoperative pain
○ Propofol (Diprivan) ○ Mechanical low back pain
2. For extensive procedures ○ Sports/exercise injuries
○ Sickle cell crisis
1. Preoperative drug: BZD ○ Arthritis
2. Induction: IV agent (Thiopental or Propofol)
3. Maintenance: combination of inhaled (nitrous oxide, vol Mixed Type
liquids) or IV drugs (propofol, opioid) ● Caused by a combination of both primary injury or
secondary effects

Neuropathic Pain

● Initiated or caused by primary lesion or dysfunction in the


nervous system
○ Postherpetic neuralgia
○ Neuropathic low back pain
○ CRPS

75
PCOL 211 - Pharmacology 1

○ Central poststroke pain


○ Trigeminal neuralgia
○ Distal polyneuropathy (eg, diabetic, HIV)

Pain Management
● ASSESMENT
○ Complaint of the patient
○ Clinical status
○ Drugs, Allergies & analgesic response
● MANAGEMENT
○ Primary pain
○ Acute pain management
○ Chronic pain management

ANALGESICS
● Analgesics are common pain relievers.
● Many analgesics also have antipyretic properties as well.
They can be used to reduce fever. P-AMINOPHENOLS
● Some analgesics are also anti inflammatory drugs as wel
● ACETAMINOPHEN
● It is rapidly and completely absorbed from the
gastrointestinal tract.
● Acetaminophen is metabolized by the liver.
● It has both analgesic and antipyretic effects.
● If large doses are ingested, an intermediate metabolite,
NAPQI, is formed.
● USES:
1. dull pain and hyperpyrexia
2. choice for children with viral infections or chicken pox.
3. 1st line agent for osteoarthritis
MILD ANALGESICS ● ADVERSE EFFECTS
● Inflammation is a protective response to tissue injury. 1. Hepatotoxicity - Can occur after the ingestion of a
○ Inflammation is triggered by the release of chemical single toxic dose (20-25 gm) or after long term use of
mediators from injured tissues and migrating cells. therapeutic doses.
○ Specific chemical mediators include histamine, 5-HT, 2. Nephrotoxicity
PGs, LTs, bradykinin, interleukin-1 3. SJS (dermatologic A/E)
● Mild analgesics include over-the-counter pain relievers and NSAIDs
fever depressants.
● Mild analgesics includes: A. Non-selective COX inhibitors
○ Para-aminophenols B. COX-2 selective inhibitors
○ NSAIDs
● Effects: antipyretic, analgesic and anti-inflamatory NON SELECTIVE NSAIDs
activities.
SALICYLATES
Antipyretic e ect
1. Pathogen (pyrogen release)
ASPIRIN ● Derivative of salicyclic acid
2. Hypothalamus (PGE2 synthesis and release) ● Blocks COX irreversibly
3. Heat production (fever) ● With anti-pyretic, antiinflammatoty,
analgesic and antiplatelet effect
Analgesic e ect
1. Injured or inflammatory tissue Uses ● 0.25 - 0.5 g
2. Prostaglandins & autacoid release ○ Hyperpyrexia ü Dull pain
● 3-4g
3. Pain receptors (pain)
○ Rheumatic fever and rheumatoid
Anti-inflammatory E ect arthritis (first-line drugs)
● 100 - 325 mg
1. Prostaglandins & autacoid release ○ prevention of thromboembolism,
2. Vasodilation (Increase permeability) stroke, myocardial infarction
3. Edema & swelling
Adverse 1. GI reaction: epigastric distress, nausea,
effects vomiting, gastric ulceration and bleeding.
2. Hepatotoxic
3. Prolonging bleeding time
4. Hypersensitivity or allergy
5. Reye’s syndrome

76
PCOL 211 - Pharmacology 1

6. Salicylism INDOLE DERIVATIVE


7. Can decrease GFR
Indomethacin ● used to accelerate closure of patent
Reye’s Syndrome ductus arteriosus
● used in Gingival inflammation
● A potentially fatal disease that causes numerous ● used in conjunctival inflammation
detrimental effects to many organs, especially the brain ● DI: Probenecid prolongs
and liver, as well as causing hypoglycemia. indomethacin’s half-life by inhibiting
both renal and biliary clearance
OTHER SALICYLATES
OXICAM DERIVATIVE
1. Mg choline salicylate ● less effective analgesics than
2. Na salicylate aspirin
Piroxicam ● used for the usual rheumatic
3. Salicyl salicylate ● do not inhibit platelet
Meloxicam indications
aggregation
● Peptic ulcer risk is as much as 9.5
times
DIFLUNISAL ● higher with piroxicam than with other
NSAIDs
Derived from USES:
salicylic acid ● Used in In rheumatoid FENAMATES
● 2% diflunisal oral ointment is a clinically
useful analgesic for painful oral lesions.
Mefenamic acid ● Analgesic only
Flufenamic acid ● Should not be used more than 5
PHENYLACETIC ACID DERIVATIVES Meclofenamic acid days (acute mgt. of pain)
● CI: <14 years old
1. Diclofenac ● Gastrointestinal ulceration may occur
2. Sulindac less frequently PYRAZOLONE DERIVATIVES
3. Alcofenac ● PREPARATIONS:
○ Diclofenac + Misoprostol
Phenylbutazone ● Powerful analgesics
○ Diclofenac + Omeprazole
Dipyrone ● Good anti-inflammatory effects
● 0.1% ophthalmic preparation -
Sulfinpyrazone ● A/Es:
prevention of postoperative ophthalmic
○ Hematotoxic
inflammation
○ Nephrotoxic
● A topical gel (3%) - solar keratoses
○ Hepatotoxic
○ Acute tubular necrosis
ACETIC ACID DERIVATIVES ○ Anasarca

Etodolac ● Racemic acetic acid derivative NABUMETONE


Tolmetin ● Induce uric acid stone formation
Only nonacid ● converted to the active acetic acid
Ketorolac ● mainly as an analgesic, not as an NSAID in current derivative in the body
antiinflammatory drug use ● given as a ketone prodrug that
● replace morphine in some situations resembles naproxen
involving mild to moderate postsurgical pain ● Associated with pseudoporphyria
and photosensitivity
PROPIONIC ACID DERIVATIVES
SULINDAC
Ibuprofen ● 2400 mg ibuprofen is equivalent to 4 g
of aspirin in antiinflammatory effect ● Sulfoxide ● Used in rheumatic disease
● effective in closing patent ductus prodrug ● suppresses familial intestinal
arteriosus in preterm infants ● It is reversibly polyposis
● Used in primary knee osteoarthritis. metabolized to ● may inhibit the development of
the active sulfide colon, breast, and prostate cancer
1. Ketoprofen ● Oxaprozin has a very long half-life metabolite ● Adverse effects:
2. Flurbiprofen ● Oxaprozin is more useful in gout than ○ SJS
3. Oxaprozin some other NSAIDs. ○ Thrombocytopenia
○ Agranulocytosis
NAPHTHYLPROPIONIC ACID DERIVATIVE ○ Nephrotic syndrome

Naproxen ● only NSAID presently marketed as a COX-2 INHIBITORS


single enantiomer
● Effective for the usual rheumatologic
“COXIBS”
indications
● Associated with pseudoporphyria and
photosensitivity 1. Celecoxib ● a sulfonamide – causes rashes
2. Meloxicam ● lesser GI effect
3. Etoricoxib ● Adverse effect:
4. Rofecoxib ○ cardiovascular thrombotic

77
PCOL 211 - Pharmacology 1

5. Valdecoxib events etorphine, pentazocine, tramadol,


loperamide, diphenoxylate
How COX-2 Inhibitors Work
BASED ON POTENCY
1. PAIN RELIEF: COX-2 is an enzyme that is activated at
sites of injury. It manufactures hormone-like substances
1. Strong agonist morphine,heroin,meperidine,
called prostaglandins, which trigger painful inflammation methadone,fentanyl
2. COX-2 inhibitors are drugs designed to block the activity of
the COX-2 enzyme and relieve pain 2. Moderate agonist codeine, propoxyphene,
3. HEART RISK: Prostacyclin, a prostaglandin produced by oxycodone, hydrocodone,
COX-2 in blood vessel walls, opens blood vessels and buprenorphine
prevents platelets from clumping
4. Researchers believe that when COX-2 is blocked, 3. Mixed pentazocine, nalbuphine,
agonist-antagonist butorphanol
prostacyclin may also be suppressed, allowing platelets to
stick together and blood vessels to constric, which can
4. Antagonist naloxone, naltrexone
lead to heart attacks and strokes

STRONG ANALGESICS BASED ON STRUCTURE

● The endogenous opioids:


1. Phenanthrenes morphine, heroin,
○ Dynorphins hydromorphone, oxymorphone,
○ Endomorphins codeine, hydrocodone,
○ Enkephalins oxycodone, nalbuphine,
○ Endorphins buprenorphine
○ Nociceptin
● Opioid receptors - mu, delta, kappa, epsilon, sigma 2. Phenylheptylamines Methadone, propoxyphene

LIGANDS 3. Phenylpiperidines fentanyl , sufentanil, alfentanil,


↓ remifentanil, diphenoxylate,
Opiod Receptors loperamide

Gi linked receptor 4. Morphinans Levorphanol, butorphanol

Inhibiting adenylate cyclase 5. Benzomorphans Pentazocine

Increasing potassium ion efflux or reducing calcium ion influx 6. Miscellaneous Tramadol
Impeding neuronal firing and transmitter release
Indications
History
1. Pain:
● Opium is extracted from poppy seeds ● Before and during surgery
● Used for thousands of years to produce: ● Before and during invasive diagnostic procedures
1. Euphoria ● During labor and delivery
2. Analgesia 2. Acute pulmonary edema
3. Sedation 3. Treating severe, nonproductive cough
4. Relief from diarrhea
Contraindications
5. Cough suppression
● Used medicinally and recreationally from early Greek and 1. Respiratory depression
Roman times 2. Chronic lung disease
● Opium and laudanum (opium combined with alcohol) were 3. Chronic liver or kidney disease
used to treat almost all known diseases 4. BPH
5. Increased intracranial pressure
● Morpheus is the Greek god of dreams and sleep.
6. Hypersensitivity reactions
● Friedrich Wilhelm Adam Sertürner, a German
pharmacist, isolated Morphine from opium, in 1805. Adverse E ects
1. Common side effects
CLASSIFICATION OF OPIOIDS
● nausea, vomiting, constipation ürespiratory
depression, hypotension, üdysphoria, acute urine
BASED ON ORIGIN
retention.
2. Tolerance and physical dependence withdrawal symptoms:
1. Natural Morphine, codeine, thebaine
● autonomic, motor and psychological responses
(insomania, dysphoria, headache, sweating, vomiting,
2. Semi-synthetic Apomorphine, hydrocodone,
oxycodone, hydromorphone, diarrhea, tremor, collapse).
oxymorphone, heroin

3. Synthetic meperidine, methadone, fentanyl,

78
PCOL 211 - Pharmacology 1

5. Rectal route for patients who cannot take oral narcotics


Drug Interactions
6. Transdermal route is an alternative for patients with
chronic pain and cannot afford to take oral narcotics
Drug Group Interaction with Opioids
CAPSAICIN
Sedative-hypnotics Increased central nervous system
depression, particularly respiratory ● From cayenne peppers; topical indicated for post-herpetic
depression neuralgia, neuropathic pain = Substance P

Antipsychotic Increased sedation. Variable effects


tranquilizers on respiratory depression.
Accentuation of cardiovascular effects DRUGS FOR MOOD DISORDERS
(antimuscarinic and α-blocking
actions)
MOOD
Monoamine Relative contraindication to all opioid ● Defined as pervasive and sustained emotion that in
oxidase inhibitors analgesics because of the high
extreme, markedly affects the person’s perception of the
incidence of hyperpyrexic coma;
hypertension has also been reported world and the ability to adequately function in the society.
○ Mood is a conscious state of mind or predominant
OPIOIDS emotion that we feel especially when we encounter
some people and some situations. Either were in a
Morphine Very strong pain reliever but also very good mood or bad mood
addictive (2nd to heroin) MOOD DISORDERS
Codeine ● Most commonly used strong analgesic A. Depression (MDD)
● 3-methyl ether of morphine B. Mania & Hypomania
● analgesic potency is 1/12 of morphine, C. BIipolar
cough depressant potency is 1/4 of ● Sub-types:
morphine
○ Dysthymic disorder
Heroin ● Sold as cough suppressant and pain ○ Cyclothymic disorder
killer AFFECTIVE DISORDER
● Incredibly addictive
● Diacetylmorphine, diamorphine ● Mental illnesses characterized by pathological changes in
MOOD
Mepiridine Loparamide (derivative) ○ Unipolar disorders - Depression Mania
○ Bipolar Disorder - Manic-depressive illness “cycling
Methadone ● Help in the withdrawal for Heroin addicts mood’
● Weaker analgesic

Fenantyl ● Analgesic effect is 80-100 times as


effective as morphine
● Short duration of action

Opioid Antagonists
1. Partial Antagonists
● To precipitate a withdrawal syndrome in opioid addicts
● Nalorphine
● Butorphanol
● Buprenorphine
● Pentazocine
● Nalbuphine
2. Full Antagonists
● Naloxone
○ May reverse the acute poisoning effects of opioid
agonists and precipitate a withdrawal syndrome in
opioid addicts
● Naltrexone

Routes of Administration
1. Oral administration - preferred particularly for patients with
chronic, stable pain.
2. IM and SQ = used in the post operative period
3. IV for stable for chronic pain
4. Epidural and intrathecal administrations are used for acute
postoperative pain.

79
PCOL 211 - Pharmacology 1

● Loss of neurotrophic support also affects the medial frontal


MAJOR DEPRESSIVE DISORDER (MDD)
cortex and anterior cingulate
● Characterized by: ○ Plays a role in the integration of emotional stimuli and
○ Depressed mood most of the time for at least 2 weeks attention functions
○ Loss of interest or pleasure in most activities ○ Involved in memory, learning and emotion
○ Disturbances in sleep and appetite ● BDNF (Brain-derived neurotrophic factor)
○ Deficits in cognition and energy ● Critical in regulation of neural plasticity, resilience and
○ Thoughts of guilt, worthlessness, and suicide neurogenesis
● Medical conditions related to MDD ● Activates tyrosine kinase receptor B
○ Chronic pain ○ Influencing neuronal survival and growth effects
○ Coronary artery disease
○ Diabetes
○ Stroke

Symptoms of Major Depressive Disorder


● Chronic emptiness
● Unyielding sadness
● Hopelessness
● Restlessness
● Fatigue
● No interest in hobbies

EPIDEMIOLOGY
● Depression is two or three times as frequent in females
than in males.
● It can occur at any age, but major incidence What to do with BDNF levels?
● Occurs at ages between 25-44 years old. ● ↑ BDNF how?
● Depressive disorders and suicide tend to cluster in 1. DIRECT INFUSION to the midbrain, hippocampus and
families. lateral ventricles
ETIOLOGY 2. Electroconvulsive therapy

● Alterations in brain monoamine neurotransmitters: Monoamine Hypothesis


norepinephrine, serotonin and dopamine play a major role ● Deficient in:
in depressive disorders ○ Serotonin
● Other factors include stressful events,medical illness. ○ Norepinephrine
PATHOPHYSIOLOGY ○ Dopamine
● In the cortical and limbic system
● Neurotrophic Hypothesis ● Altered number of receptors:
● Monoamine Hypothesis ○ 5HT1A and 5HT2C
● Neuroendocrine Factors ○ Alpha 2
● Basal ganglia - control of movements, learning, habit, ● Reduced amounts of metabolites in the CSF
cognition, and emotion ● 5-hydroxyindoleacetic acid
● Thalamus - regulation of sleep, consciousness, and ● Associated with violent and impulsive behavior including
alertness violent suicide attempts
● Hippocampus - memory, navigation
● Amygdala - memory, decision-making and emotional
responses
● Hypothalamus - controls body temperature, hunger,
fatigue, sleep

HYPOTHESIS OF DEPRESSION
● Depression is associated with the loss of neurotrophic
support from nerve growth factors such as BDNF
● Suggests that depression is related to a deficiency in the
amount or function of cortical and limbic 5-HT, NE and
dopamine.

Neurotrophic Hypothesis
● Loss of neurotrophic support
○ Contributes to atrophic structural changes in the
hippocampus
○ Important in contextual memory and for regulation of
hypothalamic-pituitaryadrenal (HPA) axis

80
PCOL 211 - Pharmacology 1

Other Clinical Uses of Anti-Depressants


1. Anxiety and eating disorder
2. Pain disorder
3. Smoking Cessation
4. Pre-menstrual dysphoric disorder

SELECTIVE SEROTONIN REUPTAKE INHIBITORS


(SSRI)
Neuroendocrine Factors ● MOA: Inhibition of the serotonin transporter (SERT)
● More modern and safe antidepressants
● Linked to Hormonal abnormalities
● Fluoxetine - 1st drug available in 1988
● DEPRESSION
● Principal mechanism of action:
1. Linked to Hormonal abnormalities - ↑ Cortisol
○ selective inhibition of 5-HT (serotonin) reuptake
levels and Corticotropin Releasing Hormone
(SERT)
Non-suppression of ACTH release
○ gradual complex changes in the density and/or
2. Thyroid dysregulation
sensitivity
3. Sex Steroids - estrogen and testosterone deficiency
○ both autoreceptors (5-HT1A) and postsynaptic
Therapy of Depression receptors (important subtype 5-HT2A )
● Non-pharmacological treatment ● Other indications of SSRI
○ Psychotherapy (cognitive behavior therapy ) ○ Anxiety disorders
○ Electroconvulsive therapy (ECT) ■ generalized anxiety, panic disorder, social anxiety
disorder, obsessive-compulsive disorder
Treatment Phase Duration Goal ○ bulimia nervosa, gambling, drug withdrawal
○ PMDD(Premenstrual Dysphoric disorder)
Acute 6 weeks Resolve ● Racemic Forms
symptoms
○ Fluoxetine (Prozac)
Continuation 6-9 months Prevent relapse ○ Sertraline (Zoloft)
○ Citalopram (Celexa)
Maintenance 3-5 years of Prevent ○ Escitalopram (Lexapro) - S-enantiomer of Citalopram
lifelong recurrence in high ● Not optically active
risk patients ○ Paroxetine (Paxil)
○ Fluvoxamine (Luvox, Faverin)
● Pharmacotherapy
● Selective Serotonin Re-uptake Inhibitors (SSRI) ● Fluoxetine → norfluoxetine
● Tricyclic antidepressants (TCA) ● Fluoxetine and paroxetine are potent inhibitors of
● Monoamine oxidase inhibitors (MAOI) CYP2D6
● Other and atypical antidepressant ● Fluvoxamine is an inhibitor of CYP3A4
● Serotonin-2 Antagonists/Reuptake Inhibitors (SARI)
Pharmacokinetics
● Serotonin and NE Reuptake Inhibitors (SNRI)
● NE and Dopamine Reuptake Inhibitors (NDRI) ● Good absorption after oral administration
● Noradrenaline Reuptake Inhibitors (NaRI) ● Important biotransformation in the liver
● Noradrenergic / Specific Serotonergic Antidepressants ○ Norfluoxetine
(NaSSA) ○ (metabolite of Fluoxetine )
○ With longest half life of all SSRIs
ANTIDEPRESSANTS
● Long half-lives of elimination(s)
● Selective Serotonin Reuptake Inhibitors ○ fluoxetine (T1/2 = 50h) + active metabolite (T1/2 =
● Serotonin-Norpepinephrine Reptake Inhibitors 240h
● 5-HT 2 Antagonists ● CYP450 Inhibitors
● Tetracyclic and Unicyclic Antidepressants ○ Fluoxetine, Paroxetine = 2d6
● Monoamine Oxidase Inhibitors ○ Fluvoxamine = 3a4

Drug Interaction
● based on plasma protein binding and CYP blockade
○ increased effect of co-administered TCA, βblockers,
benzodiazepines etc.

81
PCOL 211 - Pharmacology 1

● Milnacipran
Adverse e ects
Venlafaxine
● Relative improvement to other antidepressants (mostly
mild) ● pharmacodynamics like in TCA
● GIT: nausea, vomiting, abdominal cramps, diarrhea ● improved profile of adverse reactions
● Increased in headaches and insomnia or hypersomnia ● Very effective, better remission rate than SSRI
● Sexual dysfunctions (loss of libido, erectile dysfunction, ● Adverse reactions: nausea, vertigo (both frequent and may
delayed orgasm, or diminished arousal) improve), hypertension, manic reactions
● Restlessness (akathisia) ● Used in: depression and depression with anxiety,
● Discontinuation syndrome (dizziness, paresthesias) generalized anxiety disorders, social phobias, neuropathic
● Anxiety - an increase in anxiety or agitation during early pain
treatment
Duloxetine
● Insomnia and fatigue
● Serotonin syndrome upon intoxication or drug interactions ● first antidepressant to secure FDA approval for the
treatment of pain associated with diabetic neuropathy and
SEROTONIN AND NE REUPTAKE INHIBITORS (SNRI)
fibromyalgia
● Two classes of antidepressants act as combined serotonin
Adverse E ects
and norepinephrine reuptake inhibitors:
1. Selective serotonin-norepinephrine reuptake inhibitors ● Duloxetine - with hepatic toxicity in patients with a history
(SNRIs) of liver damage.
2. Tricyclic antidepressants (TCAs) ● SNRIs - with discontinuation syndrome
● MOA: Binds and inhibits the SERT and NET.
TRICYCLIC ANTIDEPRESSANTS (TCAs)
● Bicyclic
○ Venlafaxine ● Chemical structure: three ring nucleus
○ Desvenlafaxine ○ lipophilic nature
● Duloxetine - Three ring structure ● Originally developed as antipsychotics (1949), but were
● Milnacipran – for fibromyalgia found to have no effect in this indication.
● INDICATIONS ● Principal mechanism of action:
○ Major depression ○ Blockade of reuptake of monoamine neurotransmitters
○ Pain disorders (neuropathies, fibromyalgia) noradrenaline (NA) and serotonin (5-HT)
○ GAD ○ By competition for binding site of the carrier protein
○ Stress urinary incontinence (NET and SERT)
○ Vasomotor symptoms of menopause ○ Other action:
■ Blockade of H1-receptor, α-receptors,
Fibromyalgia M-receptors
● CNS: Chronic headeaches, sleep disorders, dizziness,
TCAs
cognitive impairment, memory impairment, anxiety,
depression ● Imipramine (Tofranil)
● Eyes: Vision problems ● Desipramine (Norpramin)
● Join of jaw: Jaw dysfunction ● Amitriptyline
● Joints: Morning stiffness ● Clomipramine (Anafranil)
● Chest: Chest pain ● Nortryptiline (Pamelora)
● Skin: Various problems ● Amoxapine
● Muscle: Myofascial pain, fatigue, twitches ● Doxepine
● Stomach: Nausea ● Dosulepine (Prothiaden)
● Reproductive: Dysmenorrhea
Most Important TCAs
● Urinary: Problems urinating
● Systematic symptoms: Pain, weight gain, cold symptoms, ● Imipramine (A Representative)
multiple chemical sensitivity ● Desipramine
○ Demethylated form, the active oetabolite of
Generalized Anxiety Disorder (GAD) Imipramine
● Excessive anxiety and worry ● Amitriptyline
● Increased muscle aches or soreness ● Nortriptyline
● Impaired concentration ○ Demethylated form, the active metabolite of
● Irritability Amitriptyline)
● Difficulty sleeping ● Clinical use and efficacy is relatively similar within the
● Restlessness group
● Fatigue ● The more significant difference is in their adverse effects

Selective Serotonin-Norepinephrine Reuptake Uses of TCAs


Inhibitors ● Depression that is unresponsive to SSRIs and SNRIs
● Venlafaxine ● Pain conditions
● Duloxetine ● Enuresis

82
PCOL 211 - Pharmacology 1

● Insomnia
MONOAMINE OXIDASE (MAO)
PHARMACOKINETICS
● In humans there are two types of MAO: MAO-A and
● Administered orally MAO-B
○ rapid absorption, extensive first pass effect → low and ● Both are found in neurons
inconsistent BAV ● Outside the central nervous system:
● Strong binding ○ MAO-A is found in the liver, gastrointestinal tract, and
○ to plasma proteins (90-95% bound). placenta.
○ in tissues + wide distribution (high lipophilicity) = large ○ MAO-B is mostly found in blood platelets.
distribution volumes (ineffectiveness of dialysis in
MONOAMINE OXIDASE INHIBITORS (MAOI)
acute intoxications)
● Biotransformation ● The first compounds (iproniazide derivatives) - originally
○ in the liver (CYP450, N-demethylation and tricyclic developed as antimycobacterial drugs by chemical
ring hydroxylation) modification of isoniazid molecule (1950s).
○ Most of these metabolites are active! CYP450 ● Hydrazine derivatives
polymorphisms ○ Phenelzine (Nardil)
○ Glucuronidation → inactive metabolites excreted in ○ Isocarboxazid
the urine. ● Non-hydrazines
● Elimination half-lives ○ Tranylcypromine (Parnate)
○ generally LONG (T1/2 =10-80h). ○ Selegiline (Deprenyl)
○ Elderly patients – even longer T1/2 ⇒ risk of ○ Moclobemide (Aurorix)
accumulation.
MAO-A MAO-B
ADVERSE EFFECTS
Present in both dopamine Is found primarily in
● TCA are effective antidepressants. Their use is and norepinephrine neurons serotonergic and
complicated by numerous troublesome adverse effects (brain, gut, placenta, and histaminergic neurons
○ Anticholinergic (atropine-like) due to M-blockade liver) (brain, liver, and platelets)
■ Dry mouth, blurred vision
■ Constipation, urinary retention (more in MAOI DRUGS
amitriptyline, less in imipramine) ● Irreversible non-selective inhibitors (hydrazides)
■ Palpitations, tachycardia ● Long lasting inhibition (up to 1-2 weeks) despite of the
○ Postural (orthostatic) hypotension + reflex tachycardia elimination rate of a drug
■ α-blockade of adrenergic transmission (frequent ○ Phenelzine
in elderly) ○ Tranylcypromine
○ Sedation - H1-blockade ● Reversible Inhibitors of MAO-A (RIMA)
■ drowsiness, difficulty in concentration ○ Moclobemide
(amitriptyline) ● Note: Reversible inhibitors of MAO-B (e.g. selegiline) are
○ Sexual dysfunction (loss of libido, impaired erection) used in the treatment of Parkinson's disease.
○ Arrythmogenic
○ Weight gain PRINCIPAL MECHANISM OF ACTION
○ Sedation ● Inhibition of intracellular enzyme MAO in CNS neurons (=
ACUTE INTOXICATION WITH TCA decrease in degradation of catecholamines and serotonin).
● Antidepressant action - MAO-A enzyme isoform inhibition
● Very dangerous and relatively frequent ○ increased cytoplasmic pool of monoamines leading
○ patients with depression often have suicidal among other(s) to spontaneous leakage of
tendencies monoamines.
● Precautions ● When given to normal non-depressed subjects they
○ patient education - remind him/her that 2-4 week delay increase motor activity and cause euphoria + excitements
in the effect is anticipated and that it is NOT a failure ⇒ risk of abuse!
of medication)
● TCA - low therapeutic index Indications
● Target systems – CNS and heart ● MAOIs are reserved for:
● CNS - pronounced atropine-like effects ○ Patients suffering from atypical depression (i.e.,
○ Excitement, hallucinations, delirium, convulsions hypersomnia, agitation, anxiety)
○ Coma and respiratory depression may follow. ○ Patients who are refractory to other antidepressants
● Cardiac dysrhythmias
○ very common Adverse Reactions and Toxicity
○ tachycardia (antimuscarinic action) ● Hypertension
○ atrial or ventricular extrasystoles, QRS complex ● Postural hypotension (in up to 1/3 patients)
widening, QT interval elongation. ● CNS stimulation – tremor, excitement, insomnia,
○ ventricular fibrillation and sudden death may occur. convulsions in overdose.
● Hypotension ● Weight gain (increased appetite)
● Treatment - diazepam (seizures) ● Rare severe hepatotoxicity (hydrazine MAOI)

83
PCOL 211 - Pharmacology 1

Interaction with food Adverse E ects


● The most serious problem of this class of drugs ● Sedation
● Much less important in novel RIMA drugs like ● Gastrointestinal disturbances
moclobemide ● Priapism - a male medical condition characterized by a
● Tyramine “Cheese And Wine” Reaction spontaneous, prolonged, and often painful penile erection
○ Tyramine ● Hepatotoxicity
■ A Natural Indirect Sympathomimetic Produced By
TETRACYCLIC AND UNICYCLIC ANTIDEPRESSANTS
Fermentation
■ Some Food Contain High Amounts ● Unicyclic:
■ Normally Metabolized By MAO In The Gut And ○ Bupropion
Liver. ○ Mirtazapine
○ After MAOI Treatment Bioavailability Of Tyramine Is ● Tetracyclic (with anticholinergic effects)
Significantly Higher ○ Amoxapine
○ Pharmacodynamic Synergism ○ Maprotiline
○ Tyramine + MAOI = hypertensive crisis
Adverse E ects
■ Severe headache and potentially fatal intracranial
hemorrhage or other organ damage. ● Amoxapine is sometimes associated with a parkinsonian
○ Dietary restrictions: maturing cheeses, wine, beer, syndrome due to its D2 -blocking action
yogurts, bananas etc. ● Mirtazapine - sedation
● Bupropion - associated with agitation, insomnia, and
Interaction with drugs anorexia
● Hypertension & hypertensive crisis
NORADRENALINE AND DOPAMINE REUPTAKE
○ TCA wash-out period (2 weeks) when switching these
INHIBITORS (NDRI)
antidepressants! Lower risk in RIMA.
○ Levodopa (catecholamine precursor), ● Bupropion - rather CNS activating effects (low sedation)
sympathomimetics ○ Use:
● Serotonin syndrome (SSRI, TCA, opioids e.g. Pethidine) ■ severe depression + smoking cessation
○ Confusion, agitation and excitation, tremor, fever, treatment.
sweating, nausea, diarrhea, sleep disruption ○ Adverse Reactions:
● Prolong and profound the effect of benzodiazepines, ■ insomnia, excitation, restless, siezures
antihistamines, alcohol (inhibition of liver enzymes – low
Mirtazapine
specificity)
● MOA: Mirtazapine inhibits presynaptic α2-adrenergic
NORADRENALINE REUPTAKE INHIBITORS (NaRI) receptors, which leads to increased central concentrations
● Reboxetine of norepinephrine and 5- HT.
○ also rather activating
Amoxapine
○ severe depression prophylaxis and treatment
● Adverse reactions: ● N-methylated metabolite of loxapine
○ dry mouth, headache, dysuria, sweating ● 7-hydroxyamoxapine
○ Metabolite
SEROTONIN (5HT2A) ANTAGONISTS/REUPTAKE ○ Potent D2 blocker and is associated with antipsychotic
INHIBITORS (SARI) effects
● In depression with significant anxiety and sleep
Miscellaneous
disturbances
● Inhibits select. SERT, antagonist on H1 and α1 and ● St John´s wort (Hypericum perforatum)
α2-receptors ○ a herb containing number of active compounds
● Trazodone (Desyrel) (among other hyperforin a MAOI)
● Nefazodone (Serzone) - newer and improved ○ clinically effective and well tolerated
○ but it induces CYP450
TRAZODONE ■ risk of drug interactions!
● Indications ■ E.g. it decreases the effect of cyclosporine which
○ Sleep aid for depressed patients with insomnia may result in transplant rejection)
(hypnotic)
SEROTONIN SYNDROME
○ m-chlorphenylpiperazine (m-cpp) – metabolite
● Precautions and monitoring effects ● Upon intoxication or drug interactions
○ Trazodone is associated with sedation, hypotension, ○ SSRI, IMAO, TCA, venlafaxine, nefazodone,
nausea and priapism. pethidine, tramadol
● A potent cytochrome P4503A4 isoenzyme (CYP3A4) ○ Drugs inhibiting SSRI metabolism (erythromycin)
inhibitor ● Serotonin system overstimulation
● Hydroxynefazodone and m-cpp ● Management:
○ Metabolites ○ Benzodiazepines (lorazepam)
● ADR: hepatic failure ○ Supportive care

84
PCOL 211 - Pharmacology 1

○ 5-HT blockers, such as methysergide, ○ Decreased need for sleep


cyproheptadine, and propranolol may be given ○ Pressured speech
○ Racing thoughts or flight of ideas
Clinical Manifestations of Serotonin Syndrome and
○ Distractibility
Serotonin Withdrawal Syndrome
○ Psychomotor agitation
○ Engaging in dangerous, high-risk activities
Classification Serotonin Serotonin ● Short-term management for acute mania:
of Dysfunction Syndrome Withdrawal
○ Olanzapine
Syndrome
○ Haloperidol (may be used if parenteral administration
Cognitive- - Confusion None is necessary to manage acute agitation.
behavioral - Hypomania ○ Benzodiazepinessuch as Lorazepam (acute agitation
dysfunction - Agitation associated with mania

Autonomic - Diarrhea - Flu-like Nature of bipolar affective mania:


nervous - Shivering symptoms 1. Drugs that increase cathecolamine activity tend to
system - Fever - Dizziness exacerbate mania
dysfunction - Diaphoresis - Light headedness 2. Drugs that reduce activity of dopamine or
- Changes in - Chills norepinephrine relieve mania.
blood pressure - Nausea and
- Nausea and vomiting Key symptoms in the manic phase
vomiting - Sleep
disturbances ● Excitement
● Impulsivity
Neuromuscular - Myoclonus - Lethargy ● Aggression
dysfunction - Hyperreflexia - Myalgia ● Psychotic symptoms in some
- Tremor - Seonsory ● Cognitive impairment
- Seizure disturbances ● Hyperactivity
- Death (e.g.,
● Disinhibition
paresthesia)
● Diminished need for sleep

BIPOLAR DISORDER Key symptoms of depressive


● Formerly known as manic-depressive disorder ● Depressed mood
○ moods which cycle between mania and depression ● Sleep disturbance
○ cyclothymic disorder (similar to but milder than BD) ● Diurnal variation
● Caused by an imbalance of neurotransmitters. ● Anxiety
○ The neurotransmitter levels fluctuate and may be
BIPOLAR DISORDER
similar or reversed, depending on the patient’s current
clinical presentation. ● The diagnostic criteria for hypomania include the same
● ↓NE, ↑Ca = Depression symptoms as mania; however, hypomania is less severe.
● ↑NE, ↓GABA, ↓Ca = Mania Although changes in mood and affect are noticeable, the
● Glutamate binding to G proteins linked to patient remains functional in society. Hypomania can
N-methyl-D-aspartate (NMDA) receptors continue to progress, or it can cycle into manic episodes.
○ Involved in cycling and long-term potentiation ● A mixed episode is the coexistence of manic and
● Diagnosis: The diagnosis of bipolar disorder is made depressive symptoms for at least 1 week. This is often
using the DSM-IV-TR criteria and reviewing the individual’s referred to as mood incongruent.
history for episodes of mania, hypomania, or depression. ● The DSM-IV categorizes it into:
○ Bipolar I Disorder
Criteria for Bipolar disorder diagnosis:
○ Bipolar II Disorder
1. Self – esteem: highly inflated, grandiosity
2. Sleep: decreased need for sleep ○ Cyclothymia
3. Speech: pressured ○ Bipolar N.O.S.
4. Thoughts: racing thoughts and flight of ideas
Bipolar I or II Disorder
5. Attention: easily distracted
6. Activity: increased goal directed activity ● Bipolar I
7. Hedonism: high excess involvement in pleasurable ○ 1+ manic or mixed episodes
activities (sex, spending, travel)
○ May have other mood episodes
● Bipolar II
Mania ○ 1+ major depressive episodes and
● The diagnosis of mania requires the presence of an ○ 1+ hypomanic episodes
elevated, expansive, or irritable mood for at least 1 week ○ Never manic or mixed episode
(less time if it precipitates hospitalization). In addition, BIPOLAR DISORDER
three or more of the symptoms must be present and
unrelated to substance abuse or a medical condition. ● Cyclothymia is diagnosed when the depressive and
Functional and social impairment must also be present. hypomanic symptoms persist for 2 years.
● Symptoms: ● Rapid cycling is diagnosed when the patient has
○ Grandiose ideations or expansive self-esteem experienced at least four depressive, manic, hypomanic, or

85
PCOL 211 - Pharmacology 1

mixed (depressive and manic) episodes within 12-month ○ Therapeutic range: 0.5-1.5 mEq/L
period ○ Acute mania: ≤0.8 mEq/L
● Pharmacotherapy: ○ Maintenance therapy: (≤1.0 mEq/L) target a
○ Depending on the clinical presentation, antipsychotics, concentration
mood stabilizers, or antidepressants may have a role ● Adverse effects can occur early in therapy or with chronic
in treatment of patients with bipolar disorder. However, therapy, and individual patients may experience toxic
the primary therapeutic agents in the management of effects despite a concentration ≤1.5 mEq/L.
this disease are lithium and anticonvulsants,
Adverse Events Associated with Lithium
commonly known as mood stabilizers.
● Bupropion (Wellbutrin) or Paroxetine (Paxil) ● Early onset
○ For patients with severe depression or suicidal ○ Gastrointestinal upset
ideation ○ Fine hand tremor
○ Less risk for conversion to mania ○ Nausea
● Lithium (Litcab) & Lamotrigine (Lamictal) ○ Leukocytosis
○ are the first-line agents recommended for the ○ Polydipsia
treatment of bipolar depression. ○ Nocturia
○ Polyuria
DRUGS FOR BIPOLAR DISORDER
○ Muscle weakness
● Lithium ○ Dry mouth
● Valproic Acid ○ Difficulty concentrating
● Carbamazepine ○ Sedation
● Others ● Side effects
○ Levels - maintenance at 0.6 - 1.0 mEq/L (monitor 2-4
LITHIUM
times per year once at stable dose)
● Indications: First-line treatment and prevention of bipolar ○ Increased urination - once daily dosing (if tolerated)
disorder, except in the cases of mixed episodes or rapid can reduce polyuria
cycling. ○ Thirsty tremors - maintain adequate fluid intake as
● Chemistry: dehydration can result in lithium toxicity
○ is a monovalent cation, similar to sodium and ○ Hair thinning hypothyroidism - occurs in 8-9% of
potassium. patients
○ Lithium carbonate and citrate are the two clinically ○ Interactions - Medications that increase Li levels:
relevant salt forms ■ NSAIDS (by 16-60%)
○ Lithium carbonate is used more often in the treatment ■ ACEI/ARBs (by 30-40%)
of bipolar disorder. ■ Diuretics (by 25-40%)
○ Lithium citrate is available as a liquid and may be ○ Upset stomach - nausea, vomiting, diarrhea
helpful in patients who are noncompliant with tablets ○ Muscle weakness
or capsules. ○ Skin effects - acne, psoriasis
● Food can delay absorption but does not affect the extent of
Long Term Use
absorption.
○ Some adverse effects (such as nausea, vomiting, and ● Weight gain
tremors) are related to the rapid rise in serum ● Rash
concentrations; therefore, administering lithium with ● Altered taste
food or giving lithium as an extended-release product ● Alopecia
may diminish such side effects. ● Nephrotoxicity (rare)
● May affect membrane stabilization. ● Acne
● Lithium may augment homeostasis by enhancing the ● Nephrogenic diabetes insipidus
function of secondary messenger systems, particularly ● Psoriasis
cyclic adenosine monophosphate (camp), cyclic guanosine ● Decreased libido
monophosphate (cGMP), and phosphatidylinositol ● Hypothyroidism
● is also noted to inhibit norepinephrine release and ● Nonspecific T-wave change
accelerate its metabolism. ● Nephrogenic diabetes insipidus
● May also decrease receptor sensitivity and increase
Toxicity
presynaptic reuptake of norepinephrine and 5-HT
● Severe drowsiness
Clinical Uses
● Ataxia
1. Bipolar affective disorder ● Coarse hand tremor
2. Schizoaffective disorder ● Hyperreflexia
3. Adjunct to tricyclic antidepressants and SSRIs in patients ● Muscle twitching
with unipolar depression ● Nystagmus
● Myoclonus
Administration and Dosage
● Seizure
● Administration and dosage: ● Choreoathetosis
○ Lithium has a narrow therapeutic index ● Coma

86
PCOL 211 - Pharmacology 1

● Cogwheel rigidity ○ Nausea, vomiting, and stomach cramps (may be


● Death diminished by giving divalproex instead of sodium
● Vomiting valproate and valproic acid)
● Confusion ○ Lethargy
● Treatment of toxicity involves: ○ Alopecia
○ Holding or discontinuing lithium therapy ○ Thrombocytopenia
○ Correcting water and electrolyte disturbances ○ Elevation of liver function tests
○ Attempting to empty the stomach by emesis or gastric ● Idiopathic side effects include:
lavage (acute toxicity) [Charcoal administration is ○ Weight gain
ineffective] ○ Hepatic failure
○ Performing hemodialysis or peritoneal dialysis [only ○ Pancreatitis
way to remove lithium in severe toxicity (i.e., ≥3 ○ Agranulocytosis
mEq/L)].
Drug-drug interactions
Precaution and Monitoring E ects
● Drugs known to increase VPA concentrations:
● Absolute contraindications ○ Cimetidine
○ Acute renal failure ○ Chlorpromazine
○ First trimester of pregnancy ○ Erythromycin
● Relative contraindications ○ Felbamate
○ Renal, cardiovascular, and thyroid disease ○ Salicylates
● Drugs known to decrease VPA concentrations include
Monitoring
rifampin, Phenobarbital, lamotrigine, phenytoin, and
● Renal and thyroid function should be assessed at baseline carbamazepine (CBZ).
and every 2-3 months for the initial 6 months, and then ● VPA can increase warfarin and zidovudine concentrations.
every 6 months to yearly in patients maintained in lithium ● Concomitant use with CNS depressants may have additive
effects
Drug Interactions
CARBAMAZEPINE
● Lithium + antipsychotics or BZD = increase the risk of
neurotoxicity ● Enzyme inducer
● Lithium + medications that increase 5-HT = serotonin ● Used in trigeminal neuralgia
syndrome ● S/E: SJS
● Indications:
Factors that Change Lithium Concentrations ○ anticonvulsant
○ mood stabilizer.
Decreased Lithium Increased Lithium ○ CBZ is typically reserved for use in bipolar patients
Concentration Concentration with partial or no response to lithium.
○ It may be considered first-line therapy in patients with
- Acetazolamide - ACE inhibitors a history of not responding to lithium or in patients with
- Methylxanthines (e.g., - NSAIDS a history of mixed episodes or rapid cycling.
theophylline, caffeine) - Thiazides
● MOA: Although the mechanism of action is unknown, it
- Osmotic diuretics - Dehydration
- Pregnancy (third - Renal dysfunction may be related to the ability to modulate norepinephrine
trimester) - Sodium loss and G protein-linked second messenger systems,
- Sodium supplements - Fluoxetine particularly cAMP.
- Urine alkalinizers (e.g., - Postpartum ● Clinically, CBZ is only considered unsuccessful for mood
sodium bicarbonate, stabilization when the patient has failed to respond and the
potassium citrate)
serum concentration was well within the range of 6-12
μg/mL for 4-6 weeks.
VALPROIC ACID (VPA) ● CBZ induces enzymatic metabolism for many drugs,
including itself.
● Enzyme inhibitor
● Autoinduction may begin as early as day 3 therapy and
● Teratogenic (Spina bifida)
continue for up to 30 days after the last dose of CBZ. For
● Indications:
this reason, careful monitoring of serum concentrations
○ Anticonvulsant
and awareness of potential drug interactions is important.
○ Can be used to treat acute mania, mixed mania, and
rapid cycling. Dose-related side e ects
○ It can be used as monotherapy in bipolar disorder or
as an adjunctive therapy with lithium or another mood ● Gastrointestinal effects
stabilizer. ○ Nausea, vomiting, diarrhea, constipation, anorexia,
● MOA: Although the mechanism of action is not fully abdominal pain
understood, it is hypothesized to be related to GABA ○ Decreasing the dose and giving with food may be
modulation and possibly second messenger systems. helpful.
● Precautions and monitoring effects:
○ Common dose-related side effects include:

87
PCOL 211 - Pharmacology 1

● Hematological effects
USE OF DUAL MOOD STABILIZERS
○ Blood dyscrasias (e.g., aplastic anemia,
thrombocytopenia, neutropenia; rare), leukopenia ● In patients who require more than one mood stabilizer, the
(transient; fairly common) combination of lithium and CBZ and VPA may be
○ Mild, transient elevations of liver function tests beneficial.
(monitor yearly) ● Lithium may cause leukocytosis, and CBZ and VPA are
known to cause leukopenia.
Non-dose-related side e ects
● Because CBZ and VPA are known to cause a variety of
● Exfoliative dermatitis (e.g., Stevens-Johnson syndrome), blood dyscrasias, the combination of these two agents is
agranulocytosis, and hepatic failure are rare but potentially not recommended.
fatal ● The more traditional mood stabilizers (i.e., lithium, VPA,
and CBZ) may also be combined with one of the newer
OTHER DRUGS FOR BIPOLAR
anticonvulsants (i.e., lamotrigine or gabapentin) for mood
1. Lamotrigine - approved for prevention of recurrence stabilization.
2. Gabapentin, oxcarbazepine, and topiramate
Mood Stabilizers in Pregnancy
3. Aripiprazole, chlorpromazine, olanzapine, quetiapine,
risperidone, and ziprasidone - approved by the FDA for ● Lithium, VPA, and CBZ can cause birth defects.
treatment of the manic phase of bipolar disorder. ● Lamotrigine, gabapentin, and oxcarbazepine are category
4. Olanzapine + fluoxetine and quetiapine - approved for C drugs, which indicates that anomalies have been seen in
treatment of bipolar depression. animal studies.

NEWER MOOD STABILIZERS

● Lamotrigine DRUGS FOR PSYCHOSIS


● Gabapentin
● Oxcarbazepine
ANTIPSYCHOTICS
LAMOTRIGINE
● aka neuroleptics, major tranquilizers
● Chemistry: The chemical structure of lamotrigine is similar ● “Neuroleptics” because of their tendency to cause
to phenytoin and CBZ. movement disorders
● MOA: The mechanism of action is not fully understood, but ● “Major tranquilizers” vs minor tranquilizers (eg.
lamotrigine appears to block sodium-mediated release of benzodiazepines)
glutamate and aspartate at low concentrations.
○ At higher concentrations, it blocks GABA and How Antipsychotics (Neuroleptics) Work
acetylcholine release.
● Antipsychotic drugs work by blocking receptor sites for
● Adverse effects: Common side effects include dizziness,
dopamine, thus reducing the effects of dopamine in
diplopia, blurred vision, nausea and vomiting, rash,
regions of the brain
photosensitivity, ataxia and headache. The rash may be
○ Antagonist for dopamine
severe or potentially life-threatening: therefore, patients
● Chlorpromazine was the first effective neuroleptic drug
should be instructed to discontinue this medication at the
● Phenothiazines block dopamine D2 receptor
first sign of a rash

GABAPENTIN

● Chemistry: Gabapentin is structurally similar to GABA but


does not alter its function as an antagonist or agonist.
● MOA: The mechanism of action is currently unknown for
its anticonvulsant and mood stabilizing properties.
However, its relatively clean profile makes gabapentin
appealing as an adjunctive mood stabilizer.
● Adverse effects: Side effects such as somnolence,
nystagmus, dizziness, ataxia, and fatigue have been
associated with gabapentin.

OXCARBAZEPINE

● Structurally and chemically similar to carbamazepine.


● Expected to have comparable efficacy in the management
of bipolar disorder, with fewer side effects and better
tolerability.
● Considered to be an alternative to carbamazepine in the ● Agonist:
○ Amphetamine promotes the release of dopamine
treatment of bipolar disorder.
and fosters symptoms of schizophrenia
○ Both amphetamine and cocaine block reuptake of
dopamine and foster symptoms of schizophrenia
● Antagonist:

88
PCOL 211 - Pharmacology 1

○ Chlorpromazine, a drug that blocks symptoms of Hallucinations


schizophrenia, occupies the dopamine site on the
D2 receptor, preventing receptor activation by ● are perceptual experiences that do not have a basis in
dopamine reality.
● Auditory is the most common. May involve sounds or
Tranquilizer tactile hallucinations, such as tingling or burning
sensations of the skin.
● a drug that is used to reduce anxiety, fear, tension,
● Perceptual distortion - Sensory perceptions not directly
agitation, and related states of mental disturbance.
attributable to environmental stimuli (appear real to the
PSYCHOSIS schizophrenic):
○ Auditory (hearing)
● Symptoms of delusions, hallucinations, and disorders of
○ Visual (seeing)
thought
○ Olfactory (smelling)
● Due to increase dopamine levels (as in amphetamines)
○ Tactile (feelings)
SCHIZOPHRENIA ○ Gustatory (tasting)
● A disorder represented by a heterogeneous syndrome of: Delusions
○ Disorganized and bizarre thoughts
○ Delusions ● are false beliefs (of persecution, that others are spying on
○ Hallucinations them or are out to get them) or of reference, that objects,
○ Inappropriate affect events, or other people have some particular significance;
○ Impaired social functioning of identity (believe you are someone else) or of
● Characterized by positive and negative symptoms, a grandiosity (that you are very special in some way,
pattern of social and occupational deterioration, and talented or prestigious).
persistence of the illness for at least 6 months ● False beliefs firmly and consistently held despite
● 1% of population, inheritable disconfirming evidence or logic.
○ Delusions of control – belief that other people,
THE COURSE OF SCHIZOPHRENIA animals, or objects are trying to control you
○ Delusions of thought broadcasting – belief that
● Prodromal phase: Onset and buildup of schizophrenic
others can hear your thoughts
symptoms
○ Delusions of persecution
● Active phase: Full-blown symptoms: severe disturbances
○ Delusions of reference – Belief that you are the
in thinking, deterioration in social relationships, and flat or
center of attention and that things happening around
inappropriate affect
you are always directed at you; that others are
● Residual phase: Symptoms no longer prominent
whispering about you or that the words in the paper
Complete recovery is rare, but schizophrenics can lead are meant just for you and have special importance.
productive lives. ○ Thought withdrawal – belief that your thoughts are
being removed from your mind.
THE SYMPTOMS OF SCHIZOPHRENIA ○ Capgras’s syndrome - belief that there are “doubles”
of you who may replace or coexist with you.
● Characteristic symptoms:
○ Two or more of the following, each persisting for a Greatest Distress
significant portion of at least a 1-month period:
■ Delusions ● When voices are dominant and insulting, and patient lacks
■ Hallucinations communication with the voices
■ Disorganized speech ● Coping strategies:
■ Grossly disorganized or catatonic behavior ○ Distraction
○ Ignoring
POSITIVE SYMPTOMS ○ Selective listening (to only some of what the voices
say)
● Symptoms which are not present in the normal individual
○ Setting limits (for example, that voices can speak to
but are added to the schizophrenic person.
you after 8pm, but not before).
○ Hallucinations and Delusions
○ Distortions or excesses of normal functioning Bizarre Behavior
● Hallucinations - auditory, visual, tactile, and/or ● Aggressive/agitated, odd clothing or appearance, odd
olfactory hallucinations; voices that are commenting social behavior, repetitive- stereotyped behavior
● Delusions - persecutory, grandiose, paranoid, religious;
thought broadcasting, thought insertion NEGATIVE SYMPTOMS
● Bizarre behavior - aggressive/agitated, odd clothing or
appearance, odd social behavior, repetitive-stereotyped ● Symptoms involving characteristics, behavior or abilities
behavior that are present in a normal person but missing in the
schizophrenic individual.
● Associated with inferior premorbid social functioning
● Flat affect (little or no emotion in situations in which strong
reactions are expected)
● Alogia (poverty of speech)

89
PCOL 211 - Pharmacology 1

● Avolition (inability to take action or to become goal


CATATONIC SCHIZOPHRENIA
oriented)
● May indicate irreversible neuronal loss in a structurally ● Includes significant disturbance of motor activity
abnormal brain ○ Motoric immobility or stupor (withdrawn catatonia; little
● Asociality - Inability to form close personal relationships or no motor activity)
● Anhedonia - Inability to feel pleasure ○ Excessive purposeless motor activity (excited
catatonia)
DISORGANIZED SYMPTOMS ● Extreme negativism or physical resistance
● Disorganized speech (Formal thought disorder) ● Peculiar voluntary movements, including waxy flexibility
● Incoherence - Inability to organize ideas ● Echolalia or echopraxia
● Loose associations (derailment) - Rambles, difficulty ● Behavior may become dangerous/violent
sticking to one topic ECHOLALIA - ginagaya yung sinasabi ng kausap
● Disorganized behavior - Odd or peculiar behavior ECHOPRAXIA - ginagaya yung movement ng kausap
○ Silliness, agitation, unusual dress
■ Example: wearing several heavy coats in hot UNDIFFERENTIATED SCHIZOPHRENIA
weather
● Behavior shows prominent psychotic symptoms not
OTHER SYMPTOMS meeting criteria for paranoid, disorganized, or catatonic
● Catatonia schizophrenia (Doesn’t fit into any other category)
○ Motor abnormalities RESIDUAL SCHIZOPHRENIA
○ Repetitive, complex gestures (usually of the fingers or
hands) ● At least one previous schizophrenic episode but current
○ Excitable, wild flailing of limbs absence of prominent psychotic features and continuing
● Catatonic immobility evidence of 2 or more symptoms, such as marked social
○ Maintain unusual posture for long period of time isolation, peculiar behaviors, blunted affect, odd beliefs, or
■ e.g., stand on one leg unusual perceptual experiences
● Waxy flexibility ○ Showing symptoms of schizophrenia which remain in
○ Limbs can be manipulated and posed by another between episodes or after medication has been used
person to treat all treatable symptoms.
● Inappropriate affect
PSYCHOTIC DISORDERS ONCE CONSIDERED
○ Emotional responses inconsistent with situation SCHIZOPHRENIA
TYPES OF SCHIZOPHRENIA ● BRIEF PSYCHOTIC DISORDER - Schizophrenic episodes
● Paranoid that last at least one day but less than one month
● Disorganized ● SCHIZOPHRENIFORM DISORDER - Schizophrenic
● Catatonic episodes that last at least one month but less than six
● Undifferentiated months
● Residual ○ Does not require impairment in social or occupational
functioning
PARANOID SCHIZOPHRENIA
OTHER PSYCHOTIC DISORDERS
● Characterized by one or more systematized delusions or
auditory hallucinations and the absence of such symptoms ● DELUSIONAL DISORDER - Holding non bizarre beliefs
as disorganized speech and behavior or flat affect (situations that could occur) lasting at least one month;
○ People wake up in the same delusions every day, except for the delusion the behavior is not odd
rather than having disorganized delusions ○ Common themes: grandiosity, jealousy, persecution,
● Includes mostly positive symptoms. and somatic complaints
● Responds more readily to medication than other forms ● SHARED PSYCHOTIC DISORDER - A person with a
close relationship to an individual with delusional/psychotic
Most common symptom: Delusions of persecution believes comes to accept those beliefs
○ Folie a deux (Madness shared by two)
DISORGANIZED SCHIZOPHRENIA
PATHOPHYSIOLOGY
● Formerly “hebephrenic schizophrenia”
● disorganized behaviors manifested by disorganized ● Neurotransmitter changes
speech and behavior, and flat or grossly inappropriate ○ Dopamine
affect ○ Glutamate
● Have delusions and hallucinations that are less organized ○ Serotonin
than those of the paranoid schizophrenic, and have
1. Dopamine Hypothesis
disorganization of thought, behavior and emotion.
● Speech may become incoherent and the person may ● Excessive dopamine in mesolimbic system
invent new words (neologisms). ● Homovanillic acid (HVA)
○ metabolite of Dopamine

90
PCOL 211 - Pharmacology 1

○ Genetic background
Dopaminergic Tracts
● Precipitating Factors
Pathway Effects ○ Stress
○ Substance of Abuse
Mesolimbic-mesocotical Behavior
MANAGEMENT OF SCHIZOPHRENIA
pathway

Nigrostriatal Tract Coordination of voluntary Psychosocial therapy


movement
● Practical advice, getting in touch with one’s feelings,
Tuberoinfundibular Inhibits prolactin release understanding one’s effect on others, understanding
reasons for one’s actions and feelings, and understanding
Medullary-periventricular Involved in eating behavior the impact of past on present behavior
● Most important quality of therapist: Friendship
Incertohypothalamic Regulate the anticipatory
motivational behavior Institutional approaches
● Social learning programs: Appropriate self-care,
2. Glutamatergic Dysfunction conversational skills, role skills
● Undesirable behaviors are decreased through
● PHENCYCLIDINE
reinforcement and modeling.
○ NMDA antagonist
○ Produce “schizophrenia- like” symptoms Cognitive-behavioral therapy
3. Altered Serotonin Levels ● Work at reducing frequency and severity of positive and
negative symptoms
4. Increased Concentration of NE ● Enhance coping skills
● Weaken beliefs regarding power/omnipotence of auditory
● observed in limbic structures of patients with chronic
hallucinations
paranoid schizophrenia, but not with other subtypes.
● Challenge false beliefs
ETIOLOGY OF SCHIZOPHRENIA ● Social skills training emphasizes communication skills and
assertiveness, encourages functional independence,
● Biological:
improves family relationships
○ Genetic, brain-structure, and biochemical explanations
Family communication and education
Physiological Factors in Schizophrenia
● Normalize family experience
● High Neurological Activity ● Educate family members about schizophrenia
○ PET scans show higher levels of activity in the ● Identify strengths and competencies
prefrontal cortex and temporal cortex of schizophrenic ● Develop problem solving and stress management skills
individuals ● Learn to cope with symptoms
● Differences in cerebral glucose metabolism, especially ● Recognize early signs of relapse
during cognitive tasks ● Create supportive family environment
Problems with brain development and activity ● Understand/meet needs of all family members

● Reversed hemispheric dominance NEUROLEPTICS


○ Suggesting that the left hemisphere did not develop ● Sometimes referred to as major tranquilizers, because
normally and become dominant some of them can tranquilize and sedate
● Failure of neural migration ● USES:
○ The movement of neurons to the gray matter of the 1. Schizophrenia, mania, and delusional disorder
cortex during prenatal development appears retarded 2. Bipolar disorder
● Cortical atrophy 3. (haloperidol, pimozide) are used off-label to treat
○ Prefrontal cortex – smaller, less active, and Tourette syndrome
decreasing in size at a faster rate than normal 4. Aripiprazole is prescribed in some cases of
(hypofrontality) Asperger's syndrome
○ Temporal cortex- smaller and less active than normal 5. Chlorpromazine – for intractable hiccups
● Subcortical atrophy 6. Promethazine – antipruritic
○ Enlarged ventricles
○ Smaller hippocampus (esp. for people with negative
sx)
○ Thalamus is smaller and less active
○ Smaller amygdala

SCHIZOPHRENIA: Who is at Risk


● Predisposing Factors
○ Season of birth
○ Pregnancy and birth complications

91
PCOL 211 - Pharmacology 1

ADVERSE EFFECTS

RECEPTOR BLOCKED SIDE EFFECTS


TYPICAL ANTIPSYCHOTICS
Dopamine D2 EPS

Histamine H1 Sedation 1. Phenothiazines

● Aliphatic - Chlorpromazine, Promazine, Trifluoperazine


Muscarinic Dry mouth, constipation,
● Piperazine - Fluphenazine, Perphenazine,
urinary retention, blurring of
vision Acetophenazine, Trifluorophenarine, Prochlorperazine
● Piperidine - thioridazine
Adrenergic A1 Orthostatic hypotension
POTENCY: Piperazine>Piperidine>Aliphatic
● These symptoms include:
○ Dystonia (continuous spasms and muscle Chlorpromazine
contractions) ● first antipsychotic developed as a surgical anesthetic
○ Akathisia (motor restlessness) ● Receptors blocked:
○ Parkinsonism (characteristic symptoms such as ○ Adrenoceptors - Hypotension
rigidity) ○ Serotonin
○ Bradykinesia (slowness of movement) ○ Histamine 1 - sedation
○ Tremor ○ Muscarinic - Constipation, dryness
○ Tardive yskinesia (irregular, jerky movements) ○ Dopamine blockade - Parkinsonian syndrome
TYPICAL VS ATYPICAL Mesoridazine

TYPICAL ATYPICAL ● major metabolite of thioridazine


● more potent than the parent compound and accounts for
Haloperidol, chlorpromazine, Risperidone most of the effect.
thioridazine Clozapine
2. Thioxanthenes
Blocks D₂ receptors Blocks D₂ and 5-HT₂ ● Chlorprothixene and Thiothixene
(Risperidone) ● Same structure as phenothiazine but N replaced by C
Blocks D₁ and 5-HT₂
(clozapine) 3. Butyrophenones

Treats positive symptoms Treats both positive and ● Haloperidol (Haldol, Serenace), Droperidol
only negative symptoms ● Haloperidol (HaldolⓇ)
○ non-selective neuroleptic
Causes movement disorders Little or no movement ○ EPS is common
disorders
4. Dibenzoxazepine
Thioridazine (Mellaril, Clozapine(Clozaril,Leponex)
● Clozapine
Melleril) Quetiapine (Seroquel)
Chlorpromazine (Thorazine, Ziprasidone(Geodon,Zeldox) ATYPICAL ANTIPSYCHOTICS
Laractyl, Psynor) Aripiprazole(Abilify)
Perphenazine (Trilafon) Olanzapine(Zyprexa) ● Dihydroindolines - Molindone
Thiothixene (Navane) Quetiapine (Seroquel) ● Diphenylbutylpiperidine - Pimozide
Fluphenazine (Prolixin, Risperidone(Risperdal) ● Dibenzoxazepine - Clozapine
Haloperidol (Haldol, ● Benzisoxazole - Risperidone
Serenace) Modezine,
● Thienobzd - Olanzapine
Phlufdek, Sydepres)
● Fluorophenylindole - Quetiapine

92
PCOL 211 - Pharmacology 1

Clozapine SUMMARY ANTIPSYCHOTIC DRUGS AND LITHIUM


● antipsychotic effects : 5-HT 2A/2C and D2 receptor
antagonism.
● more potent antagonism at the D4 receptor than the D2
receptor.
● Lowest likelihood in producing EPS and tardive
dyskinesia
● Reserve drug for severely schizoprenicpatients who are
refractory to traditional therapy.

Olanzapine (Zyprexa)
● for certain people with schizophrenia who take
perphenazine first and get no or minimal benefit and/or
experience intolerable side effects.
● Zyprexa is not a good option for people who are
overweight, have blood sugar abnormalities, diabetes, or
heart disease.

LOW VS HIGH POTENCY

LOW POTENCY HIGH POTENCY

Chlorpromazine, thioridazine, Haloperidol,


clozapine fluphenazine

Sedation, orthostatic hypotension, Extrapyramidal


anticholinergic effects symptoms PREPARATIONS AVAILABLE

SUMMARY OF ADVERSE EFFECTS


● ANS
○ Muscarinic receptor blockade :
■ Loss of accommodation
■ Difficulty in urination
■ Constipation
■ Toxic - confusional state
● Alpha adrenergic blockade :
○ Chlorpromazine - Orthostatic hypotension (the most
common cardiovascular effect)
○ Mesoridazine - Failure to ejaculate
○ Impotence
● CNS
○ Dopamine receptor blockade
● EPS
○ Prolactin elevation
■ Amenorrhea,galactorrhea,infertility
● Dopamine receptor hypersensitivity:
○ Tardive dyskinesia (involuntary movements of the face
or limbs)
○ Tardive akathisia (extreme restlessness)
● Endocrine
○ Hyperprolactinemia
● Serotonin 5-HT2 blockade
● Orthostatic hypotension
● Sedation
● Weight gain

93
PCOL 211 - Pharmacology 1

MOVEMENT DISORDERS
DRUGS FOR ANXIETY
1. Extrapyramidal symptoms (EPS)

● aka neuroleptic-induced parkinsonism ION CHANNEL & NEUROTRANSMITTER RECEPTORS


● Most common (15%)
Types of channels on the membrane of nerve cells:
● Coarse tremors, rigidity, bradykinesia
1. Voltage-gated channels
● Risk: high potency
● Na and Ca
● TX: lower dose, anticholinergics
● When sodium channels or voltage- gated channel
2. Acute Dystonia opens there will be influx of ions like sodium and
calcium. And what's they are inside the cell because
● Muscular spasm, involuntary movement they are positively charge or cationic they will reduce
● Spasmodic torticollis, trismus, tongue protrusion, electrical impulses which lead to action potential or
opisthotonos, upward movement of eyes (oculogyric crisis) movement
● Risk: high-potency antipsychotics ● Are voltage sensor component of the membrane
● Onset: early in tx (days) which control the gating
● Tx: IM/IV anticholinergics (benztropine, diphenhydramine, 2. Ligand-gated channels – (Ionotropic receptors)
biperiden) ● Binds to receptors
3. Akathisia ● Ligands are anything that binds to a receptor and
open channel to promote action
● Subjective feeling of muscular discomfort
● Agitated, pace relentlessly, alternately sit and stand ● Metabotropic receptors (coupled type receptors)
● Risk: recent increase/onset of meds ○ Once the receptor bounds to their respective drug
● Onset: 1st month of therapy they will be coupled in G proteins either stimulatory
● Tx: beta-blockers (propranolol), BZDs (lorazepam), Gs protein or inhibitory Gi protein or to Gq protein
clonidine to release secondary messengers
○ Too much = overthinking
4. Neuroleptic malignant syndrome (NMS)
● idiosyncratic, life-threatening
● Motor: Muscular rigidity, dystonia, agitation
● Autonomic: hyperpyrexia, hypertension
● Risk: high-dose, rapid dose escalation
○ IM injection, prior history
● Onset: usually within first few weeks
● Tx: discontinue meds, supportive, dantrolene,
bromocriptine

5. Tardive dyskinesia
● Choreoathetoid movements
● Tongue protrusion/twisting,lip puckering
● Risk: elderly, long-term tx, female,
● Onset: years after tx ● Ion channels leads to movement
● Tx: lower dose, change meds ● Ligand gated channels - anything that binds to the
receptor resulting to movement
OTHER ADVERSE EFFECTS ○ ↑ in brain = magiging anxious yung buong body
● Agranulocytosis - clozapine, chlorpromazine
● Pigmentary retinopathy - thioridazine Neurotransmitter Receptors
● ECG changes - prolonged QT interval-ziprasidone
● Ionotropic receptors
OTHER USES OF ANTIPSYCHOTICS ● Metabotropic receptors
○ Membrane delimited
● Antiemetic (blocks dopamine receptors) - prochlorperazine
○ Diffusible second messenger
● Intractable hiccups - chlorpromazine
● Pruritus (antihistamine) - promethazine (Zinmet, ● HYPERPOLARIZATION - efflux of K, influx of Cl-,
Thaprozine) change in cell membrane potential
○ Means relaxation during the membrane potential
● DEPOLARIZATION - influx of Na and Ca- cell becomes
positive
○ Action/gives movement
○ Over stimulated
● REPOLARIZATION - change in the membrane potential
that returns it to negative, happens after depolarization
○ Time when the neurons start to relax

94
PCOL 211 - Pharmacology 1

Metabotropic Receptors Ca-activated" K (leads to


channel paralysis)

A.3 Calcium channels

Omega conotoxin Blocks N-type Pacific cone


(ω-CTX-GVIA) channel snail (leads to
paralysis)

Agatoxin (ω-AGA-IVA) Blocks P-type Funnel web


channel spider (leads
to paralysis)

B. LIGAND-GATED

B.1 Nicotinic ACh


receptor

α-Bungarotoxin Irreversible Marine snake


Synaptic potentials antagonist

● Excitatory B.2 GABA A receptor


○ Promote depolarization/movement they excite your
muscle/brain once they are active Picrotoxin Blocks channel South Pacific
○ Excitatory postsynaptic potential (EPSP) (leads to seizures plant
and tremors)
○ Depolarization
● Inhibitory B.3 Glycine receptor
○ Inhibition/prevention it leads to hyperpolarization or
relaxation Strychnine Competitive Indian plant
antagonist
○ Inhibitory postsynaptic potential (IPSP)
○ Hyperpolarization B.4 AMPA receptor (Natural GLutamate)
Excitatory neurotransmitters
Philanthotoxin Blocks channel Wasp
● Opens Na or Ca channels/ influx→ depolarization (more
positive) → nerve impulse AMPA RECEPTOR
● e.g. Norepinephrine, Dopamine, Acetylcholine, Glutamate
(ajinomoto), Aspartate ● α-Amino-3-hydroxy-5-methylisoxazole-4-propionic acid
● Natural agonist is glutamate
Inhibitory neurotransmitters ● Permeable to Na+ and K+, but not Ca2+
● Opens Cl channels → hyperpolarization (more negative) ● Channel opening short (10 ms)
→ no nerve impulse
● e.g. glycine, gamma-aminobutyric acid (GABA)
Some toxin used to characterized ion channels

Channel Types Mode of Toxin Source


Action

A. VOLTAGE-GATED

A.1 Sodium channels (Excitatory)

Tetrodotoxin (TTX) Blocks channel from Puffer fish


outside
- Preventing the
influx of Na
promoting no action IDENTIFICATION OF CENTRAL
potential NEUROTRANSMITTERS
Batrachotoxin (BTX) Slows inactivation, Colombian ● More difficult for CNS
shifts activation frog ○ Anatomic complexity
○ Limitation of available techniques
The more colorful the animal = more toxic
PLEASE MEMORIZE!!!!
A.2 Potassium channels (less action than Na)
Anxiety Decrease GABA
Apamin Blocks "small Honeybee Increase 5GT (serotonin), D, NE
Ca-activated" K (leads to
channel paralysis)
Psychosis Increase Glutamate
Charybdotoxin Blocks "big Scorpion Increase D

95
PCOL 211 - Pharmacology 1

○ Restlessness
Depression Decrease NE, 5HT, D
○ Fatigue
Mood DIsorder Imbalance in NT ○ Difficulty in concentrating
○ Irritability
Seizure Increase Neuron Firing ○ Muscle tension
Increase 5HT, NE, D ○ Sleep disturbance

Parkinson’s Decrease D Diagnostic Criteria for Panic Attacks


Increase ACh ● At least four of the following symptoms developed
abruptly and reached a peak within 10 minutes
● Palpitations or tachycardia, Sweating, Trembling or
Alzheimer’s Increase ACh
shaking, Sensations of shortness of breath or smothering,
Feeling of choking, Chest pain or discomfort, Nausea or
ANXIETY
abdominal distress
● is an emotional state commonly caused by the perception ● Dizziness, unsteadiness, lightheadedness, Derealization
of real or potential danger that threatens the security or depersonalization, Fear of losing control, Fear of
of an individual dying, Paresthesia, Chills or hot flushes
● refers to many states in which the sufferer experiences a
sense of impending threat or doom that is not well defined Treatment
or realistically based. A. Goals of Therapy:
● usually accompanied by symptoms such as tachycardia, ● to reduce the severity, duration and frequency of the
palpitations, tachypnea, sweating, trembling and anxiety symptoms
weakness ● to improve the patient’s overall functioning
● to prevent anxiety symptoms
TYPES OF ANXIETY
● to improve quality of life
1. Panic Disorder Nonpharmacologic Therapy
● recurrent unexpected panic attacks that can occur with ● Short-term counseling
agoraphobia (fear of crowded space) in which patients fear ● Stress management
places in which escape might be difficult ● Psychotherapy - for encouragement
● Meditation
2. Specific Phobia
● Exercise
● intense fear of particular objects or situations (e.g. snakes, ● Avoidance from caffeine, nonprescription stimulants
heights);most common psychiatric disorder and diet pills
3. Social Phobia Pharmacologic Therapy
● intense fear of being scrutinized in social or public ● Sedative-hypnotic drugs - calming effect and sleep
situations (e.g., giving a speech, speaking in class) ● Anxiolytic - calming effect
● Minor tranquilizers - sleep
4. Generalized Anxiety Disorder
● intense pervasive worry CLASSIFICATIONS
● over virtually every aspect of life 1. Benzodiazepines
2. Barbiturates
5. Post-Traumatic Stress Disorder
3. Other Drugs
● persistent reexperience of a trauma, efforts to avoid ● Ethanol, Chloral Hydrate
recollecting the trauma, and hyperarousal ● Zolpidem
● Zaleplon
6. Obsessive-compulsive disorder
● Eszopiclone
● recurrent obsessions and compulsions that cause ● Ramelteon
significant distress and occupy a significant portion of ● Buspirone
one’s life
BENZODIAZEPINES
SPECIFIC PHOBIAS:
● Ophidiophobia is an extreme, overwhelming fear of ● most widely used anxiolytic
snakes. The condition is called a specific phobia (fear) ● benzene ring (A) fused to a seven-membered diazepine
● Acrophobia is an intense fear of heights ring (B)
● Aquaphobia is fear of water ● usually contain a 5-aryl substituent (ring C) and a
● Thanatophobia is an intense fear of death 1,4-diazepine
● Philophobia is fear of love
● 5-aryl-1,4-benzodiazepines
● Halogen or nitro group in 7 is required for sedative
GAD-Generalized Anxiety Disorder hypnotic activity
● Excessive and uncontrollable anxiety and worry
● Anxiety and worry, associated with three of the following
symptoms:

96
PCOL 211 - Pharmacology 1

● Drugs of choice for treating GAD Metabolism


● Many benzodiazepines are converted to
N-desmethyldiazepam (N-DMDZ), an active metabolite
with a long elimination t1/2 of more than 40 hours

● Bind to GABAA receptor subunits at CNS


● Increase frequency of chloride channel opening
enhancing membrane hyperpolarization (relaxation)

Pharmacological e ects Benzodiazepines

1. Sedative

● At low doses
● exert calming effects with concomitant reduction of anxiety
● Clinical uses: Anxiety (GAD) and panic disorders,
phobias
● Behavioral disinhibitory effects of sedative-hypnotics
1. Short-acting (2-8 hrs)
1. Euphoria - “Being in a happy world”
● Oxazepam (Serax) 2. Impaired judgment
● Triazolam (Halcion) 3. Loss of self control
● Clonazepam (Klonopin, Rivotril)
● Midazolam (Versed, Dormicum) 2. Hypnotic
● Temazepam (Restoril) ● at high doses
● Clinical uses: for insomnia
2. Intermediate acting (10-20 hrs)
● Lorazepam (Ativan) 3. Anesthesia
● Alprazolam (Xanax, Xanor) ● at high doses: CNS depression to the point known as
● Temazepam (Restoril) stage III of GA
● Flunitrazepam (Rohypnol) ● Clinical uses:
○ for induction of anesthesia: Midazolam
3. Long- acting (1-3 days)
○ for IV anesthesia: Diazepam, Lorazepam
● Diazepam (Valium, Anxionil)
● Flurazepam(Dalmane) 4. Anticonvulsant
● Chlordiazepoxide (Librium) ● Can exert anticonvulsant effects without marked central
nervous system depression, so that mental and
USES
physiologic activity are relatively unaffected.
● Anxiety - alprazolam, diazepam ● Clinical use: some forms of epilepsy (Clonazepam,
● Seizures - diazepam, clonazepam, lorazepam Nitrazepam, Lorazepam, Diazepam)
● Insomnia - flurazepam, midazolam
● Pre-operative sedation - midazolam 5. Muscle relaxant
● Carbamates and BZDs exert inhibitory effects on
Pharmacokinetics
polysynaptic reflexes and internuncial transmission and at
● ABSORPTION and DISTRIBUTION: high doses may also depress transmission at the skeletal
○ Absorbed rapidly following oral administration neuromuscular junction
○ Cross the BBB, placental barrier ● Clinical uses: Muscle spasms, spastic disorders
○ Secreted in milk
6. Respiratory and cardiovascular depression
● Due to Medullary vasomotor center depression
● Marked effects when given IV

97
PCOL 211 - Pharmacology 1

● Phenobarbital - for mixed Bzd and alcohol dependence.


Adverse E ects
Benzodiazepine Poisoning
● CNS depression - most common ADR
● Drowsiness ● DOC : Flumazenil (Anexate)-
● Sedation ● A benzodiazepine antagonist
● Anterograde amnesia ● GABA receptor antagonist
○ Loss of the ability to create new memories after the
Drug Interactions
event that caused the amnesia, leading to a partial or
complete inability to recall the recent past, while ● Alcohol + BZD
long-term memories before the event remain intact. ○ additive effect; lowers the therapeutic index of BZD
● Psychomotor impairment ● CNS depressants + BZD
● Ataxia ○ Synergism of adverse sedative effects
● Lorazepam + Clozapine
Short-term e ects
○ Respiratory suppression and death
● Disorientation ● Cimetidine + BZD
● Confusion ○ of BZ metabolism inhibition
● Irritability ● Nefazodone/Fluvoxamine
● Aggression ○ increased alprazolam concentrations.
● Excitement
BARBITURATES
Long-term e ects
● former DOC for anxiety, insomnia
● Tolerance ● barbituric acid is 2,4,6 trioxohexahydropyrimidine
○ Decreased responsiveness to a drug following ● MOA: Barbiturates increase the duration of GABA
repeated exposure mediated chloride ion channel opening
○ Result in the need for an increase in the dose required ○ They have GABA action → ↑opening time of chloride
to maintain symptomatic improvement or to promote channels → ↑conductance of chloride ions →
sleep hyperpolarization
● Dependence
● Bind to GABAA receptor subunits at CNS neuronal
○ An altered physiologic state that requires continuous synapse
drug administration to prevent abstinence or ● Increase Duration of chloride channel opening
withdrawal syndrome enhancing membrane hyperpolarization
Benzodiazepine Discontinuation Withdrawal Syndrome:
● Withdrawal symptoms may persist for days to weeks.
● Common symptoms of withdrawal include:
○ Anxiety
○ Insomnia
○ Restlessness
○ Agitation
○ irritability

Withdrawal symptoms
● Less frequently occurring symptoms:
○ Nausea
○ Malaise
○ Coryza
○ Blurred vision
○ Diaphoresis
○ Nightmares
○ Hyperreflexia
○ Ataxia
● Rarely occurring symptoms:
○ Tinnitus
○ Confusion
○ Paranoid delusions
○ Hallucinations
○ Seizures
○ Psychosis

To treat withdrawal symptoms:


● Diazepam - can be initiated as loading dose (40% of daily
consumption), followed by a daily tapering of 10%.
● Clonazepam - an alternative agent

98
PCOL 211 - Pharmacology 1

■ (Ciprofloxacin, Fluvoxamine, Tacrine,


1. Ultra-short (20 min) - Thiopental (Pentothal)
● Thiopental - more lipid-soluble, rapid CNS Zileuton)
penetration ○ Rifampin
2. Short-acting (3-8 h):
BUSPIRONE
● Pentobarbital (Nembutal)
● Amobarbital (Amytal) ● Serotonin agonist
3. Long-acting (1-2 d): Phenobarbital (Luminal) ○ azapirone anxiolytic
4. Intermediate-acting: ○ Possess no anti-convulsant, muscle relaxant,
● Amobarbital hypnotic, motor impairment and dependence
● Butabarbital properties.
● As a serotonin partial agonist, it binds presynaptically to
USES the receptors in the dorsal raphe and postsynaptically to
receptors in hippocampus and cortical brain areas.
● Induction of anesthesia -Thiopental ● It also possesses both dopamine agonist and indirect
● Seizures in children - Phenobarbital dopamine antagonist properties.
● Anxiety - Pentobarbital, Amobarbital ● It is an alternative for GAD to patients who cannot
tolerate the sedating effects and psychomotor impairment
of BZs.
● The agent of choice in the management of chronic,
persistent anxiety.
● BZ dose should be tapered before switching to buspirone.
● It is an appropriate choice for anxious patients with a
history of alcohol or drug abuse.

ADRENERGIC BLOCKING AGENTS


● Propranolol and other beta-blockers may be useful in
patients with prominent cardiovascular symptoms of
anxiety.
● They are less effective anxiolytics than BZs, and their
usefulness may be restricted to those anxiety patients with
E ects physical symptoms, especially cardiovascular
● Addicting complaints, have not adequately responded to BZ
● Dependence therapy.
● Phenobarbital is capable of inducing the hepatic ● Propranolol,10 mg bid should be used initially and
microsomal drug-metabolizing enzyme system (P-450) gradually titrated to anxiolytic response.
● Discontinuation should be gradual to avoid rebound
Newer Hypnotics (used in sleep disorders) anxiety and cardiovascular effects.
● Eszopiclone (Lunesta ™)
Old Sedative-Hypnotics
○ cyclopyrrolone
● Zaleplon (Sonata ™) ● Alcohols
○ Pyrazolopyrimidine ○ Ethchlorvynol
● Zolpidem (Ambien™) ○ Chloral Hydrate (Aquachloral)
○ Imidazopyridine ● Piperidinediones
● MOA: bind selectively to GABAA receptor isoforms that ○ Gluthethimide
contain α1 subunits to enhance hyperpolarization ○ Methyprylon
● Carbamates
Ramelteon (Rozerem™) ○ meprobamate
● Melatonin Receptor Agonist ● CHLORAL HYDRATE
● MOA: Activates MT1 and MT2 receptors in ○ Associated to MICKEY FINN
suprachiasmatic nuclei in the CNS ○ Trichloroethanol – active metabolite
● Rapid onset of effect with minimal rebound insomnia ○ Trichloroacetic acid – toxic metabolite
● Used with caution in patients with liver dysfunction ■ Accumulate with the nightly administration of the
● ADRs: drug
○ Dizziness
Others Sedative-Hypnotics
○ Somnolence
○ Fatigue 1. Zolpidem (Ambien, Stilnox)
○ Endocrine changes ● not a Bz but acts on Bz receptor
■ decrease testosterone 2. Chloral hydrate
■ Increase prolactin ● ‘knockout drops’
● Metabolized by CYP1A2 ● converted to trichloroethanol (active)
● Drug interactions: ● for preoperative sedation
○ CYP1A2 inhibitors

99
PCOL 211 - Pharmacology 1

3. Antihistamines
● diphenhydramine (Benadryl), doxylamine (Unisom), DRUGS FOR SEIZURE
hydroxyzine (Atarax, Iterax)

The Physiology of the Nerve Impulse

Nerve Impulse
● Mostly electrical impulse
● Nerve cell fiber resting potential
○ Na+ concentration higher on outside
○ K+ concentration higher on inside
○ Negative charge on inside
○ Positive charge on outside
● Way of communication within cells
● Positive charge - depolarization (action potential)
● Negative charge - hyperpolarization
● Potassium In, Sodium Out
● Magnesium In, Calcium Out

● Depolarization - happens when Na channel open and


there is an influx sodium
● Repolarization - K efflux
○ Ca channel closes
○ K will stay inside
○ Na efflux
○ Ca influx
○ K attracts Cl resulting to resting potential and
hyperpolarization

● Depolarization (+ charge intracellular)


○ Na+ rush inside cell (influx)
● Repolarization (balance yung charge sa intra and extra)
○ K+ rush out to restore balance
● Depolarization - Repolarization moves in one direction
● Hyperpolarization (more - charge intracellular)

100
PCOL 211 - Pharmacology 1

● The patient typically exhibits lethargy, confusion, and


behavioral changes.

EPILEPSY
● a chronic seizure disorder, or group of disorders,
characterized by seizures that usually recur
unpredictably in the absence of a consistent provoking
factor.
○ Seizures - neurons synchronously active
● derived from the Greek word meaning “to seize upon” or
“taking hold of.”
● first described by Hughlings Jackson as an intermittent
derangement of the nervous system due to a sudden,
excessive, disorderly discharge of cerebral neurons.

International Classification of Epileptic Seizures

● Electroencephalogram - detect ↑ NE, serotonin,


dopamine. Used to determine if may epilepsy, seizure,
convulsion

● Seizures (pwedeng magcause ng sakit)


○ Excessive abnormal electrical discharge from
cortical neurons
○ Causes: idiopathic, CNS infection, fever, metabolic
disturbance, cerebral trauma
○ A seizure is the physical findings or changes in
behavior that occur after an episode of abnormal
electrical activity in the brain. The term "seizure" is
often used interchangeably with "convulsion.“ ● Generalized - usually caused by head trauma
● Convulsions occur when a person's body shakes (accidents)
rapidly and uncontrollably. ● Absence - absent minded (tulala)
○ violent, involuntary contractions of the voluntary ● Myoclonic - nanginginig, contractions of muscles
muscles ● Tonic clonic - matigas tas nanginginig; usually in
○ need continuous treatment (1-3 years tetanus
anticonvulsant drugs) ● Tonic - matigas yung muscle
● Epilepsy ● Atonic - walang movement, biglang namlalambot
○ recurrent unprovoked seizures
○ hard to treat
DEFINITION OF TERMS:
○ Life long disorder
○ ↑ in neurotransmitter (excitatory) ● JERKING → o throw or toss with a quick abrupt motion.
○ ↓ in inhibitory neurotransmitter ● PARESTHESIA → an abnormal sensation, typically
tingling or pricking
PHASES OF SEIZURE ● DYSPHASIA → language disorder marked by deficiency in
the generation of speech
● It should happen in less than 1 minute
○ Tongue tied
○ If more than that, the brain will not send electrical
● Deja vu → rare movements when the present feels like the
charges to heart and lungs (neurotransmission)
past
1. Prodrome
I. PARTIAL SEIZURES
● May precede the seizure by hours or days.
● Half of the brain is damaged
● Unusual behavior
● most common seizure type
● Aura (subjective sensation)
● occurring in approximately 80% of epileptic patients.
● In behavior or mood typically occur during the prodrome.
● clinical and EEG changes indicate initial activation of a
● This phase may include an aura – subjective sensation,
system of neurons limited to part of one cerebral
such as unusual smell or flashing light
hemisphere that may spread to other or all brain areas.
2. Ictal Phase ● Focal area is nadadamay
○ Left - hindi na nakakapag produce ng word
● The seizure itself. In some cases, a scream or cry heralds
○ Right - nalelessen yung creativity
its onset.
CLASSIFICATION OF PARTIAL SEIZURE
3. Postictal Phase
● Takes place immediately after the seizure. 1. Simple Partial Seizures
● Px exhibits lethargy
● generally do not cause loss of consciousness.
● Confused
● There are consciousness but there are motor symptoms

101
PCOL 211 - Pharmacology 1

● Gising pero nagseseizure


2. Absence “Petitmal” Brief loss of consciousness
● SIGNS AND SYMPTOMS:
○ Motor Signs: convulsive jerking, chewing motions 3. Myoclonic Sporadic (isolated), jerking
and lip smackling. movements
○ Sensory and somatosensory manifestations:
paresthesias and auras. 4. Clonic Repetive, jerking
○ Autonomic signs: sweating, flushing, and pupil movements
dilation.
○ Behavioral manifestations: sometimes accompanied 5. Tonic Muscle stifness, rigidity
by impaired consciousness, includes:
6. Atonic “Drop attacks” Loss of muscle tone
■ déjà vu experiences
■ structured hallucinations
■ dysphasia. Generalized Seizures
1. ABSENCE (Petit mal) SEIZURES
● Tulala
● Present as alterations of consciousness (absences)
lasting 10-30 seconds.
● Staring (with occasional eye blinking) and loss or
reduction in postural tone is typical.
● Enuresis - biglang naiihi
● Pwedeng masundan ng tonic (30-60 secs) → clonic
(30-60 secs) → post ictal phase (lethargy)
2. MYOCLONIC SEIZURES (bilateral massive epileptic
myoclonus)
● present as involuntary jerking of the facial, limb, or
trunk muscles, possibly in rhythmic manner.
● consist of sporadic jerks, usually on both sides of the
body. Patients sometimes describe the jerks as brief
electrical shocks. When violent, these seizures may
2. Complex Partial Seizures result in dropping or involuntarily throwing objects
3. CLONIC SEIZURES
● Accompanied by impaired consciousness; however in
● characterized by sustained muscle contractions
some cases, the impairment precedes or follows the
alternating with relaxation
seizure.
● are repetitive, rhythmic jerks that involve both sides
● Hinimatay tas nagseizure
of the body at the same time.
● MANIFESTATIONS:
4. TONIC SEIZURES
○ Purposeless behavior is common.
● involve sustained tonic muscle extension (stiffening).
○ The affected person may have a glassy stare, may
5. GENERALIZED TONIC-CLONIC SEIZURES (grand mal)
wander about aimlessly, and may speak unintelligibly.
● cause sudden loss of consciousness.
○ Psychomotor (temporal lobe) epilepsy may lead to
● Tongue depressor
aggressive behavior (e.g., outbursts of rage or
6. ATONIC SEIZURES (drop attacks)
violence).
● characterized by a sudden loss of postural tone so
■ Biglang sumisigaw
that the individual falls to the ground. They occur
● Postictal confusion usually persists for 1-2 minutes after
primarily in children (below 2 years old)
the seizure ends.
● Automatism (e.g., picking at clothes) is common and may Generalized Tonic-Clonic Seizures (grand mal)
follow visual, auditory or olfactory hallucinations). ● The individual becomes rigid and falls to the ground.
○ Action performed unconciously Respirations are interrupted. The leg extended, and the
II. GENERALIZED SEIZURES back arches; contraction of the diaphragm may induce
grunting. This tonic phase lasts for about 1 minute.
● Entire brain is involved
○ Sardonic smile
● motor manifestations are bilateral.
● A clonic phase follows, marked by rapid bilateral muscle
● diffuse, affecting both cerebral hemispheres.
jerking, muscle flaccidity, and hyperventilation.
● clinical and EEG changes indicate initial involvement of
Incontinence, tongue biting, tachycardia, and heavy
both hemispheres.
salivation sometimes occur.
Generalized Seizures Symptoms ● Postictal phase
(produced by the entire ○ the individual may experience headache, confusion,
brain) disorientation, nausea, drowsiness, and muscle
soreness. This phase may last for hours.
1. “Grand Mal” or Unconsciousness, ● Some epileptics have serial grand mal seizures, regaining
Generalized convulsions, muscle rigidity
consciousness briefly between attacks. In some cases,
tonic-clonic
grand mal seizures occur repeatedly with no recovery of
consciousness between attacks (status epilepticus)

102
PCOL 211 - Pharmacology 1

○ Status epilepticus DOC: Diazepam IV ○ Multiple-drug therapy may be required.


● Achieve steady-state kinetics
CLASSIFICATION BASED ON ETIOLOGY
● Monitor plasma drug levels
● PRIMARY (idiopathic) SEIZURES ● Avoid sudden withdrawal
○ have no identifiable cause.
PHARMACOKINETICS
○ “genetic”
● SECONDARY SEIZURES (symptomatic or acquired ● Most classical antiepileptic drugs exhibit similar
seizures) pharmacokinetic properties.
○ occur secondary to an identifiable cause. ● Good absorption.
○ Infection accidents ● Low plasma protein binding (except for phenytoin,
BDZs, valproate, and tiagabine).
SECONDARY SEIZURES
● Conversion to active metabolites (carbamazepine,
● Intracranial neoplasms (cancer sa brain) primidone, fosphenytoin)
● Infectious diseases (meningitis, influenza, toxoplasmosis, ○ Carbamazepine - enzyme inducer
mumps, measles, syphilis) ● Cleared by the liver
● High fever (in children) (39-42°) ● Plasma clearance is slow.
● Head trauma ● At high concentrations phenytoin exhibits zero order
● Congenital diseases kinetics.
● Metabolic disorders (hypoglycemia, hypocalcemia)
Notable Adverse E ects of Antiseizure Medications
● Alcohol or drug withdrawal
● Lipid storage disorders ● Nausea and vomiting (caused by inhibiting effect of GABA)
● Developmental abnormalities (autism) ● Drowsiness-sedation (caused by inhibiting effect of GABA)
● Ataxia - less muscle tone (caused by inhibiting effect of
ANTI-EPILEPTIC DRUGS
GABA)
● Rash
● Hyponatremia (because of Na blockade)
● Weight gain or weight loss
● Teratogenicity
● Osteoporosis

Treatment of Seizures
● Strategies:
○ Modification of ion conductances.
○ Increase inhibitory (GABAergic) transmission.
○ Decrease excitatory (glutamatergic) activity.

Drug Treatment of Seizures


● Life-long treatment may be necessary.
● NMDA receptor - if blocked, no muscle contraction ● It may take weeks to establish adequate drug plasma
levels and to determine the adequacy of therapeutic
improvement.
● Lack of compliance is responsible for many treatment
failures.

ANTICONVULSIVE DRUG CLASSIFICATION


● BARBITURATES
○ Phenobarbital - for seizures especially in infants
○ Primidone
○ Mephobarbital
● HYDANTOINS
○ Phenytoin
○ Mephenytoin
○ Ethotoin
○ Fosphenytoin - converted into Phenytoin after it
metabolized
TREATMENT
● SUCCIMIDES
● Try to find a cause. (e.g. fever, head trauma, drug abuse) ○ Ethosuximide
● Therapy is aimed at control (drugs do not cure) ○ Methsuximide
● The type of seizure determines the choice of drug! ○ Phensuximide
● More than 80% of patients with epilepsy can have their ○ Zonisamide
seizures controlled with medications. ● OXAZOLIDINEDIONES
● Monotherapy with anticonvulsant ○ Paramethadione
○ Increase dose gradually until seizures are ○ Trimethadione
controlled

103
PCOL 211 - Pharmacology 1

● BENZODIAZEPINE ○ Inhibition of calcium channels.


○ Clonazepam
Na+ Channel Inhibitors
○ Diazepam
● MISCELLANEOUS
○ Lamotrigine
○ Felbamate
○ Gabapentin
○ Carbamazepine
○ Valproic acid
○ Phenacemide
○ Topiramate
○ Tiagabine ● Blocks voltage-gated sodium channels by selectively
○ Levetiracetam binding to the channel in the inactive state and slowing its
rate of recovery
Indications
Na+ CHANNEL BLOCKERS
1. GTC and partial seizures
● valproic acid, carbamazepine, phenytoin ● Phenytoin (Dilantin)
● GTC - generalized tonic clonic (formerly known as ● Fosphenytoin (Cerebyx)
grandmal seizure) ● Carbamazepine (Tegretol)
2. Absence ○ Cause its own metabolism
● ethosuximide, valproic acid (absence with tonic clonic) ○ Enzyme inducer
3. Myoclonic
○ Auto inducer
● clonazepam, valproic acid
4. Status epilepticus (life threatening seizure) ● Oxcarbazepine (Trileptal)
● Diazepam (adults), lorazepam, phenytoin ● Valproic Acid (Valproate; Depakene, Depakote)
5. Febrile seizures (in children) ● Lamotrigine (Lamictal)
● phenobarbital ● Topiramate (Topamax)
6. Tonic Clonic ● Zonisamide (Zonegran)
● Lamotrigene, VPA, Topiramide ● Lacosamide (Vimpat)
Classification of Epilepsies and Drug Selection ● Rufinamide (Banzel)
● Partial seizures 1. Phenytoin (Dilantin, Epilantin)
○ Carbamazepine, phenytoin
● MOA: closes Na channels
○ Valproic acid, lamotrigine, gabapentin,
● Oldest nonsedative antiepileptic drug.
benzodiazepines, barbiturates
● S/E:
○ Adjunct: Tiagabine, topiramate, levetiracetam,
○ CNS: ataxia, nystagmus, diplopia
zonisamide
○ Connective: hirsutism, gingival hyperplasia
● Generalized seizures:
(20-40%)
○ Tonic-clonic (grand mal):
○ “Fetal hydantoin syndrome”- cleft palate, congenital
■ Carbamazepine, phenytoin
heart disease, microcephaly, growth and mental
■ Valproic acid, lamotrigine, gabapentin,
retardation
benzodiazepines, barbiturates
○ DERMATOLOGICAL EFFECTS maculopapular
■ Adjunct: Topiramate, zonisamide
rashes sometimes with fever, SJS, and lupus
○ Absence (petit mal):
erythematosus.
■ Ethosuximide
○ CyP450 inducer (carbamazepine, valproate, warfarin,
■ Valproic acid (when absence seizures coexist with
OCPs)
tonic-clonic seizures)
○ Displaced from protein binding by aspirin,
■ Clonazepam
sulfonamides
■ Adjunct: Lamotrigine, benzodiazepines
● No effect on infantile spasms or absence seizures
○ Myoclonic syndromes:
■ Valproic acid Phenytoin Induced Gingival Hyperplasia
■ Clonazepam and other benzodiazepines
● 17 year old boy treated with 300mg/day phenytoin for 2
■ Adjunct: levetiracetam
years (unsupervised)
○ Status epilepticus:
● Partial recovery at 3 months after discontinuation
■ Treatment is intravenous diazepam or
lorazepam followed by intravenous Phenytoin (Dilantin)
fosphenytoin (or phenytoin) or phenobarbital (for
● Adverse Effects:
children).
1. CNS EFFECTS – ataxia, (limiting side effect),
Mechanism of Action dysarthria, and insomnia.
● transient hyperkinesia may follow IV phenytoin
● 3 main categories of therapeutics:
infusion.
○ Inhibition of voltage-gated Na+ channels to slow
● Alcoholic beverages should be avoided while on
neuron firing.
this medication.
○ Enhancement of the inhibitory effects of the
2. Decreased serum concentrations of folic acid,
neurotransmitter GABA.
thyroxine (T4), and vitamin K with long-term use.

104
PCOL 211 - Pharmacology 1

3. “Fetal hydantoin syndrome”: ● Rare: Idiosyncratic blood dyscrasias and severe rashes
● includes growth retardation, microencephaly, and
Carbamazepine Drug Interactions
craniofacial abnormalities (e.g., cleft palate) and
is possibly due to an epoxide metabolite of ● CBZ metabolism is affected by many drugs, and CBZ
phenytoin. affects the metabolism of many drugs.
● Determination of plasma levels and clearance may be
Congeners of Phenytoin
necessary for optimum therapy.
● Mephenytoin (Mesantoin) ● Exhibits AUTOINDUCTION
○ (withdrawn form the market) ○ Metabolize itself
● Ethotoin (peganone)
● Phenacemide 4. Oxcarbazepine (Trileptal):

● NIRVANOL ● FDA approved in 2000 for partial seizures


○ Metabolite that contributes most of the antiseizure ○ Complex partial seizures
activity of mephenytoin. ○ Primary & secondarily generalized tonic-clonic
seizures
2. Fosphenytoin ● Is a prodrug whose actions are similar to those of
carbamazepine; it has a short half-life of 1—2 hour.
● is a prodrug
● Can be administered IM ● Its activity is due to a 10-hydroxy metabolite with a
● rapidly converted to phenytoin in the blood (↑phenytoin), half-life of 10 hours.
providing high levels of phenytoin within minutes. ● Fewer adverse effects than CBZ, phenytoin
● may also be administered intramuscularly (IM). ● Eslicarbazine (ESL) (Stedesa)
● Phenytoin sodium should never be given IM because it ○ Prodrug
can cause tissue damage and necrosis.
○ Advantage: once daily dosing regimen
● Fosphenytoin is the drug of choice and standard of care
for IV and IM administration. 5. Valproic Acid (Valproate; Depakene, Depakote)
● Due to sound-alike and look-alike names, there is a risk for
medication error to occur. ● Closes Na channels
○ fosphenytoin tradename is Cerebyx® ● Other Mechanisms of Action:
○ Celebrex®, the cyclooxygenase-2 inhibitor - 1. Some inhibition of T-type Ca2+ channels.
Celecoxib
● T-type - low voltage Ca channel
○ Celexa®, the antidepressant. - Cetalopram
2. Increases GABA production and decreases GABA
3. Carbamazepine (Tegretol) metabolism. (Inhibition of GABA transaminase )
● Also used for trigeminal neuralgia ● 90% protein bound- displaced by phenytoin and aspirin
○ Physical jerk of the face ● S/E:
○ cramping/numbness ○ GI disturbance, rare pancreatitis and hepatotoxicity,
● MOA: closes Na channels sedation and ataxia at high doses,
● First choice for complex partial and generalized ○ fetal malformation (spina bifida)
tonic-clonic seizures. ○ CyP450 inhibitor (phenytoin, carbamazepine,
● Tricyclic, antidepressant (bipolar) Phenobarbital)
● S/E: ● Indications:
○ CNS effects: dizziness, ataxia, ○ Simple or complex partial, & primary generalized
○ GI: nausea, vomiting tonicclonic
○ Metabolic: hyponatremia ○ Also used for absence, myoclonic, and atonic
○ Hematopoietic: leukopenia seizures.
○ Derma: rashes, SJS ○ Highly effective for photosensitive epilepsy and
○ CyP450 inducer (warfarin, phenytoin, valproate, juvenile myoclonic epilepsy.
OCPs), autoinducer (induces its own metabolism) ● Contraindications:
○ Liver disease
Signs of an Attack of Tic Douloureux or Trigeminal ● Adverse Effects:
Neuralgia (TN) ○ Weight gain (30-50%)
● 5th cranial nerve ○ Dose-related tremor
● It is important that you and your caregiver get acquainted ○ Transient hair loss
with the signs of an attack of a patient suffering from Tic ○ Polycystic ovary syndrome and menstrual
Douloureux or Trigeminal Neuralgia disturbances
1. A physical jerk ○ Bone loss
2. Sudden immobility or silence ○ Ankle swelling
3. Involuntary facial twitches 6. Lamotrigine (Lamictal):
4. A sudden shout of pain
● Other Mechanism of Action:
Carbamazepine Adverse E ects ○ Also inhibits voltage gated calcium channels (N and
● Common: Diplopia and ataxia (most common), P/Q type)
gastrointestinal disturbances; sedation at high doses ○ May inhibit synaptic release of glutamate.
● Occasional: Retention of water and hyponatremia; rash, ● Adverse Effects:
agitation in children ○ Rash (10%)

105
PCOL 211 - Pharmacology 1

■ Rare progression to serious systemic illness ○ Abnormal thinking


○ Increased alertness ○ Nervousness
● Indications: ○ Agitation/irritability
○ Adjunct therapy (ages 2 & up): ○ Usually well tolerated
■ Simple & complex partial seizures
9. Lacosamide (Vimpat)
■ Generalized seizures of Lennox-Gastaut
Syndrome ● Amino acid related compound
○ Monotherapy (adults): ● Also binds to collapsin-response mediator protein
■ Simple & complex partial seizures (CRMP) thereby blocking the effect of neurotrophic
● Contraindications: May make myoclonic seizures worse. factors such as BDNF (Brain Derived Neurotropic
Factor) and NT3 (Neurotropin) on axonal and dendritic
Lennox-Gastaut Syndrome
growth
● is the most distressing of childhood epilepsies. ○ Brain derived neurotrophic factor
● The children suffer frequent fits of many different types,
10. Rufinamide (Banzel)
and experience gradual mental deterioration.
● Infantile spasms often evolve into Lennox Gastaut ● Triazole derivative
syndrome, and the age of onset is from 1 to 8 years, ● Prolong the inactive state of the sodium channel
peaking at 3 to 5 years. ● Adjunct for Lennox-Gastaut syndrome
● Lennox-Gastaut syndrome is a form of severe epilepsy
that begins in childhood. It is characterized by multiple ENHANCEMENT OF GABA (1st MOA)
types of seizures and intellectual disability. People with 1. Barbiturate drugs: DURATION
Lennox Gastaut syndrome begin having frequent seizures ● MOA: Bind to GABA-A receptor at the post synapse to
in early childhood, usually between ages 3 and 5. increase the duration of opening the Cl- channel
a. Phenobarbital (Luminal)
7. Topiramate (Topamax)
b. Primidone (Mysoline)
● Other Mechanism of Action: 2. Benzodiazepine drugs: FREQUENCY
○ Enhances post-synaptic GABA receptor currents. ● MOA: Bind to GABA-A receptor to increase the
○ Kainate receptor antagonist (blocks a certain type of frequency of opening the Cl- channel
glutamate channel) a. Diazepam (Valium)
● S/E: Incidence of renal stone formation or renal calculi in b. Lorazepam (Ativan)
older patients is associated c. Clonazepam (Klonopin)
● Indications: d. Clorazepate (Transxene-SD)
○ Adjunct therapy for partial and primary generalized ● GABA ENHANCERS:
○ seizures in adults and children over 2. 3. Tiagabine (Gabitril)
○ Decreases tonic and atonic seizures in children with 4. Stiripentol (Diacomit)
Lennox-Gastaut syndrome. 5. Vigabatrin (Sabril):
● Contraindications:
Barbiturate Drugs
○ History of kidney stones
● Drug Interactions: ● Phenobarbital (Luminal): (For infants)
○ CBZ, phenytoin, phenobarbital, & primidone ○ Indications:
decrease blood levels ■ Second choice for partial and generalized
tonicclonic seizures
8. Zonisamide (Zonegran) ■ Rapid absorption has made it a common choice
● is a sulfonamide derivative that has a broad spectrum of for seizures in infants, but adverse cognitive
action effects cause it to be used less in older children
● Other Mechanism of Action: and adults.
○ Inhibits T-type Ca2+ currents (low voltage) ■ Status epilepticus
○ Binds to GABA receptors. ● Primidone (Mysoline):
○ Facilitates dopaminergic and serotonergic ○ 2-desoxyphenobarbital
neurotransmission. ○ was metabolized to phenobarbital and
● Indications: phenylethylmalonamide (PEMA)
○ Approved for adjunct treatment of partial seizures in ENHANCEMENT OF GABA INHIBITION (2nd MOA)
adults.
○ Appears to have a broad spectrum: 1. Barbiturate Drugs
■ Myoclonic seizures
■ Infantile spasms ● Primidone (Mysoline):
■ Generalized & atypical absence seizures ○ Indications:
■ Lennox-Gastaut Syndrome ■ Adjuvant or monotherapyfor partial and
● Drug Interactions: generalized tonic-clonic seizures
○ Phenytoin and carbamazepine decrease its half-life by ■ May control refractory generalized tonic-clonic
half. seizures
● Adverse Effects: ○ Contraindications:
○ Weight loss

106
PCOL 211 - Pharmacology 1

■ History of porphyria (liver disease; group of rare ● Potent inhibitor of CYP3A4, CYP1A2, CYP2C19
disorder in which substances called porphyrins
5. Vigabatrin (Sabril)
build up in the body negatively affecting the skin
and nervous system) ● Irreversible inhibitor of GABA aminotransferase
● Phenobarbital (Luminal) & Primidone (Mysoline): (GABA-T)
○ Drug Interactions: ● Also inhibits the vesicular GABA transporter
■ Other CNS depressants ● Uses: partial seizure and infantile spasms (reserve drug)
● Can lead to depression coma or even death ● ADRs:
■ Increased metabolism of vitamin D (↓bone ○ Typical: drowsiness, dizziness, weight gain
formation) and K (↓formation of clotting factor) ○ Irreversible lesion in the retina
■ Phenytoin increases the conversion of primidone
CALCIUM CHANNEL BLOCKERS (3rd MOA)
to phenobarbital.
● Phenobarbital & Primidone ● Inhibit lowthreshold (T-type) Ca 2+ currents, especially
○ Adverse Effects: in thalamic neurons that act as pacemakers to generate
■ Agitation and confusion in the elderly. rhythmic cortical discharge.
■ Worsening of pre-existing hyperactivity and 1. Ethosuximide (Zarontin)
aggressiveness in children 2. Gabapentin (Neurontin)
■ Sexual side effects 3. Phensuximide, Methsuximide (Celontin) (withdrawn
■ Physical dependence from the market)
4. Oxazolidinediones → Paramethadione, Dimethadione
2. Benzodiazepine Drugs Trimethadione
● Indications: 5. Gabapentin (Neurontin)
○ Only clonazepam & clorazepate approved for 6. Pregabalin (Lyrica)
long-term treatment.
Voltage-Gated Ca2+ Channel T Currents
○ Clorazepate
■ In combination for partial seizures 1. Ethosuximide (Zarontin):
○ Clonazepam ○ Mechanism of Action:
■ Lennox-Gastaut Syndrome, myoclonic, atonic, ■ Reduces low -threshold Ca2+ currents (T
and absence seizures currents) in the thalamic neurons.
■ Tolerance develops after about 6 months ■ Half-life is ~60 hrin adults; ~30hr in children.
○ Diazepam and lorazepam are used in treatment of ○ Indications:
status epilepticus. ■ First line for absence seizures
■ Diazepam is painful to inject ○ Contraindications:
■ Lorazepam is more commonly used in acute ■ May exacerbate partial & tonic-clonic seizures
treatment. ○ Adverse Effects:
○ Diazepam + Phenytoin ■ Psychotic behavior
■ Intermittent use for control of seizure clusters ■ Blood dyscrasias
■ Diazepam frequently combined with phenytoin. ■ Persistent headaches
● Contraindications: ■ Anorexia
○ Diazepam in children under 9 ■ Hiccups
○ Narrow angle glaucoma ■ Lupus-like syndromes (butterfly rash)
○ Toxicity:
3. Tiagabine (Gabitril) ■ parkinson-like symptoms
● Mechanism of Action: ■ photophobia
○ Inhibition of GABA transporter (GAT-1) – reduces 2. Oxazolidinediones
reuptake of GABA by neurons and glial cells. ● Paramethadione, Dimethadione
● Indications: ● Trimethadione
○ Approved in 1998 as an adjunct therapy for partial ○ Use in absence seizure
seizures in patients at least 12 years old. ○ Raises the threshold for seizure discharges
● Contraindications: after repetitive thalamic stimulation (same effect
○ Absence seizures like ethosuximide)
● Interactions: ○ ADR: sedation
○ Blood levels decreased by CBZ, phenytoin,
Blockade of Calcium Channels
phenobarbital, & primidone
● Adverse Effects: ● Gabapentin (Neurontin): (For adult)
○ Asthenia (weakness) ○ Mechanism of Action:
○ Abdominal pain ■ It blocks the PQ type Ca+ channel
● PQ type - high voltage
4. Stiripentol (Diacomit) ■ Originally designed to be a centrally acting
● Adjunct therapy for refractory generalized tonic clonic GABA agonist.
seizure in patients with severe myoclonic epilepsy in ○ Indications:
infancy ■ adjunct therapy in adults and children with partial
● Barbiturate-like effect & secondarily generalized seizures.

107
PCOL 211 - Pharmacology 1

■ Also effective as monotherapy.


Magnesium Sulfate
○ Adverse Effects:
■ Weight Gain (5%) with ankle edema ● for eclampsia (HTN + proteinuria + seizures)
■ Irritability ● S/E: CNS, cardiovascular and respiratory depression
■ Behavioral problems in children (6%) ● Antidote: Calcium chloride/gluconate
■ Has been associated with movement disorders.
○ Contraindications:
■ Can exacerbate myoclonic & absence seizures.
● Pregabalin (Lyrica):
○ Mechanism of Action:
■ Same as gabapentin; Inhibits PQ type Ca+
channel
○ Indications:
■ Approved in 2005
■ Adjunct therapy for partial & secondarily
generalized seizures
○ Other uses:
■ Prescribed for neuropathic pain, fibromyalgia (A
syndrome characterized by chronic pain in the
muscles of soft tissues surrounding joints, fatigue,
and tenderness at specific sites in the body)

Block the N-methyl-D-aspartate (NMDA) glutamate


receptor
● Felbamate (Felbatol)
○ has a broad spectrum of anticonvulsant action.
○ The drug has multiple proposed mechanisms
including
■ blocking voltage-dependent sodium channels
■ competing with the glycine-coagonist binding site
on the N-methyl-D-aspartate (NMDA) glutamate
receptor
■ blocking calcium channels
■ potentiation of GABA actions.
● Felbamate
○ It is reserved for use in refractory epilepsies
(particularly Lennox-Gastaut syndrome) because of
the risk of aplastic anemia (about 1:4000) and
hepatic failure.

Others/Unknown MOA
● Levetiracetam (Keppra):
○ Bind selectively to the synaptic vesicular protein SV2A
○ Modifies release of glutamate and GABA
○ Indications:
■ Approved in 1999 as an adjunct therapy for adults
with partial seizures.
■ Some patients have success with monotherapy
● Levetiracetam (Keppra):
○ Contraindications:
■ Renal dysfunction
○ Adverse Effects:
■ Asthenia
■ Infection
■ Behavioral problems in children
● Retigabine
○ Also ezogabine
○ A potassium channel facilitator
○ ADRs:
■ Dizzines, somnolence, blurred vision, confusion,
dysarthria, bladder dysfunction

108
PCOL 211 - Pharmacology 1

○ Moving
○ Losing consciousness
● Only experienced by patients
○ Fears
○ Strange smells
● SYMPTOMS:
○ Depend on neurons affected

1. Partial Seizure
● One hemisphere or lobe
a. Simple Partial
● Remains conscious
● Small area of brain
● Strange sensations
● Jerking movements
b. Complex Partial
● Impaired consciousness
● Lose consciousness or impaired awareness and
responsiveness
● May not remember
Jacksonian March
● Starts with one group → spreads to other group
● Usually knows something is happening
● Often remembers

2. Generalized Seizure
● Both hemispheres + loss of consciousness
● Sometimes starts partial
SUMMARY ● “Tonic” - stiff/flexed
● “Clonic” - convulsions
Diagnosis ● “Atonic” - relaxed
● Brain imaging - look for abnormalities (e.g., tumors) ● “Tonic-clonic”
○ MRI ○ tonic phase (stiff) → clonic phase (convulsion)
○ CT scan ● “Myoclonic” - short muscle twitches
● ETG (electroencephalogram) - detects electrical signals in ● “Absence” - lose and regain consciousness (spaced out)
brain STATUS EPILEPTICUS
Because epilepsy varies, diagnosis requires test and ● If seizure last ≥ 6 mins
examination of clinical history ○ Ongoing or without returning to normal
○ Usually tonic-clonic
Treatment ● “Medical emergency”
● Daily medication (Anticonvulsants) ○ Often treated with Benzodiazepines (enhance GABA)
○ Wide variety
SYMPTOMS FOLLOWING SEIZURES
○ Depends on px needs
● Epilepsy surgery ● Postictal confusion
○ Removes the cause of seizures ○ Post (after) ictal (seizure)
● Nerve stimulation ● Todd’s Paralysis (Paresis)
○ Stimulate vagus nerve ○ In arms or legs
○ Lasts average - 15 hrs
EPILEPSY ○ Subsides completely - 2 days
○ Temporary and severe suppression of seizure affected
area

● Outward signs:
○ Jerking

109
PCOL 211 - Pharmacology 1

DRUGS FOR PARKINSON

PARKINSONISM
● A slowly progressive degenerative neurological disorder,
characterized tremors, muscular rigidity, bradykinesia, and
postural and gait abnormalities
● AKA Parkinson’s Disease, Idiopathic Parkinsonism,
Primary Parkinsonism, Paralysis Agitans
● Associated with the depigmentation of substantia nigra
and the loss of dopaminergic input to the basal ganglia
(Extrapyramidal System)
● A progressive neurodegenerative disease characterized by
a combination of:
1. Rigidity
2. Bradykinesia
3. Tremor
TYPES OF ABNORMAL BODY MOVEMENT
4. Postural instability
5. Cognitive decline occurs in many patients as the 1. TREMOR
disease advance. ● a rhythmic oscillatory movement around a joint and is
● Affective disorders, personality changes, abnormalities of best characterized by its relation to activity
autonomic function, sleep disorders, and sensory ● Tremor at rest is characteristic of parkinsonism
complaints or pain may be present. 2. CHOREA
● consists of irregular, unpredictable, involuntary muscle
DIAGNOSIS jerks that
1. Diagnosis depends on clinical findings. ● occur in different parts of
2. Tests (including imaging) are most often used to rule out ● the body and impair
an etiology of secondary Parkinson’s disease. ● voluntary activity
3. New technologies [e.g., positron emission tomography 3. TICS
(PET) scan] ● sudden coordinated abnormal movements that tend to
4. A specific form of single-photon-emission computed occur repetitively, particularly about the face and head
tomography (SPECT) ● EX. repetitive sniffing or shoulder shrugging

ETIOLOGY

1. Primary (Idiopathic) Parkinson’s Disease


● AKA Classic Parkinson’s Disease of Paralysis Agitans
● The cause is unknown while treatment may be palliative
an thye disease is incurable
● Most patients suffer from this type of parkinsonism
● Hypothesis of neuronal loss might be cause by: toxins and
exposure to free radicals
● Primary pathology is dopamine deficiency
● Good response to dopaminergic drugs
● Later in disease: falling & dementia

2. Secondary Parkinson’s Disease


● From a known cause
● May be caused by drugs, including DA antagonist such as
Phenothiazines, Butyrophenones and Reserpine
● Caused by poisoning from chemicals or toxins such as
CO, heavy metals and MPTP
● May be cause by atherosclerosis, degenerative diseases
of the CNS and metabolic dse
● Type of parkinsonism that is caused by a disease or
medication.
● Examples:
○ Hypothyroidism
○ Drugs
○ Poisoning
○ Infections
○ Metabolic disorders

110
PCOL 211 - Pharmacology 1

○ Part III: is a clinician-scored motor evaluation


PARKINSON’S DISEASE PARKINSONISM
○ Part IV: the Hoehn and Yahr staging of severity of
Parkinson’s disease.
Also referred to as idiopathic Also referred to as atypical
Parkinson’s disease (IPD) parkinsonism or Parkinson’s ○ Part V: the Schwan and England ADL scale.
plus syndrome
SIGNS AND SYMPTOMS OF PD
SYMPTOMS SYMPTOMS ● Tremor
Main motor symptoms: Main motor symptoms: ○ Because of inc Ach, most evident at rest and with
● Tremors ● Tremors low-frequency movement.
● Limb rigidity ● Limb rigidity
○ Aka PILL-ROLLING TREMOR
● Bradykinesia ● Bradykinesia
● Limb Rigidity
Many other motor symptoms Other symptoms occur, not ○ Detected when arm responds with rachet-like
may occur generally present in PD, and movement when the limb moved passively
vary based on the specific ● Akinesia or Bradykinesia
Non-motor symptoms: form of the Parkinson’s plus ○ Difficulty in initiating movements; (B) Slowness in
● Cognitive changes syndrome
performing common voluntary movements
● Depression, anxiety,
apathy, irritability The 6 most common forms ● Gait and Postural Difficulties
● Hallucinations and are: ○ Patient walk with a stopped, flexed posture
delusions ● Multiple system atrophy ● Changes in Mental Status
● Sleeping problems (MSA) ○ Depression (50%), Dementia (25%) and Psychosis
● Swallowing, chewins, ● Progressive
speaking difficulty supranuclear palsy T- Tremor
● Urinary or constipation (PSP) R- Rigidity
problems ● Corticobasal syndrome A- Akinesia
(CBS) P- Postural Difficulties / Postural instability and abnormalities
● Dementia with Lewy
bodies (DLB)
● Drug-induced STAGES OF PARKINSON’S DISEASE
parkinsonism
● Stage 0 – no clinical signs evident
● Vascular parkinsonism
(VP) ● Stage 1 – Unilateral movements
● Stage 2 – Bilateral involvement but no postural
abnormalities
DRUG-INDUCED PARKINSONISM
● Stage 3 – Mild to moderate bilateral dse
● RESERPINE and other related TETRABENAZINE ● Stage 4 – Bilateral involvement with postural instability
○ deplete biogenic monoamines from their storage sites ● Stage 5 – Severe Diseases
○ Haloperidol and the Phenothiazines block DA
receptors DRUGS USED IN PARKINSONISM
○ These drugs may therefore produce a parkinsonian
○ syndrome, usually 3 within 3 months after introduction

Drug for Other Movement Disorders

Tremor Huntington’s & Wilson's disease


UPDRS (Propranolol) Tourette's (penicillamine )
(haloperidol,
● To evaluate the clinical efficacy of antiparkinson drugs and tetrabenazine )
to monitor disease progression.
○ Part I : evaluation of mentation, behavior & mood
○ Part II: a self-reported evaluation of the activities of
daily living (ADL)

111
PCOL 211 - Pharmacology 1

● Behavioral effects
MOA OF DRUGS USED IN PARKINSONISM
○ depression, anxiety, agitation,
1. Precursor of Dopamine Agonists ○ insomnia, somnolence,
● Levodopa ○ confusion, delusions, hallucinations, nightmares,
● Carbidopa euphoria
2. Direct Acting Dopamine Agonist ○ Choreoathetosis
● Ergot Derivative – Bromocriptine, Pergolide
Drug Holidays
● Non-Ergot Derivative – Pramipexole, Roniperole
3. Catechol – O – Methyltransferase (COMT) inhibitors ● Discontinuance of the drug for 3–21 days
● Tolcapone ● Temporarily improve responsiveness to levodopa and
● Entacapone alleviate some of its adverse effects
4. Indirect-Acting Dopamine Agonists
● Dopamine Releaser – Amantadine Drug Interactions
● Monoamine Oxidase Inhibitor – Selegiline ● Vitamin B6 increases the peripheral breakdown of
5. Anticholinergics/Antimuscarinics levodopa and diminishes its effectiveness
● Benztropine, Biperiden, Trihexyphenidyl ● MAO-A inhibitors can lead to hypertensive crisis
● Orphenadrine, Procyclidine
CARBIDOPA LEVODOPA (SINEMET)

● MOA: L-dopa is converted to DA by dopa decarboxylase


thereby increasing the CNS levels of DA in the brain
Carbidopa in metabolism of L-dopa thereby inc level of DA
in the CNS

Carbidopa Levodopa (Sinemet) Pharmacokinetics

● Absorption: 40% to 70% of dose is absorbed after oral


administration.
DOPAMINE PRECURSOR ● Distribution: Carbidopa is distributed widely in body
tissues except the CNS. Levodopa is also distributed into
LEVODOPA breast milk.
● 3-(3,4-dihydroxyphenyl)-L-alanine ● Metabolism: Carbidopa isn’t metabolized extensively. It
● Levorotatory stereoisomer of dopa inhibits metabolism of levodopa in the GI tract, thus
● Drug of choice for Parkinsonism increasing its absorption from the GI tract and its level in
● DOPA decarboxylase inhibitor plasma.
● CARBIDOPA ● Excretion: 30% of the dose is excreted unchanged in
● Effects: urine within 24 hours. When given with carbidopa, the
○ peripheral metabolism of levodopa is reduced amount of levodopa excreted unchanged in urine
○ plasma levels of levodopa are higher increases by about 6%. Half-life is 1 to 2 hours.
○ plasma half-life is longer Carbidopa Levodopa (Sinemet) ADR
○ more dopa is available for entry into the brain
○ Reduce the daily requirements of levodopa by ● GI: Anorexia, N/V, abdominal pain
approximately 75% ● CV: Postural hypoTN and Tachycardia
● Muskuloskeletal: Dyskinesia/Choreiform movement
Clinical Use ● CNS: Depression, Fluctuations in response
1. Best results of levodopa treatment are obtained in the first ● Hematologicals: Anemia
few years of treatment ● Misc: Mydriasis, glaucoma, hot flushes, Gout and
2. Benefits of levodopa treatment often begin to diminish after abnormal smell, Agranulocytosis, Wearing-off reaction
about 3 or 4 years of therapy Carbidopa Levodopa (Sinemet) Drug Interaction
Adverse E ect ● Amantadine, benztropine, procyclidine, trihexyphenidyl-
● GI effects May increase efficacy of levodopa
○ Anorexia ● Pyridoxine – inc peripheral breakdown of L-dopa
○ Nausea and vomiting ● L-dopa + Phenelzine = HTN crisis

112
PCOL 211 - Pharmacology 1

● Antipsychotic drugs = block DA receptor (Parkinsonian


Pramipexole (Mirapex) ADR
receptor)
● Symptomatic hypoTN, Constipation
ERGOT DERIVATIVES
ROPINIROLE (REQUIP)
BROMOCRIPTINE (PARLODEL)
● MOA: D2 agonist
● MOA: Partial D2 Agonist, Adjunct to L-dopa ● Monotherapy in px with mild dse and as a means of
● Used for hyperprolactinemia smoothing the response to L-dopa

Bromocriptine (Parlodel) Pharmacokinetics Ropinirole (Requip) Pharmacokinetics

● Absorption: delayed by food and peak plasma levels are ● Absorption: Rapidly absorbed from the GI tract after oral
achieved at ~120 min in the fed state. admin. Bioavailability: about 50%.
● Metabolism: extensive hepatic firstpass extraction and ● Distribution: Widely distributed. Plasma protein binding:
metabolism, and only 5–10% of the ingested dose reaches 10-40%.
the systemic circulation. ● Metabolism: Extensively metabolised in the liver by
● Excretion: 90% is excreted via the biliary route with an CYP1A2.
elimination half life of ~6 h ● Excretion: Excreted in the urine as inactive metabolites;
<10% of the oral dose is excreted unchanged. Elimination
Bromocriptine (Parlodel) ADR half-life: about 6 hr.
● first dose phenomenon Ropinirole (Requip) ADR
● Dizziness
● Syncope
● Drowsiness
● Bradycardia
● Fainting
● Dizziness
● cardiac arrhythmias
● Somnolence
● postural hypoTN
● symptomatic hypoTN
● tachycardia
● Hallucinations
PERGOLIDE (PERMAX) ● HA
● Fatigue
● MOA: Stimulates D1, D2 receptors
● More effective than bromocriptine (1000x) APOMORPHINE (APOKYN)
● Inh secretion of prolactin, inc serum concn of GH, dec the
● MOA: DA agonist
serum concn of LH
● Temporary relief of “off-periods” of akinesia
● Used as adjunct tx to L-dopa and C-dopa
Apomorphine (Apokyn) Pharmacokinetics
Pergolide (Permax) Pharmacokinetics
● Absorption: Well absorbed.
● Absorption: peak plasma concentrations within 2-3 h
● Distribution: Volume of distribution: 218 L. Plasma protein
● Distribution: 90% protein bound yet has negligible drug
binding: Approx 90% (mainly albumin).
interactions.
● Metabolism: Undergoes conjugation w/ glucuronic acid or
● Elimination: completely removed after 4-5 days. Half-life of
sulfate to its major metabolite apomorphine sulfate and
about 21 h
demethylation to form norapomorphine.
Pergolide (Permax) ADR ● Excretion: Via urine (93%) as metabolites and faeces
(16%). Terminal elimination halflife: 40 min.
● Hypersensitivity
Apomorphine (Apokyn) ADR
NON-ERGOT DERIVATIVES
● SEVERE Nausea (tx: antimetics)
PRAMIPEXOLE (MIRAPEX) ● Dyskinesia
● hypoTN
● MOA: D3 receptor agonist
● Drowsiness
● Used as monotherapy for mild parkinsonism
● sweating
● Helpful in px with adv dse permitting dec dose of L-dopa
● Has ability to scavenge H2O2 and inc neuropathic act in DOPAMINE AGONIST
mesencephalic DA cell
BROMOCRIPTINE D2 agonist
Pramipexole (Mirapex) Pharmacokinetics
● Absorption: peak concentrations in approximately 2 hours. PERGOLIDE Stimulates both D1 and D2 receptors
BA 90%
● Distribution: volume of distribution of about 500 L, 15% PRAMIPEXOLE Preferential affinity for the D3 receptors
bound to plasma proteins
ROPINIROLE Pure D2 receptor agonist
● Elimination: 8 hours in healthy volunteers and 12 hours in
elderly volunteers. 90% of dose recovered in urine, almost ROTIGOTINE Delivered daily through a skin patch.
all as unchanged drug

113
PCOL 211 - Pharmacology 1

CATECHOL-O-METHYLTRANSFERASE INHIBITOR Rasagiline (Azilect) Drug Interaction


● Meperidine, tramadol, methadone, propoxyphene,
TOLCAPONE (TASMAR)
cyclobenzaprine, or St. John’s wort, dextromethorphan
● MOA: Selective, reversible inh ● Over-the-counter cold preparations
● Used as adjunct to L-dopa/C-dopa therapy ● Tricyclic antidepressants or serotonin reuptake inhibitors
● Inh COMT peripherally and centrally
INDIRECT-ACTING DOPAMINE AGONIST (MAO
● Prolong axn of L-dopa
INHIBITORS)
● Improve response and prolonging “on-time”
● Results in doubling of the elimination t1/2 of L-dopa and
SELEGILINE (SELEGOS)
inc oral BA of L-dopa (40-50%)
● MOA: Selective irreversible inhibitor of MAO-B at normal
Tolcapone (Tasmar) Pharmacokinetics doses
● Absorption: Rapidly absorbed (absolute bioavailability is ● Retards breakdown of dopamine
about 65%) ● Enhance and prolong fx of L-dopa
● Distribution: VD=9 L, Protein binding > 99.9% (to serum ● MAO-A inh (high doses)
albumin) ● Used as adjunctive therapy for px with declining response
● Metabolism: glucuronidation to L-dopa/C-dopa
● Excretion: (0.5% of dose) found unchanged in urine. Also ● Reduce "end of dose" or "on -off" fluctuations in response
excreted in the bile. Half-life= 2-3.5 hours ● Enhances and prolongs the antiparkinsonism effect of
levodopa
Tolcapone (Tasmar) ADR ● May cause insomnia when taken later during the day
● Liver Failure, Hyperpyrexia and confusion
Selegiline (Selegos) Pharmacokinetics
ENTACAPONE (COMTAN) ● Absorption: peak plasma concentrations occur in 30
● MOA:Selective and reversible inhibitor of COMT minutes after oral doses. bioavailability is about 10% and
● Inh COMT only peripherally is increased when given with food
● Improves duration of “on-time” and dec “offtime ● Distribution: crosses the blood-brain barrier
● Metabolism and Elimination: 15% appears in the feces,
Entacapone (Comtan) Pharmacokinetics excreted mainly in urine
● Absorption: rapidly absorbed, with a Tmax of ● Metabolites: 1-(-)-desmethylselegiline (norselegiline),
approximately 1 hour. BA- 35%. 1-(-)-N-methylamfetamine, and 1-(-)-amfetamine
● Distribution: VD- 20 L (iv admin) / high plasma protein
Selegiline (Selegos) ADR
binding 98% binds mainly to serum albumin.
● Metabolism and Elimination: 0.2% of dose found ● HTN crisis (if with SSRIs)
unchanged in urine. After oral administration, 10% is ● inc L-dopa S/E
excreted in urine and 90% in feces.
DOPAMINE RELEASERS
Entacapone (Comtan) ADR
AMANTADINE
● rapid withdrawal causes hyperpyrexia and confusion
● MOA: Inc DA lvl at post synaptic receptor sites by dec
INDIRECT-ACTING DOPAMINE AGONIST (MAO reuptake and enhance DA synthesis and release
INHIBITORS)
● Potentiate dopaminergic function by influencing the
synthesis, release, or reuptake
RASAGILINE (AZILECT)
● Also an anti-viral agent
● MOA: Selective irreversible inhibitor of MAO-B ● Have anticholinergic effect
● MAO-A inh (High doses) (NE, 5-HT, DA) ● Dec TRAP
● More potent than selegiline in preventing MPTP-induced ● Given in combi with L-dopa
parkinsonism ● Less efficacious than levodopa
○ MPTP (1-methyl-4-phenyl-1,2,3,6-tetrahydropyridine) - ● Tolerance develops more readily
byproduct of meperidine
Amantadine Pharmacokinetics
● Neuroprotective agent
● Absorption: peak concentration: W/in approx 4 hrs
Rasagiline (Azilect) Pharmacokinetics
● Distribution: Crosses the placenta and the blood-brain
● Absorption: rapidly absorbed, with a Tmax of barrier; enters breast milk. Volume of distribution: 3-8 L/kg.
approximately 1 hour. Plasma protein binding: Approx 67%;
● Distribution: VD- 87 L ● Metabolism: mainly by N-acetylation
● Metabolism and Elimination: 62% of total dose in urine ● Excretion: Via urine (80-90% as unchanged drug and
and 7% of total dose in feces over 7 days small amounts of an acetylated metabolite). Plasma
elimination half-life: Approx 15 hr
Rasagiline (Azilect) ADR
● Flu syndrome, Arthralgia, Depression, Dyspepsia

114
PCOL 211 - Pharmacology 1

C. INTENTION TREMOR
Amantadine ADR
● Present during movement
● Restlessness, depression, irritability, insomnia, agitation, D. REST TREMOR
excitement, hallucinations, and confusion ● Due to parkinsonism
● Overdosage: acute toxic psychosis
HUNTINGTON’S DISEASE
● Livedo Reticularis (netlike rashes)
● Inherited disorder characterized by progressive chorea and
Amantadine Contraindication
dementia that begins at adulthood
● hx of seizures and HF ● Related to imbalance of DA, Ach, GABA, and perhaps
other neurotransmitter in the basal ganglia
ANTICHOLINERGICS
● MOA: Blocks the excitatory neurotransmitter cholinergic BALLISMUS
influence in the basal ganglia ● Violent movements of the limbs, as in chorea, sometimes
● More effective for tremor/rigidity affecting only one side of the body
● Used for mild symptoms, tremors
TOURETTE’S SYNDROME
● Less effective for bradykinesia and less effective for
postural imbalance ● Unknown cause that frequently responds to haloperidol
● Centrally acting antimuscarinic and other dopamine D2 receptor blockers, pimozide.
● They improve the tremor and rigidity of parkinsonism but
ATHETOSIS
have little effect on bradykinesia
● DRUGS: ● A continuous stream of slow, sinuous writhing movements,
1. Benztropine mesylate typically of the hands and feet
2. Biperiden ● Movements typical to athetosis is called athetoid
3. Trihexyphenidyl movements
● It is said to caused damage by corpus striatum of the brain
ADR
RESTLESS LEGS SYNDROME
● Peripheral anticholinergic fx (Hypohydrosis, urinary
retention, constipation, dry mouth, tachycardia, inc ● Unknown causes
intraocular tension and nausea) ● Unpleasant creeping discomfort in the limbs that occurs
● Other fx: dizziness, delirium, disorientation, anxiety, particularly when the px is at rest
agitation, orthostatic hypoTN, impaired memory ● More common in pregnant women, uremic and diabetic
patients
OTHER MOVEMENT DISORDER
● Dopaminergic therapy is the preferred treatment and
ropinirole, a long acting drug
TREMOR
● Opioid analgesics and BZD are also used
● Consist of rhythmic oscillatory movements
WILSON’S DISEASE
● May be alleviated by B-blockers including Propranolol but
caution with CHF, asthma, diabetes, hypoglycemia ● Inherited disorder of copper metabolism results in
● Metoprolol has been used with patients with concomitant deposition of copper salts in the liver and other tissues.
pulmonary dse
● Antiepileptic drugs including primidone and Topiramate
have been used to treat essential tremor ALZHEIMER’S DISEASE

ALZHEIMER'S DISEASE
● Is a neurodegenerative disease that causes memory loss,
behavioural changes and immobility
● The risk for Alzheimer’s disease increases with advancing
age
● Nearly 5% of the adult population aged 65 years will have
Alzheimer’s disease. The percentage will double by age 70
years, and by age 85, nearly half of all men and women
will have the disease.
● Dementia is not a normal part of the aging process
○ Dementia is caused by oxidative stress by lewy bodies
TYPES OF TREMOR (damage in the nuerons) it prevents from having new
memories
A. PHYSIOLOGIC POSTURAL TREMOR
● Normal phenomenon, enhanced amplitude by anxiety,
fatigue, thyrotoxicosis, and IV epi
B. ESSENTIAL TREMOR
● A postural tremor, sometimes familial

115
PCOL 211 - Pharmacology 1

Brain scans done with Positron Emission Tomography (PET)


show how Alzheimer’s affects brain activity. The left image
shows a normal brain, while the right is from a person with
Alzheimer’s. The blue and black areas in the right image
indicate reduced brain activity resulting from the disease. ● Healty neuron - smooth sending messages between
neuron to neuron
● Diseased neuron
COMORBIDITY
● The following are believed to speed the progression of
Alzheimer’s disease:
○ Cerebrovascular disease
○ High blood pressure
○ Diabetes
○ Sleep apnea
○ Parkinson’s disease - drugs for Parkinson’s can cause
Alzheimer’s. Drugs for Alzheimer’s can cause
Parkinson’s

RISK FACTORS
● Advancing age (commonly seen in 60 and 65 years old
and above
● Family history: Early-onset, familial Alzheimer’s disease
affects adults between 30 years and 60 years.
● Familial Alzheimer’s is associated with the gene that
makes the protein apolopoprotein E (ApoE)
● 15% of persona with defective ApoE4 allele will develop
ROLE OF NEUROTRANSMITTERS
Alzheimer’s disease
● Several neurotransmitters play a significant role in
PATHOPHYSIOLOGY Alzheimer’s disease:
● Alzheimer’s disease damages the hippocampus ○ Acetylcholine (ACh)
● The disease causes cholinergic nerve cells in the brain to ○ Glutamate
die ○ Dopamine
○ Cholinergic system or parasympathetic system ○ Norepinephrine
(Acetylcholine helps in memory development) ○ Serotonin
● The cerebral cortex shrinks in size and the ability to think - A normal leve or a good balance of this excitatory
and function diminishes. neurotransmitter will enhance our memory,
● Abnormal structures, called plaques and tangles, develop locomotion, cognition
in the brain. ● Excess activity of the N-methyl-D-aspartate (NMDA)
○ PLaques and tangles are caused by oxidative stress receptor is linked to Alzheimer’s disease and is believed to
inside our body contribute to the process of neurodegeneration
○ PLaques and tangles can cause death of the neurons - Because too much of the glutamatergic transmission
● Plaques: are sticky, dense substances composed of or the NMDA receptor activity. If there’s too much of
protein called beta amyloid - fill the spaces between the NMDA activity or stimulation of the NMDA it will
neurons in the brain and interfere with the transmission of cause cytotoxicity (too much excitation will cause
signals between neurons. toxicity) and will promote the cell death which is an
● Tangles: are twisted fibers and are made of protein clumps underlying and potential mechanism of
called tai - block the neurodegeneration
● Transmission of messages between neurons.

116
PCOL 211 - Pharmacology 1

NEUROTRANSMITTERS INVOLVED IN ALZHEIMER’S Level 3: Mild cognitive decline


DISEASE (Mild Cognitive Impairment)
● Neurodegeneration reduces: ● Earliest clear-cuts deficits. Manifestations in more than one
○ Acetylcholine: levels (cholinergic pathways are of the following areas:
involved in memory and cognition) (a) Patient may have gotten lost when travelling to an
○ Glutamate: binding (involved in memory formation unfamiliar location;
and learning) N-Methyl-D aspartate (NMDA) receptor (b) Co-workers become aware of patient’s relatively poor
binding (associated with memory and learning) performance
(c) Word and name finding deficit becomes evident to
SIGNS AND SYMPTOMS intimates;
1. Cognitive: forgetfulness, confusion, difficulty reading, (d) Patient may read a passage or a book and retain
speaking, writing relatively little material
2. Physical: difficulty with body movements (e) Patient may demonstrate decreased facility in
3. Psychological: anxiousness, anger, aggressiveness remembering names upon introduction to new people
(f) Patient may have lost or misplaced an object of value
Alzheimer’s Top 10 Early Signs (g) Concentration deficit may be evident on clinical testing
● Memory loss ● Objective evidence of memory deficit obtained only with an
● Changes in mood intensive interview. Decreased performance in demanding
● Misplacing belongings employment and social settings. Denial begins to become
● Hard to complete familiar task manifest in patient. Mild to moderate anxiety accompanies
● Confusion of time and place symptoms.
● Changes in vision
● Struggling to communicate Level 4: Moderate cognitive decline
● Poor judgement (Mild Dementia)
● Social withdrawal ● Clear-cut deficit on careful clinical interview. Deficit
manifest in following areas:
The Global Deterioration Scale for Assessment of (a) Decreased knowledge of current and recent events;
Primary Degenerative Dementia (b) May exhibit some deficit in memory of one's personal
history
● The Global Deterioration Scale (GDS), developed by Dr. (c) Concentration deficit elicited on serial subtractions
Barry Reisberg, provides caregivers an overview of the (d) Decreased ability to travel, handle finances, etc.
stages of cognitive function for those suffering from a ● Frequently no deficit in following areas:
primary degenerative dementia such as Alzheimer’s (a) Orientation to time and place
disease. It is broken down into 7 different stages. Stages (b) Recognition of familiar persons and faces;
1-3 are the pre-dementia stages. Stages 4-7 are the (c) Ability to travel to familiar locations
dementia stages. Beginning in Stage 5, an individual can ● Inability to perform complex tasks. Denial is the dominant
no longer survive without assistance. Within the GDS, defense mechanism. Flattening of affect and withdrawal
each stage is numbered (1-7), given a short title (i.e., from challenging situations frequently occur.
Forgetfulness, Early Confusional, etc. followed by a brief
listing of the characteristics for that stage. Caregivers can Level 5: Moderately severe cognitive decline
get rough idea of where an individual is at in the disease (Moderate Dementia)
process by observing that individual’s behavioral ● Patient can no longer survive without some assistance.
characteristics and comparing them to the GDS. For more Patient is unable during interview to recall a major relevant
specific assessments, use the accompanying Brief aspect of their current lives, e.g., an address or telephone
Cognitive Rating Scale (BCRS) and the Functional number of many years, the names of close family
Assessment Staging (FAST) measures. members (such as grandchildren), the name of the high
Level 1: No cognitive decline school or college from which they graduated. Frequently
some disorientation to time (date, day of week, season,
● No subjective complaints of memory deficit. No memory etc.) or to place. An educated person may have difficulty
deficit evident on clinical interview. counting bsck from 40 by 4s or from 20 by 2s. Persons at
this stage retain knowledge of many major facts regarding
Level 2: Very mild cognitive decline
themselves and others. They invariably know their own
(Age Associated Memory Impairment)
names and generally know their spouses’ and children’s
● Subjective complaints of memory deficit, most frequently in names. They require no assistance with toileting and
following areas: eating, but may have some difficulty choosing the proper
(a) Forgetting where one has placed familiar objects; clothing to wear.
(b) Forgetting names one formerly knew well
Level 6: Severe cognitive decline
● No objective evidence of memory deficit on clinical
(Moderately Severe Dementia)
interview. No objective deficits in employment or social
situations. Appropriate concern with respect to ● May occasionally forget the name of the spouse upon
symptomatology. whom they are entirely dependent for survival. Will be
largely unaware of all recent events and experiences in
their lives. Retain some knowledge of their past lives but

117
PCOL 211 - Pharmacology 1

this is very sketchy. Generally unaware of their ● AChE-S appears to enhance the degeneration of
surroundings, the year, the season, etc. May have difficulty cholinergic neurons
counting from 10, both backward and, sometimes, forward. ● AChE-R has been shown to be neuroprotective in human
and animal studies. (Neuroprotective agents protect nerve
Will require some assistance with activities of daily living,
cells from damage.)
e.g., may become incontinent, will require travel assistance ● AChE inhibitors increase levels of ACh.
but occasionally will be able to travel to familiar locations. ● AChE increases the number and the activity of nicotinic
Diurnal rhythm frequently disturbed. Almost always recall receptors
their own name. Frequently continue to be able to
DONEPEZIL (ARICEPT)
distinguish familiar from unfamiliar persons in their
environment. Personality and emotional changes occur. ● MOA: is a specific and reversible inhibitor of
These are quite variable and include: acetylcholinesterase (AChE), the predominant
(a) Delusional behavior, e.g., patients may accuse their cholinesterase in the brain. It exerts its therapeutic effect
spouse of being an impostor, may talk to imaginary by enhancing cholinergic function in the central nervous
figures in the environment, or to their own reflection in system by increasing the concentration of acetylcholine
the mirror; through reversible inhibition of its hydrolysis by AChE.
(b) Obsessive symptoms e.g., person may continually
Pharmacokinetics
repeat simple cleaning activities;
(c) Anxiety symptoms, agitation, and even previously ● Absorption: well absorbed orally, BA - 100%, peak
nonexistent violent behavior may occur; plasma concentrations in 3-4 hours
(d) Cognitive abulla, i.e., loss of willpower because an ● Distribution: VD is 12 L/kg. It is approximately 96% bound
individual cannot carry a thought long enough to to human plasma proteins, mainly to albumins (about 75%)
determine a purposeful course of action and alpha 1-acid glycoprotein (about 21%) over the
concentration range of 2-1,000 mg/mL
Level 7: Very severe cognitive decline ● Metabolism: glucuronidation
(Severe Dementia) ● Excretion: excreted in the urine intact
● All verbal abilities are lost over the course of this stage.
Frequently there is no speech at all -only unintelligible Contraindications
utterances and rare emergence of seemingly forgotten ● CVD - may cause heart block
words and phrases. Incontinent of urine, requires ● Asthma - bronchoconstriction
assistance toileting and feeding. Basic psychomotor skills,
e.g., ability to walk, are lost with the progression of this ADR
stage. The brain appears to no longer be able to tell the
body what to do. Generalized rigidity and development ● Anemia, abnormal dreams, abnormal crying, aggression,
neurolofic reflexes are frequently present. agitation, anxiety, apathy, aphasia, liver failure
TREATMENT FOR ALZHEIMER’S DISEASE GALANTAMINE (RAZADYNE)
● Drug used to treat Alzheimer’s disease either: ● MOA: is a reversible inhibitor of acetylcholine esterase and
1. Increases ach levels at the synapse enhances the intrinsic action of acetylcholine on nicotinic
2. Block glutamate activity receptors, leading to increased cholinergic
neurotransmission in the CNS.
Drug Name Drug Type Stage of
Alzheimer’s
DIsease

Donepezil Anti-cholinesterase All stages

Galantamine Anti-cholinesterase Mild to


moderate

Rivastigmine Anti-cholinesterase Mild to


moderate

Memantine N-methyl-D-aspartate Moderate to


(NMDA)-receptor severe
antagonist

Combination of Moderate to
Donepezil and severe
Pharmacokinetics
Memantine
● Absorption: Well absorbed from the gastrointestinal tract.
ACETYLCHOLINESTERASE INHIBITORS Food delays rate of absorption.
○ Bioavailability: Approx 90%
● Typically prescribed for mild to moderate disease
○ Time to peak plasma concentration: Approx 1 hr
● For the treatment of vascular dementia (dementia
associated with cerebrovascular disease) ● Distribution: Volume of distribution: 175 L.
● Acetylcholinesterase (AChE) is an enzyme degrades ACh. ○ Plasma protein binding: 18%
● Several types of AChE ● Metabolism: Metabolised in the liver mainly by CYP2D6
isoenzyme and CYP3A4 isoenzyme to

118
PCOL 211 - Pharmacology 1

O-desmethyl-galantamine, and galantamine-N-oxide ● Minimally subject to drug interactions involving CYP450


metabolites, respectively. isozymes; however, drug clearance decreased by alkaline
● Excretion: Via urine (approx 20-3-% as unchanged drug); urine by as much as 80%
faeces (approx 6%.
ADR
○ Elimination half-life: Approx 7 hours.
● Abdominal pain and cramping
Contraindications
● Dizziness
● Several renal and hepatic impairment. ● Diarrhea
● Nausea (except memantine)
ADR
● Sedation (except donepezil)
● Serious skin reaction ● Hypotension (except donepezil)
● CNS depression ● Increased urination (donepezil only)
● Vagotonic effects on heart rate ● Pseudoparkinsonism (rivastigmine and memantine)
● Weight loss ● Myocardial infarction, erectile dysfunction, and suicide
● Seizures ideation (memantine)
● Increase gastric acid secretion
MEMANTINE (EBIXA/NAMENDA)
● Bladder outflow obstruction
● BPH symptoms exacerbation ● MOA: Is a non-competitive N-methyl-D-aspartate
(NMDA)-receptor antagonist which binds preferentially to
RIVASTIGMINE (EXELON)
NMDA-receptor-operated cation channels. It blocks the
● MOA: is a cholinesterase inhibitor, inhibits both action of glutamate, the principal excitatory
butyrylcholinesterase and acetylcholinesterase (unlike neurotransmitter in the CNS.
donepezil, which selectively inhibits acetylcholinesterase).
It is thought to work by inhibiting these cholinesterase Pharmacokinetics
enzymes, which would otherwise break down the brain ● Absorption: Well absorbed.
neurotransmitter acetylcholine. ○ Absolute bioavailability: Approx 100%
○ Time to peak plasma concentration: Approx 3-8 hr
Pharmacokinetics
● Distribution: Volume of distribution: 9-11 L/kg
● Absorption: readily and completely absorbed from the ○ Plasma protein binding: Approx 45%
gastrointestinal tract. Food delays absorption. ● Metabolism: Undergoes partial hepatic metabolism to
○ Bioavailability: 36% main metabolites including N-3,5-dimethyl-gludantan and
○ Time to peak plasma concentration: Approx 1 hour 1-nitroso-3,5-dimethyl-adamantane
(oral); 8 to 16 hours (transdermal) ● Excretion: Mainly via urine (approx 57-82% as unchanged
● Distribution: widely distributed in the body. Readily drug).
crosses the blood-brain barrier ○ Terminal half-life: 60-100 hr
○ Volume of distribution: 1.8 to 2.7 L/kg.
○ Plasma protein binding: Approx 40% Contraindication
● Metabolism: rapidly and extensively metabolised in the ● Patients with CV disease, epilepsy, history of convulsions
brain via cholinesterase-mediated hydrolysis to weakly or those with predisposing factors for epilepsy.
active decarbamylated metabolite; further metabolised in ● Moderate to severe renal and severe hepatic impairment.
the liver via N-demethylation and/or sulphate conjugation. ● Pregnancy and lactation
● Excretion: mainly via urine (97% as metabolites); faeces
(0.4%) ADR
○ Half-life elimination: 1.5 hours (oral); approx 3 hours ● Headache, dizziness
(transdermal). ● Constipation
● HTN
ADR
● Somnolence
● Allergic dermatitis (patch) ● Anxiety
● CNS depression ● Confusion, hallucinations
● Extrapyramidal symptoms ● Fatigue
● Nausea, vomiting, diarrhoea ● Fungal infections
● Anorexia ● Cystitis
● Weight loss, decreased appetite ● Thromboembolism
● Bradycardia, heart block ● Increased libido
● Psychotic reactions
N-METHYL-D-ASPARTATE RECEPTOR INHIBITOR
● Prescribed for moderate to severe disease MEMANTINE + DONEPEZIL (NAMZARIC)
● Memantine (Ebixa/Namenda) - Blocks the actions of ● Is used to treat dementia (memory loss and mental
glutamate at NMDA receptor sites. changes) associated with moderate or severe Alzheimer’s
● Theorized to protect the neurons from further damage disease.
● 100% bioavailability of all dosage forms ● Hydrochloride extended-release and 10 mg donepezil
○ Hydrochloride (14 mg/10 mg) or 28 mg memantine

119
PCOL 211 - Pharmacology 1

○ Hydrochloride extended-release and 10 mg


kdonepezil REWARD AND REWARD SYSTEM
○ Hydrochloride (28 mg/10 mg) 1. A reward
● an appetitive stimulus given to a human or some other
ADR
animal to alter its behavior. It's typically serve as
● Diarrhea reinforcers, which causes the probability of that
● Anorexia behavior's occurrence to increase when presented
● Vomiting after a behavior.
● Nausea 2. The reward system
● Ecchymosis ● a collection of brain structures that attempts to
regulate and control behavior by inducing pleasurable
ALTERNATIVE TREATMENT FOR ALZHEIMER’S
effects
DISEASE
● Coconut oil - Caprylic acid is a fatty acid found in ANATOMY OF THE REWARD SYSTEM
processed coconut oil. The human body breaks down ● Two pathways: mesolimbic (major) and mesocortical
caprylic acid into the protein ketone. A similar protein is
pathway
used in a drug called Ketasyn.
● Omega-3 fatty acids - regular consumption of omega-3 Mesolimbic pathway
fatty acids reduced cognitive impairment. But, it’s important
to note that this research was conducted in animals, not ● Many drugs of abuse activate the reward circuitry of the
humans. MFB, which requires a stimulation to activate the
● Acupuncture - is an alternative medicine that is believed mesocorticolimbic dopamine system.
to promote self-healing by using fine, sterile needles. This
therapy is thought to stimulate the body and improve the Ventral tegmental area (VTA) medial forebrain
flow of energy.
● → bundle (MFB) nucleus accumbens (NAc)
● Aromatherapy - enhances mood and memory
● Herbal medicine - gingko biloba may benefit people with
cognitive impairment caused by AD.

DRUGS OF ABUSE

PATHOPHYSIOLOGY
● The primary factor in the development of addiction is
neurophysiologic reinforcement (reward). One specific
mesolimbic "reward pathway" has been identified in the
brain, and others may exist. This pathway involves
dopaminergic neurons that originate in the ventral
tegmental area (VTA) and project into the forebrain,
particularly the nucleus accumbens.

Nucleus accumbens (NAc/NAcc)


● Nucleus accumbens, also the ventral striatum--a part of
basal ganglia.
● It's the major target of the electric stimulation producing
pleasure
● NAc reacts with reward
● NAc is involving in drug abuse substance dependence

HYPOTHESIS OF ADDICTION

1. DOPAMINE HYPOTHESIS
● Dopamine is crucial for the rewarding effects of the
psychomotor stimulants and is important but perhaps not
crucial for the rewarding effects of the opiates (morphine,
heroin) nicotine, cannabis and ethanol.

2. NUCLEUS ACCUMBENS HYPOTHESIS


● Lesions of the nucleus accumbens attenuated the
rewarding effects of cocaine and amphetamine.

120
PCOL 211 - Pharmacology 1

● Morphine, amphetamine and cocaine are self administered ● You become dependent on drugs and unable to
directly into the nucleus accumbens live without them.Your physical and mental health
deteriorates.
DEFINITION OF TERMS
Drug Abuse and Addiction
DEPENDENCE
● Marijuana
● occurs when you need one or more drugs to function. It ● P.C.P
can be a bodily response to a substance. This often occurs ● Methamphetamine
if you rely on medications to control a chronic medical ● L.S.D
condition. ● Steroids
○ Psychological Dependence - arise from the mind ● Cocaine
and emotions ● Ketamine
○ Physiological dependence – in accord with ● Painkillers
characteristic of the normal functioning of a living ● Heroin
organism ● INhalants
● Ecstasy
ADDICTION
● Alcohol
● can occur without being dependent on drugs.
SCHEDULE OF CONTROLLED SUBSTANCES
● Addiction may involve:
○ using drugs despite the consequences
Schedule I Schedule II
○ being unable to stop using drugs
○ neglecting social and work obligations because of
High abuse potential High abuse potential
drug use

TOLERANCE Has NO currently accepted Have accepted medical use


medical use
● decreased responsiveness to a stimulus
○ MetabolicTolerance – resulting from metabolism Lack of accepted safety for Abuse of the drug will lead
○ Behavioral tolerance – rxns of persons in response use to dependence
to external stimuli
○ Functional Tolerance – affecting physiological No prescriptions given Prescriptions can’t be refilled
functions
● GHB ● Morphine
WITHDRAWAL ● Marijuana ● Cocaine
● MDMA ● PCP
● is the combination of physical and mental effects that a ● Mscaline ● Oxycodone
person experiences after they stop using or reduce their ● Heroin
intake of a substance such as alcohol and prescription or
recreational drugs.

ABUSE Schedule III Schedule IV


● to use wrongly or improperly
Have potential for abuse but Low potential for abuse
MISUSE less than I & II relative to the drugs
● improper use
Has accepted use Has currently accepted
FACTORS THAT TRIGGERS DRUG ABUSE medical use

● Having a family history of addiction May lead to moderate or low Abuse has limited physical
● Living in an environment where illegal drugs are often used physical dependence dependence
and easy to access
● Having a history of anxiety Refilled (5x) NMT 6mos Refilled (5x) NMT 6mos
● Having a history of depression
● Having a history of other mental health conditions ● Hydrocodone ● BZD
● Buprenorphine ● Zolpidem, Zopictone
Jellinek Curves Stages ● Long acting barbiturates
● Agonist opioid
● These stages include: analgesics
● You use drugs for recreation. You take them
infrequently and in social settings.
● You start using drugs on a regular basis, often Schedule V
abandoning family and friends in favor of drug use.
You become concerned about losing access to drugs. Low abuse potential
● You become addicted to drugs as you become
more tolerant to their effects and preoccupied with Has currently accepted medical use
getting them. You may abandon most or all your
Abuse may lead to limited physical dependence
previous interests and relationships.

121
PCOL 211 - Pharmacology 1

○ Dysphoria
No controlled substance which may be distributed or
dispensed other than a medical purpose ○ increases appetite
○ pain relief
● Cough Suppressants ● EXAMPLES: Marijuana, Hashish
● Prep containing opium ● FDA APPROVED: Dronabinol, Nabilone- for
● Pregabalin chemotherapy
● Centrally-acting anti-diarrheals
3. GAMMA-HYDROXYBUTYRIC ACID

BASIC PHARMACOLOGY OF DRUG ABUSE ● Gama-hydroxybutyric acid (GHB) targets GABA B


receptors, which are located on both cell types. However,
1. Drugs that activate g-coupled receptors GABA neurons are more sensitive to GHB than are DA
2. Drugs that mediate their effects via ionotropic receptors neurons, leading to disinhibition at concentrations typically
3. Drugs that bind to transporters of biogenic amines obtained with recreational use.
● first synthesized in 1960 and introduced as a general
anesthetic
● “club drug”
● “liquid ecstasy,” “grievous bodily harm,” or“date rape drug.”
● is a weak agonist, only GABA neurons are inhibited at the
concentrations typically obtained with recreational use.
● EFFECTS:
○ Euphoria
○ enhanced sensory perceptions
○ feeling of social closeness
○ amnesia

4. LSD, MESCALINE, & PSILOCYBIN


● LSD ergot alkaloid
DRUGS THAT ACTIVATE G1O-COUPLED RECEPTOR
● The main molecular target of LSD and other hallucinogens
is the 5-HT2A receptor. This receptor couples to G proteins
1. OPIOIDS
of the Gq type and generates inositol trisphosphate (IP3 ),
● Disinhibition of dopamine (DA) neurons in the ventral leading to a release of intracellular calcium.
tegmental area (VTA) through drugs that act ● are commonly called hallucinogens*
viaGio-coupled receptors ● EFFECTS:
● Opioids may have been the first drugs to be abused ○ psychotomimetics
(preceding stimulants), and are still among the most ○ Psychosis-like manifestations*
commonly used for nonmedical purposes. ○ dizziness, nausea, paresthesias, and blurred vision
● opioids comprise a large family of endogenous and ○ abortion
exogenous agonists at three G protein-coupled receptors: ○ perceptual effects *
the μ-, κ-, and δopioid recept ○ repetitive exposure still leads to tachyphylaxis
● EXAMPLES: Morphine, Heroin, Codein, Oxycodone,
DRUGS THAT MEDIATE THEIR EFFECTS VIA
Meperidine
IONOTROPIC RECEPTORS
● TREATMENT:
○ NALOXONE NALTREXONE- opioid antagonist
1. NICOTINE (in CIGARETTE SMOKING)
(toxicity)
○ METHADONE, BUPRENORPHINE- long-acting ● One of the most addictive drugs
opioid (tx of addiction) ● is a selective agonist of the nicotinic acetylcholine receptor
○ CLONIDINE – mitigates many of sympathetic signs (nAChR) that is normally activated by acetylcholine
○ LOFEXIDINE – clonidine analog with less hypoTN ● WITHDRAWAL: irritability and sleep problems
○ L-ACETYLMETHADOL – longer-acting methadone ● TREATMENT:
analog 1. NICOTINE that is chewed, inhaled, or transdermally
delivered can be substituted for the nicotine in
2. CANNABINOIDS
cigarettes
● mainly act through presynaptic inhibition 2. CYSTISINE- plant-extract
● a powerful psychoactive substance 3. VARENICLINE- synthetic derivative of cystisine*
● Like opioids, THC causes disinhibition of dopamine ADR: may impair the capacity to drive and has been
neurons, mainly by presynaptic inhibition of GABA neurons associated with suicidal ideation
in the VTA. 4. BUPROPION - is approved for nicotine cessation
● EFFECTS therapy. It is most effective when combined with
○ euphoria and relaxation behavioral therapies.
○ feelings of well-being, grandiosity, and altered
2. BENZODIAZEPINES
perception of passage of time
○ Dose-dependent- visual distortions, drowsiness, ● are positive modulators of the GABAA receptor, increasing
diminished coordination, and memory impairment may

122
PCOL 211 - Pharmacology 1

both single-channel conductance and open-channel ● CV


probability. ○ hypertrophy and fibrosis. Heavy drinking causes
● are commonly prescribed as anxiolytics and sleep reduction in cardiac size and improved function
medications. ● Blood
● EXAMPLE: Flunitrazepam (Rohypnol, Roofies) “date-rape ○ Mild anemia
drug” ● Endocrine
● WITHDRAWAL: irritability, insomnia, phonophobia and ○ hypoglycemia
photophobia, depression, muscle cramps, and even ● GI
seizures ○ Inc gastric acid secretion and pancreatic secretion
● TREATMENT: Chlordiazepoxide, Phenobarbital
Withdrwal
3. BARBITURATES
● Tremor (mainly of the hands), nausea and vomiting,
● sedative-hypnotics excessive sweating, agitation, and anxiety
● visual, tactile, and auditory hallucinations 12–24 hours
4. ALCOHOL
after cessation
● AKA absolute alcohol, drinking alcohol, ethyl alcohol, grain
Treatment
alcohol, rectified spirit, spiritus rectificatus
● Present in wines (10-13%), beers (3-5%) and distilled 1. Benzodiazepines - Lorazepam, Oxazepam,
liquors (35-90%) Chlordiazepoxide
● alters the function of several receptors and cellular 2. Disulfiram (Antabuse) - causes extreme discomfort to px
functions, including GABAA receptors, Kir3/GIRK due to inh of aldehyde dehydrogenase
channels, adenosine reuptake (through the equilibrative ● ADR: N/V, flushing, HA and hypoTN for 30 mins in
nucleoside transporter, ENT1), glycine receptor, NMDA mild cases and several hour in severe ones
receptor, and 5-HT3 receptor. 3. Naltrexone (Revex) - An orally opioid receptor antagonist,
Admin of opioid antagonist inhibited self-admin of ROH*
DEENDENCE: inhibition of ENT1 causes an
4. Acamprosate- An NMDA receptor antagonist is used to
accumulation of adenosine, stimulation of adenosine A2
treat alcoholism
receptors, and ensuing enhanced CREB signaling.
5. KETAMINE & PHENCYCLIDINE (PCP)
Acute Alcohol Consumption E ects ● were developed as general anesthetics
● CNS ● “club drugs”
○ Sedation, relief of anxiety, “Blackout” ● “angel dust,” “Hog,” and “Special K”
○ Intoxication = slurred speech, ataxia, impaired ● noncompetitive antagonism of the NMDA receptor
○ judgement, disibhibited behaviour ● EFFECTS:
○ High Blood Concn = coma, resp depression, death ○ Psychedelic effects last for about 1 hour and also
● Heart include increased blood pressure, impaired memory
○ Myocardial depression (hypoTN), bradycardia function, and visual alterations*
● Smooth Muscle ○ Chronic use may lead to long-lasting psychosis
○ Vasodilator (flushing) closely resembling schizophrenia, which may persist
○ Severe overdose = hypothermia, relaxes uterine beyond drug exposure.

Consequences of Chronic Alcohol Consumption 6. INHALANTS

● Fetal Alcohol Syndrome ● Defined as recreational exposure to chemical vapors, such


○ Chronic maternal alcohol abuse during pregnancy as nitrates, ketones, and aliphatic and aromatic
appears to be a leading cause of mental retardation hydrocarbons
and congenital malformation ● These substances are present in a variety of household
■ Small head, epicanthal folds, flat midface, smooth and industrial products that are inhaled by “sniffing,”
philtrum, low nasal bridge, small eye openings, “huffing,” or “bagging.”
short nose, thin upper lip, underdeveloped jaw ● Nitrous oxide, binds to NMDA receptors and fuel additives
● Immune Syndrome enhance GABAA receptor function
○ Have a higher normal rate of infection and are prone ● EFFECTS:
to respiratory infections such as pneumonia and ○ increased excitability of the VTA has been
tuberculosis documented for toluene and may underlie its addiction
● Increased Risk of Cancer risk.
○ Chronic alcohol use increases the risk of CA of the ○ Amyl nitrite “Poppers” produce smooth muscle
mouth, pharynx, larynx, esophagus and liver Small relaxation and enhance erection, but are not addictive
increase in the risk of breast CA DRUGS THAT BIND TO TRANSPORTERS OF BIOGENIC
● Liver AMINES
○ Most common complication 15-30% drinkers develops
severe liver diseases
1. COCAINE
● CNS
○ tolerance and physical dependence, Peripheral ● is highly addictive (relative risk = 5), and its use is
Neuropathy (numbness) is the most common associated with a number of complications.

123
PCOL 211 - Pharmacology 1

● alkaloid found in the leaves of Erythroxylon coca ● Meperidine (Demerol)


● Cocaine hydrochloride - water-soluble salt that can be ● Propoxyphene (Darvon)
injected or absorbed by any mucosal membrane (eg, nasal ● Anabolic Steroids (Anadrol, Oxandrin) - Roids, juice, gym
snorting). candy, pumpers
● “crack cocaine”- free base, can be smoked when heated in ○ For inc for muscle mass and strength STREET
an alkaline solution - “rush”* NAMES
● inhibits voltage-gated sodium channels, thus blocking ● Methamphetamine - Meth, ice, crank, chalk, crystal, fire,
initiation and conduction of action potentials. glass, go fast, speed
● CNS: cocaine blocks the uptake of dopamine, ● LSD - Acid, blotter, cubes, microdot yellow sunshine, blue
noradrenaline, and serotonin through their respective heaven
transporters. * ● Dextroamphetamine - DXM, Robotripping, Robo, Triple C
● Psilocybin - Magic Mushrooms, purple passion, shrooms,
2. AMPHETAMINES
little smoke
● indirect-acting sympathomimetic drugs that cause the
TYPES OF ALTERNATIVE THERAPIES
release of endogenous biogenic amines, such as
dopamine and noradrenaline • “club drugs”
1. Emotional Freedom Techniques (EFT)
● “hard-core” addiction is far more common with
amphetamines ● involves using the tips of your fingers to tap on a sequence
● EFFECTS: of points on the body that are related to acupressure points
○ on the dopamine system mediate euphoria but may or meridians
also cause abnormal movements and precipitate
psychotic episodes 2. Yoga and Mindfulness-Based Therapies
○ on serotonin transmission may play a role in the ● learn to respond to the stresses in your life rather than
hallucinogenic and anorexigenic functions as well as reacting to them automatically and thus may be able to
in the hyperthermia often caused by amphetamines. make more conscious decisions.
3. ECSTASY (MDMA) 3. Art therapy
● appears to be to foster feelings of intimacy and empathy ● actual process of creating your art can be therapeutic and
without impairing intellectual capacities relaxing
● Causes release of biogenic amines by reversing the action
of their respective transporters. It has a preferential affinity 4. Adventure Therapy (AT)
for the serotonin transporter (SERT) and therefore most ● adventure-based therapy incorporated into treatment
strongly increases the extracellular concentration of programs. If you tend to be a more active person you may
serotonin feel more motivated to engage in this form of treatment as
● TOXIC EFFECTS: hyperthermia, dehydration, Serotonin it will help you self reflect and work through your addiction
Syndrome, Seizures, water intoxication, irreversible in a more dynamic and mobile way.
damage to brain
● WITHDRAWAL: “offset” mood- depression for several 5. Psychodrama
weeks
● participants act out a scene, often related to an
STREET NAMES interpersonal issue.

● Marijuana - blunt, dope, ganja, grass, Mary Jane, Miss


Ema, Miss M, pot, reefer, green, trees, smoke, sinsemilla,
weed, skunk
● Barbiturates - downers, sedatives
● BZD - downers, tranqs
● Methaqualone (Qualudes)
● Flunitrazepam (Rohypnol) - Roofies, Rophies, Mexican
Valium, Rope, Roaches, forget me drug
● Amphetamine - Bennies, black beauties, crosses, hearts,
LA turnaround, speed, truck drivers, upper
● Cocaine - Coke, blow, crack, bump, C, candy, Charlie,
flake, rock, snow, toot
● PCP - Angel dust, boat, hug, love boat, peace pill
● Mescaline - Buttons, cactus, mesc, peyote
● MDMA - Ecstacy, XTC, Disco Biscuit, Crystal, X, Adam,
Hug drug, Eve
● Heroin - Smack, junk, horse, brown sugar, H, skag, white
horse, china white
● Hydroquinone (Viscodin)
● Fentanyl (Durogesic, Sublimaze)
● Oxycodone (Oxycontin, Percocet)
● Hydromorphone (Dilaudid)

124

You might also like